Вы находитесь на странице: 1из 237

Назва наукового напрямку (модуля): Семестр: 2

Стомат англ задачі 1540


Опис:
2019
ATTENTION!!!! Dear students! If you find a mistake in the test tasks, fill in the form
Перелік питань:
1. A victim with a head trauma in the temporal region has been diagnosed with epidural hematoma.
What artery is most likely to be damaged?
A. * Medial meningeal artery
B. Medial cerebral artery
C. Super?cial temporal artery
D. Anterior meningeal artery
E. Posterior auricular artery
2. A patient was consulted a doctor about loss of taste sensitivity on the tongue root. The doctor
revealed that it is caused by nerve affection. Which nerve is it?
A. * Glossopharyngeal
B. Vagus nerve
C. Facial nerve
D. Superlaryngeal nerve
E. Trigeminal nerve
3. A patient underwent an extraction of a part of a CNS structures by medical indications. As a result of
the extraction the patient developed atony, astasia, intention tremor, ataxy and adiadochokinesis.
Which part of CNS structure had been extracted?
A. * Cerebellum
B. Amygdaloid corpus
C. Hippocamp
D. Basal ganglions
E. Limbic system
4. While performing an inguinal canal operation on hernia a surgeon damaged the male canal’s
contents. What exactly was damaged?
A. * Funiculus spermaticus
B. Urarchus
C. Lig. teres uteri
D. Lig. inguinalе
E. A and B
5. A patient consulted a doctor about a sensation of imbalance which appeared after a trauma. Which
nerve is damaged?
A. * Vestibulocochlear nerve
B. Trigeminal nerve
C. Facial nerve
D. Intermediate nerve
E. Vagus nerve
6. A patient has a right-sided fracture in the region of the frontal third of mandible accompanied by a
haematoma in the region of chin. It is caused by the injury of the following artery:
A. * Mental
B. Inferior labial
C. Lingual
D. Facial
E. Palatine

7. As a result of a cold a patient has the abnormal pain and temperature sensitivity of the frontal 2/3 of
his tongue. Which nerve must have been damaged?
A. * Trigeminus
B. Sublingual
C. Accessory
D. Vagus
E. Glossopharyngeal
8. A 60 year old patient has impaired perception of high-frequency sounds. These changes were caused
by damage of the following auditory analyzer structures:
A. * Main cochlea membrane near the oval window
B. Main cochlea membrane near the helicotrema
C. Eustachian tube
D. Middle ear muscles
E. Tympanic membrane
9. After a trauma of soft tissues in the region of the posterior sorface of medial condyle of humerus a
patient has got a skin prickle of medial forearm surface. Which of the listed nerves is located in the
affected region?
A. * N.ulnaris
B. N.musculocutaneu
C. N.dorsalis scapularis
D. N.subscapularis
E. N.radialis
10. A boy has fallen down from a tree. Now he ?nds it dif?cult to abduct his arm into horizontal position.
Which muscle is most likely to be injured?
A. * M.deltoideus
B. M.triceps brachii
C. M.coracobrachialis
D. M.supinator
E. M.anconeus
11. A female patient with a tumour of pancreas has developed mechanic jaundice resulting from
compression of a bile-excreting duct. Which duct is compressed?
A. * Ductus choledochus
B. Ductus cysticus
C. Ductus hepaticus communis
D. Ductus hepaticus dexter
E. Ductus hepaticus sinister
12. A 28 year old woman has been diagnosed with extrauterine pregnancy complicated by tha fallopian
tube rupture. The blood is most likely to penetrate the following peritoneal space:
A. * Rectouterine
B. Vesicouterine
C. Right mesenteric sinus
D. Left mesenteric sinus
E. Intersigmoid sinus
13. Autonomic abnormalities in the sleep, heat regulation, all kinds of metabolism, diabetes insipidus are
developing in the patient due to grouth of the tumour in the III ventricle of brain. Irritation of the
nucleus of what part of the brain can cause this symptoms?
A. * Hypothalamus
B. Cerebral peduncles (cruces cerebri)

C. Mesencephalic tegmentum
D. Pons cerebelli
E. Medulla
14. The patient with thymoma (thymus gland tumour) has cyanosis, extention of subcutaneous venous
net and edema of the soft tissues of face, neck, upper part of the trunk and upper extremities. What
venous trunk is pressed with tumour?
A. * Superior vena cava
B. External jugular vein
C. Clavicular vein
D. Internal jugular vein
E. Frontal jugular vein
15. A 70 year old female patient was diagnosed with fracture of left femoral neck accompanied by
disruption of ligament of head of femur. The branch of the which following artery is damaged:
A. * Obturator
B. Femoral
C. External iliac
D. Inferior gluteal
E. Internal pudendal
16. A man who was riding the carousel presents with increased heart rate, sweating, nausea. This
condition is caused primarily by the stimulation of the following receptors:
A. * Vestibular ampullar
B. Proprioceptors
C. Vestibular otolithic
D. Auditory
E. Visual
17. In order to prevent massive haemorrhage in the region of oral cavity ?oor it is required to ligate an
artery which is located within Pirogov’s triangle. What artery is it?
A. * Lingual artery
B. Superior thyroid artery
C. Facial artery
D. Ascending pharyngeal artery
E. Maxillary artery
18. As a result of an injury a patient cannot extend his arm at the elbow. This may cause abnormal
functioning of the following muscle:
A. * Musculus triceps brachii
B. Musculus infraspinatus
C. Musculus levator scapulae
D. Musculus teres major
E. Musculus subscapularis
19. After casualty a man has a fracture in the region of the inner surface of the left ankle. What is the
most likely site for the fracture?
A. * Medial malleolus
B. Lower third of the ?bula
C. Astragalus
D. Lateral malleolus
E. Calcaneus

20. A patient was consulted a doctor about being unable to abduct his right arm after a trauma.
Examination revealed that the passive movements were not limited. The patient was found to have
the atrophy of the deltoid muscle. What nerve is damaged?
A. * Axillary
B. Radial
C. Ulnar
D. Median
E. Suprascapular
21. After a trauma of the upper third of the anterior forearm a patient exhibits dif?cult pronation,
weakening of palmar ?exor muscles and impaired skin sensitivity of 1-3 ?ngers. Which nerve has
been damaged?
A. * n. medianus
B. n. musculocutaneus
C. n. ulnaris
D. n. cutaneus antebrachii medialis
E. n. radialis
22. On examination of a patient was found medial strabismus, the inward deviation of the eyeball and
inability to abduct the eyeball outwards. What nerve is damaged?
A. * Abducent
B. Ocular
C. Trochlear
D. Visual
E. Oculomotor
23. As a result of an injury of the knee joint a patient shows the anterior and posterior displacement of
the tibia relative to the femur. What ligaments are damaged?
A. * Cruciate ligaments
B. Arcuate popliteal ligaments
C. Oblique popliteal ligament
D. Interosseous membrane
E. Collateral ligaments
24. To the neurosurgical department was admitted a 54-year-old male complaining of no sensitivity in the
lower eyelid skin, lateral surface of nose, upper lip. On examination the physician revealed the in?
ammation of the second branch of the trigeminal nerve. This branch comes out of the skull through
the following foramen:
A. * Rotundum foramen
B. Lacerum foramen
C. Ovale foramen
D. Spinous foramen
E. Superior orbital ?ssure
25. Examination of a patient with ischemic heart disease revealed the impaired venous blood ?ow in the
area of the cardiac vein running in the anterior interventricular sulcus of heart. What vein is it?
A. * V. cordis magna
B. V. cordis media
C. V. cordis parva
D. V. posterior ventriculi sinistri
E. V. obliqua atrii sinistri
26. For the direct injection of medications into the liver surgeons use the round ligament of liver. This
manipulation involves bougienage (lumen dilatation) of the following vessel:

A. * V. umbilicalis
B. A. umbilicalis
C. Ductus venosus
D. V. porta
E. A. hepatica propria
27. After the diagnostic tests a 40-year-old male has been referred for the lymphography of the thoracic
cavity. The surgeon revealed that the tumor had affected an organ whose lymphatic vessels drain
directly into the thoracic duct. Specify this organ:
A. * Esophagus
B. Trachea
C. Left main bronchus
D. Heart
E. Pericardium
28. After a car accident a 23-year-old male presented to the hospital with a cut wound of the
anteromedial region of shoulder and arterial bleeding. Which artery was damaged?
A. * A. brachialis
B. A. radialis
C. A. axillaris
D. A. subscapularis
E. A. profunda brachii
29. During the operation on the small intestine the surgeon revealed an area of the mucous membrane
with a single longitudinal fold among the circular folds. Which portion of the small intestine is this
structure typical for?
A. * Pars descendens duodeni
B. Pars horizontalis duodeni
C. Pars ascendens duodeni
D. jejunum
E. Distal ileum
30. As a result of a craniocerebral injury a patient has decreased skin sensitivity. What area of the
cerebral cortex may be damaged?
A. * Posterior central gyrus
B. Occipital region
C. Cingulate gyrus
D. Frontal cortex
E. Anterior central gyrus
31. A patient complains of pain in the right lateral abdominal region. Palpation revealed a dense,
immobile, tumor-like formation. A tumor is likely to be found in the following part of the digestive
tube:
A. * Colon ascendens
B. Colon transversum
C. Colon descendens
D. Colon sigmoideum
E. Caecum
32. A surgeon examined the patient and found the injury of the upper third of the kidney. Considering the
syntopy of the left kidney, the intactness of the following organ should be checked at the same time:

A. * Stomach
B. Liver

C. Small intestine
D. Transverse colon
E. Descending colon
33. A specimen of a parenchymal organ shows poorly delineated hexagonal lobules surrounding a central
vein, and the interlobular connective tissue contains embedded triads (a artery, a vein and an
excretory duct). What organ is it?
A. * Liver
B. Pancreas
C. Thymus
D. Spleen
E. Thyroid
34. After resection of the middle third of the femoral artery obliterated by a thrombus the limb is
supplied with blood through the bypasses. What artery plays the main part in the restoration of the
blood ?ow?
A. * Deep femoral artery
B. Super?cial iliac circum?ex artery
C. Descending genicular artery
D. Super?cial epigastric artery
E. External pudendal artery
35. When examining a patient, the doctor revealed a tumor of the bronchus which borders on the aorta.
Which bronchus is affected?
A. * Left principal
B. Right principal
C. Right upper lobar
D. Left upper lobar
E. Middle lobar
36. A patient complains that at the bare mention of the tragic events that once occurred in his life he
experiences tachycardia, dyspnea and an abrupt rise in blood pressure. What structures of the CNS
are responsible for these cardiorespiratory reactions in this patient?
A. * Cerebral cortex
B. Cerebellum
C. Lateral hypothalamic nuclei
D. Speci?c thalamic nuclei
E. Quadrigemina of mesencephalon
37. While examining foot blood supply a doctor checks the pulsation of a large artery running in the
separate ?brous channel in front of articulation talocruralis between the tendons of long extensor
muscles of hallux and toes. What artery is it?
A. * A. dorsalis pedis
B. A. tibialis anterior
C. A. tarsea medialis
D. A. tarsea lateralis
E. A. ?bularis
38. A comminuted fracture of infraglenoid tubercle caused by shoulder joint injury has been detected
during X-ray examination of a patient. What muscle tendon attached at this site has been damaged?
A. * Long head of m. triceps brachii
B. Long head of m. biceps brachii
C. Medial head of m. triceps brachii
D. Lateral head of m. triceps brachii

E. Short head of m. biceps brachii


39. A 40-year-old patient has ulcer perforation in the posterior wall of stomach. What anatomical
structure will blood and stomach content leak to?
A. * Bursa omentalis
B. Bursa praegastrica
C. Right lateral channel (canalis lateralis dexter)
D. Left lateral channel (canalis lateralis sinister)
E. Bursa hepatica
40. A female patient has facial neuritis that has caused mimetic paralysis and hearing impairment.
Hearing impairment results from the paralysis of the following muscle:
A. * Stapedius muscle
B. Anterior auricular muscle
C. Superior auricular muscle
D. Posterior auricular muscle
E. Nasal muscle
41. The receptors under study provide transfer of information to the cortex without thalamic
involvement. Specify these receptors:
A. * Olfactory
B. Tactile
C. Gustatory
D. Visual
E. Auditory
42. During an animal experiment, surgical damage of certain brain structures has caused deep prolonged
sleep. What structure is most likely to cause such condition, if damaged?
A. * Reticular formation
B. Basal ganglion
C. Red nuclei
D. Hippocampus
E. Cerebral cortex
43. A patient complains of acute pain attacks in the right lumbar region. During examination the
nephrolithic obturation of the right ureter in the region between its abdominal and pelvic segments
has been detected. What anatomical boundary exists between those two segments?
A. * Linea terminalis
B. Linea semilunaris
C. Linea arcuata
D. Linea transversa
E. Linea inguinalis
44. A male patient complains of skin insensitivity of inferior eyelid, external lateral surface of nose and
upper lip. A doctor in the course of examination has revealed in?ammation of the second divisin of
trigeminal nerve. What cranial foramen does this branch go through?
A. * Infraorbital
B. Lacerum
C. Superior orbital ?ssure
D. Spinosum
E. Oval
45. X-ray examination of a patient allowed diagnosing a tumor in the superior lobe of the right lung.
There is a probability of metastases spread to the following lymph nodes:
A. * Sternal

B. Inferior mediastinum
C. Anterior mediastinum
D. Axillary
E. Deep lateral cervical
46. In course of invasive abdominal surgery a surgeon has to locate the origin of the mesenteric root.
Where it is normally localized?
A. * Duodenojejunal ?exure
B. Right ?exure of colon
C. Left ?exure of colon
D. Sigmoid colon
E. Right mesenteric sinus
47. As a result of a road accident in a 37-year-old female victim developed urinary incontinence. What
segments of the spinal cord had been damaged?
A. * S2 ? S4
B. Th1 ? Th5
C. L1 ? L2
D. Th2 ? Th5
E. Th1 ? L1
48. A 50 y.o. patient was admitted to the hospital with complaints about pain behind his sternum,
asphyxia during physical activity. Angiography revealed pathological changes in the posterior
interventricular branch of the right coronary artery. What heart parts are affected?
A. * Posterior wall of the right and left ventricles
B. Left atrium
C. Anterior wall of the right and left ventricles
D. Right atrium
E. Right atrioventricular valve
49. A patient in three weeks after acute myocardial infarction suffers from pain in the heart and joints and
pneumonia. What is the main mechanism of development of post-infarction Dressler’s syndrome?

A. * Autoimmune inflammation
B. Ischemia of myocardium
C. Resorption of enzymes from necrotized area of myocardium
D. Secondary infection
E. Vessels ' thrombosis
50. During the fetal period of the development in the vascular system of the fetus large arterial (Botallo's)
duct is functioning which converts into lig. arteriosum after birth. What anatomical formations does
this duct connect between each other?
A. * Pulmonary trunk and aorta
B. Right and left auricle
C. Aorta and inferior vena cava
D. Pulmonary trunk and superior vena cava
E. Aorta and superior vena cava
51. Examination of a 2-year-old child revealed physical developmental lag, the child often has
pneumonias. The child was diagnosed with nonclosure of ductus arteriosus. Haemodynamics disorder
was caused by the intercommunication of the following vessels:
A. * Aorta and pulmonary trunk
B. Pulmonary trunk and pulmonary veins
C. Superior cava and aorta

D. Superior cava and pulmonary trunk


E. Aorta and pulmonary veins
52. The patient has come to the hospital from the smelting workshop in the condition of hyperthermia.
What is the direct cause of loss of consciousness at the heat stroke?
A. * Decreased brain blood supply
B. Arterial pressure drop
C. Increased water loss through sweating
D. Decrease of heart output
E. Dilatation of peripheral vessels
53. While preparing a patient to the operation the heart chambers' pressure was measured. In one of them
the pressure changed during one heart cycle from 0 to 120 mm Hg. What chamber of heart is it?
A. * Left ventricle
B. Right ventricle
C. Right atrium
D. Left atrium
E. All of these
54. A 19 year-old patient was diagnosed with appendicitis and was hospitalized. The surgical operation
on appendectomy is to be performed. What artery must be fixed to stop bleeding during the surgical
operation?
A. * The ileocolic artery
B. The colica dextra
C. The colica media
D. The colica sinistra
E. The iliac
55. A 70 year old female patient was diagnosed with fracture of left femoral neck accompanied by
disruption of ligament of head of femur. The branch of the following artery is damaged:
A. * Obturator
B. Femoral
C. External iliac
D. Inferior gluteal
E. Internal pudendal
56. A woman underwent an operation at extrauterine (tubal) pregnancy. In course of the operation the
surgeon should ligated the branches of the following arteries:
A. * Uterine and ovarian
B. Superior cystic and ovarian
C. Inferior cystic and ovarian
D. Uterine and superior cystic
E. Uterine and inferior cystic
57. After resection of the middle third of femoral artery obliterated by a thromb the lower extremity is
supplied with blood due to the surgical bypass. Name an artery that plays the main role in
reestablishment of blood flow:
A. * Deep femoral artery
B. Superficial circumflex artery of hip bone
C. Descending genicular artery
D. Superficial epigastric artery
E. Deep external pudendal artery

58. After trauma a 44-year-old patient had a rupture of left palm muscle tendons and of the superficial
blood vessels. After operation and removal of the most part of the necrotically changed muscle tissue
the bloodstream was normalized. What vessels have helped with restoration of bloodstream?
A. * Arcus palmaris profundus
B. Arcus palmaris superficialis
C. Aa. digitales palmares communes
D. Aa. metacarpeae palmares
E. Aa. perforantes
59. On examination of a road accident victim a doctor revealed left clavicle fracture and disturbed blood
circulation in an extremity (no pulsing of radial artery). What is the most probable cause of blood
circulation disturbance?
A. * Compression of subclavian artery
B. Compression of axillary artery
C. Compression of subclavian vein
D. Compression of vertebral artery
E. Compression of axillary vein
60. While performing an operation in the area of axillary cavity a surgeon has to define an arterial vessel
surrounded by fascicles of brachial plexus. What artery is it?
A. * A. axillaris
B. A. vertebralis
C. A. transversa colli
D. A. profunda brachii
E. A. subscapularis
61. A 54-year-old man was admitted to the hospital with complaints of pain in the right subcostal region,
vomiting with blood. Objectively: enlarged liver, varicose veins in the stomach and esophagus.
Disfunction of what vessel is likely to be?
A. * Vena porta
B. Aorta abdominalis
C. Vena hepatica
D. Vena cava superior
E. Vena cava inferior
62. A patient complains about edema of legs, skin cyanosis, and small ulcers on one side of the lateral
condyle. Examination revealed a swelling, enlarged veins, formation of nodes. The pathological
process has started in the following vein:
A. * V. saphena parva
B. V. saphena magna
C. V. femoralis
D. V. profunda femoris
E. V. iliaca externa
63. A patient has tissue ischemia below the knee joint accompanied with intermittent claudication. What
artery occlusion should be suspected?
A. * Popliteal artery
B. Peroneal artery
C. Posterior tibial artery
D. Anterior tibial artery
E. Proximal part of femoral artery
64. A woman was consulted by a doctor about swelling and tenderness of the lower extremity, swollen
veins and nodes on the medial surface of thigh. Which vein was affected?

A. * Greater saphenous
B. Lesser saphenous
C. Femoral
D. Popliteal
E. Tibial
65. Where should be the catheter for evacuation of the lymph from the thoracic lymph duct is inserted?
A. * To the left venous angle
B. To the right venous angle
C. To the superior vena cava
D. To the inferior vena cava
E. To the left inguinal vein
66. A man suffering from osteochondrosis got acute pain in the abdominal muscles (lateral and anterior).
During objective examination a physician diagnosed increased pain sensitivity of skin in the
hypogastric region. This pain might be caused by affection of the following nerve:
A. * Iliohypogastric
B. Sciatic
C. Obturator
D. Femoral
E. Genitofemoral
67. A man with a stab wound in the area of quadrilateral foramen applied to a doctor. Examination
revealed that the patient was unable to draw his arm aside from his body. What nerve is most
probably damaged?
A. * N. axillaris
B. N. medianus
68. A patient has an exudative pleuris. At what level should the pleural puncture along the posterior
axillary line be done?
A. * ІХ intercostal space
B. VІІІ intercostal space
C. VІІ intercostal space
D. ХІ intercostal space
E. VІ intercostal space
69. A patient has lost skin sensitivity in the region of the medial surface of his shoulder. This is the result
of dysfunction of the following nerve:
A. * Medial brachial cutaneous nerve
B. Medial antebrachial cutaneous nerve
C. Radial nerve
D. Ulnar nerve
E. Axillary nerve
70. A patient with neuritis of femoral nerve has disturbed flexion of thigh as well as disturbed crus
extension in the knee joint. What muscle's function is disturbed?
A. * Quadriceps femoris muscle
B. Biceps femoris muscle
C. Triceps surae muscle
D. Semitendinous muscle
E. Semimembranous muscle
71. A patient doesn't extend his knee joint, there is no knee-jerk reflex, skin sensitivity of the anterior
femoral surface is disturbed. What nerve structure is damaged?
A. * Femoral nerve

B. Superior gluteal nerve


C. Big fibular nerve
D. Obturator nerve
E. Inferior gluteal nerve
72. A woman suffering from osteochondrosis felt acute pain in her humeral articulation that became
stronger when she abducted her shoulder. These symptoms might be caused by damage of the
following nerve:
A. * Axillary nerve
B. Subscapular nerve
C. Dorsal scapular nerve
D. Subclavicular nerve
E. Throracodorsal nerve
73. After a road accident a driver was delivered to the hospital with an injury of the medial epicondyle of
humerus. What nerve might be damaged in this case?
A. * n. ulnaris
B. n. radialis
C. n. axillaris
D. n. muscolocutaneus
E. n. medianus
74. Victim has elbow joint trauma with avulsion of medial epicondyle of humerus. What nerve can be
damaged in this trauma?
A. * Ulnar
B. Radial
C. Musculocutaneous nerve
D. Cardiac cutaneous nerve
E. Medial cutaneous nerve of forearm
75. A patient has a malignisation of thoracic part of esophagus. What lymphatic nodes are regional for
this organ?
A. * Anulus lymphaticus cardiae
B. Nodi lymphatici paratrachealis
C. Nodi lymphatici prevertebralis
D. Nodi lymphatici pericardiales laterales
E. Nodi lymphatici mediastinales posteriores
76. While palpating mammary gland of a patient a doctor revealed an induration in form of a node in the
inferior medial quadrant. Metastases may extend to the following lymph nodes:
A. * Parasternal
B. Posterior mediastinal
C. Profound lateral cervical
D. Bronchopulmonary
E. Superior diaphragmal
77. Three separate bones connected with cartilage in the area of pelvis cavity are noticed on the X-ray of
the pelvis. What are these bones?
A. * Iliac, pubic, sciatic
B. Pubic, sciatic, femoral
C. Sciatic, femoral, sacral
D. Iliac, sacral, coccyx
E. Sacral, pubic, coccyx

78. A 45-year-old man fell on the right knee and felt the acute pain in the joint. On examination: severe
edema on the anterior surface of the knee joint. Crunching sounds are heard while moving the joint.
Which bone is destroyed?
A. * Knee-cap
B. Neck of the thigh bone
C. Left epicondyle of the thigh
D. Right epicondyle of the thigh
E. Head of the thigh bone
79. During the operation on the hip joint of a 5-year-old child her ligament was damaged which caused
bleeding. What ligament was damaged?
A. * Capitis femoris
B. Perpendicular of the acetabule
C. Iliofemoral
D. Pubofemoral
E. Ischiofemoral
80. A 6-year-old child fell on the cutting object and traumatized soft tissues between tibia and fibula.
What kind of bone connection was injured?
A. * Interosseus membrane
B. Suture
C. Ligament
D. Fontanel
E. Gomphosis
81. It is necessary to take the cerebrospinal fluid from a patient with suspected inflammation of brain
tunics. Diagnostic puncture was performed between the arches of the lumbar vertebras. During the
puncture the needle went through the following ligament:
A. * Yellow (flaval)
B. Iliolumbar
C. Anterior longitudinal
D. Posterior longitudinal
82. An old woman was hospitalized with acute pain, edema in the right hip joint; the movements in the
joint are limited. Which bone or part of it was broken?
A. * The neck of the femur
B. The body of the femur
C. Condyle of the femur
D. Pubic bone
E. Ischial bone
83. A patient has been diagnosed with a compressive fracture of a lumbar vertebra. As a result he has a
considerable increase in curvature of the lumbar lordosis. Which ligament damage can induce
such changes in the spine curvature?
A. * Anterior longitudinal ligament
B. Posterior longitudinal ligament
C. Yellow ligament
D. Iliolumbar ligament
E. Interspinous ligament
84. During the operation on the hip joint of a 5-year-old child her ligament was damaged which caused
bleeding. What ligament was damaged?
A. * Capitis femoris
B. Perpendicular of the acetabule

C. Iliofemoral
D. Pubofemoral
E. Ischiofemoral
85. Examination of a patient revealed an abscess of pterygopalatine fossa. Where can the infection spread
to unless the disease is managed in time?
A. * To the orbit
B. To the interpterygoid space
C. To the frontal sinus
D. To the subgaleal temporal space
E. To the tympanic cavity
86. Examination of a 6-month-old child revealed a delay in closure of the occipital fontanelle. When
should it normally closed?
A. * Until 3 months
B. Before the child is born
C. Until 6 months
D. Until the end of the first year of life
E. Until the end of the second year of life
87. A 25 year old patient was examined by a medical board. Examination revealed pathology of chest.
Transverse dimensions were to small and the sternum was strongly protruding. What chest type is it?
A. * Keeled chest
B. Funnel chest
C. Flat chest
D. Cylindrical chest
E. Barrel chest
88. A 5-year-old child was admitted to the otorhinolaryngological department with diagnosis -
suppurative inflammation of the middle ear. Disease started from the inflammation of the
nasopharynx. Through what canal of the temporal bone did the infection get into the tympanic
cavity?
A. * Musculotubarius canal
B. Canaliculus tympanicus
C. Carotid canal
D. Canaliculus chordal tympani
E. Canaliculi caroticotympanici
89. A patient arrived to the oral surgery department with dislocation of temporomandibular joint and
injury of its main ligament. Name this ligament:
A. * Lateral
B. Mandibular
C. Styloid-mandibular
D. Pterygoid-mandibular
E. Medial
90. The operative dentistry department admitted a newborn girl who choked during sucking.
Examination revealed cleft palate arising from non-union of the middle frontal process and maxillary
process of the I-st branchial arch. The cleft was located in the palate between:
A. * Os incisivum et processus palatinus maxillae
B. Processus palatinus maxillae dextrae et sinistrae
C. Lamina horizontalis os palatinum dextrum et sinistrum
D. Processus palatinus maxillae et lamina horizontalis os palatinum
E. In the region of canalis incisivus

91. An eye trauma caused soft tissues infection of orbit. Through what anatomical formation can the
infection penetrate into the middle cranial fossa?
A. * Through the superior orbital fissure
B. Through the anterior ethmoidal foramen
C. Through the posterior ethmoidal foramen
D. Through the inferior orbital fissure
E. Through the zygomatic orbital foramen
92. Chronic rhinitis was complicated by inflammation of frontal sinus. What nasal meatus did the
infection get into this sinus through?
A. * Middle
B. Inferior
C. Common
D. Superior
E. Nasopharyngeal
93. Trauma of occipital region of head resulted in crack fracture in the region of transverse sinus. What
part of occipital bone is damaged?
A. * Squama
B. Left lateral
C. Right lateral
D. Base
E. Condyle
94. A victim of a road accident has an abruption of a part of mandibular angle, displacement of fragment
backwards and upwards. What ligament is responsible for this displacement?
A. * Stylomandibular
B. Intraarticular
C. Lateral
D. Sphenomandibular
E. Pterygomandibular
95. A 69 year old patient has got an abscess of frontal lobe as a result of purulent infection in nasal
cavity. What anatomical formation did the infection penetrate through?
A. * Foraminae cribrosae
B. Foramen ovale
C. Foramen ethmoidalae posterior
D. Foramen sphenopalatinum
E. Foramen rotundum
96. X-ray examination revealed an accumulation of suppuration in maxillary sinus. Into what nasal
meatus excretes the suppuration?
A. * Middle nasal
B. Nasopharyngeal
C. Inferior nasal
D. Superior nasal
E. Common nasal
97. A 5 year old child was admitted to the hospital with suppurative inflammation of middle ear
(tympanitis). It began with the inflammation of nasopharynx. What canal of temporal bone did the
infection get into tympanic cavity through?
A. * Musculotubarius canal
B. Canaliculus of chorda tympani
C. Caroticotympanic foramina

D. Tympanic canaliculus
E. Carotid canal
98. A boxer who got a punch in the region of temporomandibular joint has a traumatic dislocation of
mandible. Displacement of what articular surfaces will overstep the limits of physiological norm?
A. * Head of mandible and mandibular fossa
B. Coronoid process and pterygoid fossa
C. Coronoid process and submandibular fossa
D. Head of mandible and submandibular fossa
E. Neck of mandible and submandibular fossa
99. Underdevelopment of which parts of facial skeleton in the embryonal period is the reason for such a
malformation as cleft palate?
A. * Palatine processes
B. Frontal processes
C. Frontal and maxillary processes
D. Mandibular processes
E. Mandibular and palatine processes
100. It is necessary to take the cerebrospinal fluid from a patient with suspected inflammation of brain
tunics. Diagnostic puncture was performed between the arches of the lumbar vertebras. During the
puncture the needle went through the following ligament:
A. * Yellow (flava)
B. Iliolumbar
C. Anterior longitudinal
D. Posterior longitudinal
E. Intertransverse
101. A patient has an inflammation in the pterygopalatine fossa. The infection has spread into the nasal
cavity. Which anatomical structure has the infection spread through?
A. * Foramen sphenopalatinum
B. Foramen rotundum
C. Canalis palatinus major
D. Canalis palatinus minor
E. Canalis ptherygoideus
102. A doctor asked a patient to make a deep exhalation after a normal inhalation. What muscles contract
during such exhalation?
A. * Abdominal muscles
B. External intercostal muscles
C. Diaphragm
D. Trapezius muscles
E. Pectoral muscles
103. Young man felt sharp pain in the back during active push up on the horizontal bar. Objectively: pain
while moving of upper extremity, limited pronation and adduction functions. Sprain of what muscle
is presented?
A. * М. latissimus dorsi
B. М. levator scapulae
C. М. romboideus major
D. М. trapezius
E. М. subscapularis
104. A 55-year-old patient was hospitalized in result of the trauma of the medial group of femoral
muscles. What kind of movements is the patient unable to do?

A. * Adduction of femur
B. Abduction of femur
C. Flexion of femur
D. Extension of femur
E. Suppination of femur
105. A 45-year-old man applied to the traumatology department because of shoulder trauma. Objectively:
flexibility, reduction and pronation functions of the shoulder are absent. What muscle was injured?
A. * Teres major muscle
B. Subscapular muscle
C. Teres minor muscle
D. Infraspinous muscle
E. Supraspinous muscle
106. A 7-year-old child can't abduct the arm, raise it to the horizontal level. He can raise the hand to the
face only with dorsal side with abduction of the shoulder (with the help of supraspinous muscle ) -
"bugler" arm. Active function of what muscle is absent?
A. * Deltoid
B. Infraspinous
C. Pectoral major
D. Teres minor
E. Teres major
107. A patient complained about being unable to adduct and abduct fingers in the metacarpophalangeal
articulations towards and away from the 3rd finger. Which muscles' function is impaired?
A. * Interosseous muscles
B. Lumbrical muscles
C. Flexor digitorum brevis
D. Flexor digitorum longus
E. Extensors
108. A 35 year old man with a trauma of his left hand was admitted to the traumatology department.
Objectively: cut wound of palmar surface of left hand; middle phalanxes of II–V fingers don't bend.
What muscles are damaged?
A. * Flexor digitorum superficialis
B. Flexor digitorum profoundus
C. Lumbrical muscles
D. Palmar interosseous muscles
E. Dorsal interosseous muscles
109. A 38-year-old patient came to a traumatology centre and complained about an injury of his right
hand. Objectively: the patient has a cut wound in the region of the thenar eminence on the right hand;
distal phalanx of the I finger cannot be flexed. What muscle was injured?
A. * Flexor pollicis longus
B. Flexor pollicis brevis
C. Abductor pollicis brevis
D. Opponens pollicis
E. Abductor pollicis
110. A patient with neuritis of femoral nerve has disturbed flexion of thigh as well as disturbed crus
extension in the knee joint. What muscle's function is disturbed?
A. * Quadriceps femoris muscle
B. Biceps femoris muscle
C. Triceps femoris muscle

D. Semitendinous muscle
E. Semimembranous muscle
111. A patient has difficulties with hand movement. Examination revealed inflammation of common
synovial sheath of flexor muscles. It is known from the patient's anamnesis that he got a stab wound
of finger a week ago. Which finger was most probably damaged?
A. * Digitus minimus
B. Pollex
C. Digitus medius
D. Index
E. Digitus annularis
112. As a result of an accident a patient has intense painfulness and edema of the anterior crus surface;
dorsal flexion of foot is hindered. Function of which crus muscle is most likely to be disturbed?
A. * M.tibialis anterior
B. M.flexor digitorum longus
C. M.flexor hallucis longus
D. M.peroneus longus
E. M.peroneus brevis
113. A patient has a deep cut wound on the posterior surface of his shoulder in its middle third. What
muscle might be injured?
A. * Triceps brachii muscle
B. Biceps brachii muscle
C. Anconeus muscle
D. Brachialis muscle
E. Coracobrachialis muscle
114. After a trauma a patient lost ability of elbow extension. This might have been caused by dysfunction
of the following main muscle:
A. * m. triceps brachii
B. m. subscapularis
C. m. teres major
D. m. infraspinatus
E. m. levator scapulae
115. A patient was admitted to the surgical department with inguinal hernia. During the operation the
surgeon performs plastic surgery on posterior wall of inguinal canal. What structure forms this wall?
A. * Transverse fascia
B. Aponeurosis of abdominal external oblique muscle
C. Inguinal ligament
D. Loose inferior edge of transverse abdominal muscle
E. Peritoneum
116. A man with an injury of the dorsal area of his neck was admitted to the resuscitation department.
What muscle occupies this area?
A. * M.trapezius
B. M.sternocleidomastoideus
C. M.latissimus dorsi
D. M.rhomboideus minor
E. M.scalenus anterior
117. A patient with a knife wound in the left lumbal part was delivered to the emergency hospital. In
course of operation a surgeon found that internal organs were not damaged but the knife injured one
of pelvic muscles. What muscle is it?

A. * Psoas major muscle


B. Iliac muscle
C. Erector spine muscle
D. Internal oblique abdominal muscle
E. External oblique abdominal muscle
118. Usually the intravenous injection is done into median cubital vein because it is slightly movable
due to fixation by the soft tissues. What does it fix in the cubital fossa?
A. * Aponeurosis of biceps muscle
B. Tendon of the triceps muscle
C. Brachialis muscle
D. Brachioradialis muscle
E. Anconeus muscle
119. A patient displays abnormal retraction of his lower jaw as a result of trauma in the region of
mandibular coronal process. What muscle is most likely to be damaged?
A. * M.temporalis
B. M.masseter
C. M.pterygoideus lateralis
D. M.pterygoideus medialis
E. M.levator anguli oris
120. A 5 year old child suffers from the neck deformity. Clinical examination revealed such symptoms:
apparent flexion of head to the left, his face is turned right, passive movements of the head to the
right are restricted. What muscle's development was disturbed in this case?
A. * Sternocleidomastoid
B. Trapezius
C. Splenius capitis muscle
D. Sternohyoid
E. Longus capitis muscle
121. A 38-year-old patient came to a traumatology centre and complained about an injury of his right
hand. Objectively: the patient has a cut wound in the region of the thenar eminence on the right hand;
distal phalanx of the I finger cannot be flexed. What muscle was injured?
A. * Flexor pollicis longus
B. Flexor pollicis brevis
C. Abductor pollicis brevis
D. Opponens pollicis
E. Abductor polliis
122. During manipulations aimed at treatment of mandible dislocation a physician should pay particular
attention to a muscle that pulls a capsule and interarticular disc of temporomandibular articulation
exteriorly. What muscle is it?
A. * M. pterygoideus lateralis
B. M. masseter
C. M. pterygoideus medialis
D. M. temporalis
E. M. mylohyoideus
123. After a trauma a patient lost ability of elbow extension. This might have been caused by dysfunction
of the following main muscle:
A. * m. triceps brachii
B. m. subscapularis
C. m. teres major

D. m. infraspinatus
E. m. levator scapulae
124. A patient presents with dysfunction of shin muscles. He cannot stand on tiptoes. Which muscle is
affected?
A. * M. triceps surae
B. M. tibialis posterior
C. M. extensor digitorum longus
D. M. flexor digitorum longus
E. M. tibialis anterior
125. When processing a molar tooth with a dental cutter the dentist has by accident deeply wounded the
patient's cheek and damaged not only the mucosa but also a muscle. Which muscle was damaged?
A. * Buccal muscle
B. Zygomatic major muscle
C. Masseter muscle
D. Orbicularis oris muscle
E. Mylohyoid muscle
126. A patient with a knife wound in the left lumbal part was delivered to the emergency hospital. In
course of operation a surgeon found that internal organs were not damaged but the knife injured one
of muscles of pelvis. What muscle is it?
A. * Psoas major muscle
B. Iliac muscle
C. Erector spine muscle
D. Internal oblique abdominal muscle
E. External oblique abdominal muscle
127. Children often have heavy nasal breathing resulting from excessive development of lymphoid tissue
of pharyngeal mucous membrane. What tonsils growth may cause this effect?
A. * Tonsilla pharyngea
B. Tonsilla palatina
C. Tonsilla lingualis
D. Tonsilla tubaria
E. All above mentioned tonsils
128. A patient with a stab wound of the anterior stomach wall is in surgical care. What formation of
abdominal cavity did the stomach contents get into?
A. * Pregastric bursa
B. Omental bursa
C. Hepatic bursa
D. Left mesenteric sinus
E. Right mesenteric sinus
129. A man with internal abdominal right side injury and suspicion of liver rupture was admitted to the
traumatological department. In what peritonial structure will blood accumulate?
A. * Excavatio rectovesicalis
B. Bursa omentalis
C. Recessus intersigmoideus
D. Fossa ischiorectalis
E. Recessus duodenalis inferior
130. A mother of a newborn complains of her baby's constant belching with undigested milk. Which
developmental anomaly is it an evidence of?
A. * Esophageal atresia

B. Labium leporium
C. Faux lupinum
D. Anal atresia
E. Esophageal fistula
131. During the endoscopy the inflammation of a major papilla of the duodenum and the disturbances of
bile secretion were found. In which part of duodenum were the problems found?
A. * Descendent part
B. Ascendant part
C. Bulb
D. Upper horizontal part
E. Lower horizontal part
132. Obturative jaundice developed in a 60-year-old patient because of malignant tumour of the greater
papilla of the duodenum. Lumen of what anatomical structure is pressed with tumour?
A. * Hepatopancreatic ampulla
B. Cystic duct
C. Common hepatic duct
D. Right hepatic duct
E. Left hepatic duct
133. A 53-year-old female patient was diagnosed with liver rupture resulting from a blunt abdominal
injury. The blood will be assembled in the following anatomical formation:
A. * Rectouterine pouch
B. Vesicouterine pouch
C. Right mesenteric sinus
D. Omental bursa
E. Left mesenteric sinus
134. A patient was admitted to the surgical department with suspected inflammation of Meckel's
diverticulum. What part of bowels should be examined in order to discover the diverticulum in course
of an operation?
A. * Ileum
B. Duodenum
C. Jejunum
D. Caecum
E. Colon ascendens
135. A surgeon has to find the common hepatic duct during the operative intervention on account of
concrements in the gall ducts. The common hepatic duct is located between the leaves of:
A. * Hepatoduodenal ligament
B. Hepatogastric ligament
C. Hepatorenal ligament
D. Round ligament of liver
E. Venous ligament
136. A 45-year-old patient was admitted to the surgical department with complaints of abrupt sharp pain in
the epigastric region. After examination it was diagnosed: perforated ulcer of the posterior wall of the
stomach. Where did content of the stomach flow out while perforation?
A. * To the omental bursa
B. To the liver bursa
C. To the proventriculus sack
D. To the left mesenteric sinus
E. To the right mesenteric sinus

137. In case of a penetrating wound of the anterior abdominal wall the wound tract went above the lesser
curvature of stomach. What peritoneum formation is most likely to be injured?
A. * Ligamentum hepatogastricum
B. Ligamentum gastrocolicum
C. Ligamentum hepatoduoduodenale
D. Ligamentum hepatorenale
E. Ligamentum triangulare sinistrum
138. ?While of oral cavity examination the dentist revealed the formation of the first big cheekteeth on the
lower jaw of a child. How old is this child?
A. * 6-7 years old
B. 4-5 years old
C. 8-9 years old
D. 10-11 years old
E. 12-13 years old
139. X-ray examination revealed an accumulation of suppuration in maxillary sinus. Into what nasal
meatus excretes the suppuration?
A. * Median nasal
B. Nasopharyngeal
C. Inferior nasal
D. Superior nasal
E. Common nasal
140. A 17 year old student pressed out a pustule in the medial angle of eye. In 2 days she was taken to the
institute of neurosurgery with thrombosis of cavernous sinus. Through what vein did the infection get
into this sinus?
A. * V.angularis
B. V.maxillarіs
C. V.profunda faciei
D. V.transversa faciei
E. V.diploicae frontalis
141. A 28 year old man with cut wound of frontal skin was admitted to the hospital. A vessel that supplies
blood to the frontal part of head was ligated in order to stop bleeding. What vessel was ligated?
A. * A.supraorbitalis
B. A.infraorbitalis
C. A.angularis
D. A.dorsalis nasi
E. A.temporalis superficialis
142. A 60 year old patient has problems with formation and moving of food mass, it disturbs eating
process. His tongue is stiff, speaking is impossible. What nerve is damaged?
A. * XII
B. V
C. IX
D. XI
E. VII
143. A patient complains of aching gums and maxillary teeth. What nerve is inflamed?
A. * I branch of the V pair
B. III branch of the V pair
C. I branch of the V pair

D. Sublingual
E. Accessory
144. A patient consulted a doctor about the inflammation of the ethmoid bone cells (ethmoiditis).
Examination revealed the disorder of blood supply to the bone. The ethmoidal cells are normally
supplied with blood by the branches of the following artery:
A. * A. ophthalmica
B. A. infraorbitalis
C. A. facialis
D. A. cerebri anterior
E. A. transversa faciei
145. A patient consulted dental surgeon about an injury of submandibular triangle. During the wound
cleansing the surgeon found that the artery leading to the soft palate is damaged. What artery is
damaged?
A. * A. palatina ascendens
B. A. palatina descendens
C. A. sphenopalatina
D. A. pharingea ascendens
E. A. facialis
146. A patient has air embolism as a result of a skin injury in the middle portion of the
sternocleidomastoid muscle. Which cervical vein was injured?
A. * External jugular vein
B. Anterior jugular vein
C. Internal jugular vein
D. Posterior auricular vein
E. Transverse cervical vein
147. A patient has assymetric face, it is especially noticeable during active muscle contraction. What nerve
may be damaged?
A. * Facial (motor unit)
B. Trigeminal, І branch
C. TrigeminaІ, IІ branch
D. Trigeminal, ІІІ branch
E. Sublingual
148. A patient has difficulties with jaw joining when he is chewing. There is partial atrophy of masticatory
muscles situated below the zygomatic arch. What nerve branches do these muscles innervate?

A. * N. mandibularis
B. N. maxillaris
C. N. alveolaris inferior
D. Nn. alveolares superiores
E. N. infraorbitalis
149. A patient has secretory dysfunction of the submandibular salivary galnd. Which nerve is responsible
for its vegetative innervation?
A. * Chorda tympani
B. N. auriculotemporalis
C. N. mandibularis
D. N. petrosus major
E. N. petrosus minor
150. A patient suffers from middle ear inflammation (otitis). He complains also of disordered test
sensation in the anterior tongue part. What nerve is damaged?
A. * N. facialis
B. N. trigeminus
C. N. vestibulo-cochlearis
D. N. vagus
E. N. glossopharyngeus
151. A patient with cancer of the back of tongue had an intensive bleeding as a result of tumor spread to
the dorsal artery of tongue. What vessel should be ligated in order to stop bleeding?
A. * Lingual artery
B. Dorsal artery of tongue
C. Deep artery of tongue
D. Facial artery
E. Ascending pharyngal artery
152. A woman in grave condition was admitted to a hospital with thediagnosis of the hemorrhagic stroke
in the region of frontal part of the right cerebral hemisphere. The damage of what artery most likely
causedthis condition?
A. * A. сerebri anterior
B. A. cerebri posterior
C. A. communicans anterior
D. A. cerebri media
E. A. communicans posterior
153. After an operation a patient's sensitivity of front and lateral surface of neck has reduced. What nerve
is damaged?
A. * N. transversus colli
B. N. auricularis magnus
C. Nn. supraclaviculares
D. N. occipitalis minos
E. N. phrenicus
154. An 18-year-old man was delivered to the hospital after a road accident. Examination at the
traumatological department revealed multiple injuries of soft tissues of face in the region of the
medial eye angle. The injuries caused massive haemorrhage. What arterial anastomosis might have
been damaged in this region?
A. * a. carotis externa et a. carotis interna
B. a. carotis externa et a. subclavia
C. a. carotis interna eta. subclavia
D. a. subclavia et a. ophthalmica
E. a. carotis interna et a. ophthalmica
155. During ablation of the nose wing lypoma a dentist injured a vessel, that caused a saphenous
hematoma. What vessel was damaged?
A. * А.facialis
B. А.maxillaris
C. А.supraorbitalis
D. А.infraorbitalis
E. А.angularis
156. In order to make a functional complete denture the left superior canine of a patient should be
extracted. After the infraorbital anesthesia the patient got a rapidly growing hematoma in the front
part of face. It was found that the injured artery is a branch of:
A. * А. maxillaris
B. А. alveolaris inferior
C. А. temporalis superficialis
D. А. ophthalmica
E. А. labialis superior
157. Surgical approach to the thyroid gland from the transverse (collar) approach involves opening of
interaponeurotic suprasternal space. What anatomic structure localized in this space is dangerous to
be damaged?
A. * Jugular venous arch
B. External jugular vein
C. Subclavicular vein
D. Inferior thyroid arthery
E. Superior thyroid arthery
158. The patient with thymoma (thymus gland tumour) has cyanosis, extention of subcutaneous venousnet
and edema of the soft tissues of face, neck, upper part of the trunk and upper extremities. What
venous trunk is pressed with tumour?
A. * Superior vena cava
B. External jugular vein
C. Clavicular vein
D. Internal jugular vein
E. Frontal jugular vein
159. When a patient puts his tongue out the tip of it deflects to the left. Motor innervation of what cranial
nerve is disturbed in this case?
A. * N. hypoglossus dexter
B. N. glossopharyngeus dexter
C. N. vagus dexter
D. N. trigeminus sinister
E. N. facialis sinister
160. ?A 19 year-old patient was diagnosed with appendicitis and was hospitalized. The surgical operation
on ablating appendix vermiformis is to be performed. What artery must be fixed to stop bleeding
during the surgical operation?
A. * The ileocolic artery
B. The colica dextra
C. The colica media
D. The colica sinistra
E. The iliac
161. A 50 year-old patient was admitted to the hospital with complaints about pain behind his breastbone,
asphyxia during physical activity. Angiography revealed pathological changes in the posterior
interventricular branch of the right coronary artery. What heart parts are affected?
A. * Posterior wall of the right and left ventricles
B. Left atrium
C. Anterior wall of the right and left ventricles
D. Right atrium
E. Right atrioventricular valve
162. A 54-year-old man was admitted to the hospital with complaints of pain in the right subcostal region,
vomiting with blood. Objectively: enlarged liver, varicose veins in the stomach and esophagus.
Disfunction of what vessel is likely to be?
A. * Vena porta

B. Aorta abdominalis
C. Vena hepatica
D. Vena cava superior
E. Vena cava inferior
163. A 70 year old female patient was diagnosed with fracture of left femoral neck accompanied by
disruption of ligament of head of femur. The branch of the following artery is damaged:
A. * Obturator
B. Femoral
C. External iliac
D. Inferior gluteal
E. Internal pudendal
164. A man suffering from osteochondrosis got acute pain in the abdominal muscles (lateral and anterior).
During objective examination a physician diagnosticated increased pain sensitivity of skin in the
hypogastric region. This pain might be caused by affection of the following nerve:
A. * Iliohypogastric
B. Sciatic
C. Obturator
D. Femoral
E. Genitofemoral
165. A man with a stab wound in the area of quadrilateral foramen applied to a doctor. Examination
revealed that the patient was unable to draw his arm aside from his body. What nerve is most
probably damaged?
A. * N. axillaris
B. N. medianus
C. N.radialis
D. N. ulnaris
E. N. Musculocutaneus
166. A patient complains about edema of legs, skin cyanosis, and small ulcers on one side of the lateral
condyle. Examination revealed a swelling, enlarged veins, formation of nodes. The pathological
process has started in the following vein:
A. * V. saphena parva
B. V. saphena magna
C. V. femoralis
D. V. profunda femoris
E. V. iliaca externa
167. A patient has a malignisation of thoracic part of esophagus. What lymphatic nodes are regional for
this organ?
A. * Anulus lymphaticus cardiae
B. Nodi lymphatici paratrachealis
C. Nodi lymphatici prevertebralis
D. Nodi lymphatici pericardiales laterales
E. Nodi lymphatici mediastinales posteriores
168. A patient has an exudative pleurisy. At what level should the pleural puncture along the posterior
axillary line be taken?
A. * ІХ intercostal space
B. VІІІ intercostal space
C. VІІ intercostal space
D. ХІ intercostal space

E. VІ intercostal space
169. A patient has lost skin sensitivity in the region of the medial surface of his shoulder. This is the result
of dysfunction of the following nerve:
A. * Medial brachial cutaneous nerve
B. Medial antebrachial cutaneous nerve
C. Radial nerve
D. Ulnar nerve
E. Axillary nerve
170. A patient has lost skin sensitivity in the region of the medial surface of his shoulder. This is the result
of dysfunction of the following nerve:
A. * Medial brachial cutaneous nerve
B. Medial antebrachial cutaneous nerve
C. Radial nerve
D. Ulnar nerve
E. Axillary nerve
171. A patient has tissue ischemia below the knee joint accompanied with intermittent claudication. What
artery occlusion should be suspected?
A. * Popliteal artery
B. Peroneal artery
C. Posterior tibial artery
D. Anterior tibial artery
E. Proximal part of femoral artery
172. A patient in three weeks after acute myocardial infarction has pain in the heart and joints and
pneumonia. What is the main mechanism of development of post-infarction Dressler’s syndrome?
A. * Autoimmune inflammation
B. Ischemia of myocardium
C. Resorption of enzymes from necrotized area of myocardium
D. Secondary infection
E. Vessels thrombosis
173. A patient with neuritis of femoral nerve has disturbed flexion of thigh as well as disturbed crus
extension in the knee joint. What muscle's function is disturbed?
A. * Quadriceps muscle of thigh
B. Biceps muscle of thigh
C. Triceps muscle of thigh
D. Semitendinous muscle
E. Semimembranous muscle
174. A patient's knee joint doesn't extend, there is no knee-jerk reflex, skin sensitivity of the anterior
femoral surface is disturbed. What nerve structures are damaged?
A. * Femoral nerve
B. Superior gluteal nerve
C. Big fibular nerve
D. Obturator nerve
E. Inferior gluteal nerve
175. A woman consulted a doctor about swelling and tenderness of the lower extremity, swollen veins and
nodes on the medial surface of thigh. Which vein was affected?
A. * Great saphenous
B. Small saphenous
C. Femoral

D. Popliteal
E. Tibial
176. A woman suffering from osteochondrosis felt acute pain in her humeral articulation that became
stronger when she abducted her shoulder. These symptoms might be caused by damage of the
following nerve:
A. * Axillary nerve
B. Subscapular nerve
C. Dorsal scapular nerve
D. Subclavicular nerve
E. Throracodorsal nerve
177. A woman underwent an operation on account of extrauterine (tubal) pregnancy. In course of the
operation the surgeon should ligate the branches of the following arteries:
A. * Uterine and ovarian
B. Superior cystic and ovarian
C. Inferior cystic and ovarian
D. Uterine and superior cystic
E. Uterine and inferior cystic
178. After a road accident a driver was delivered to the hospital with an injury of the medial epicondyle of
humerus. What nerve might be damaged in this case?
A. * n. ulnaris
B. n. radialis
C. n. axillaris
D. n. muscolocutaneus
E. n. medianus
179. After resection of the middle third of femoral artery obliterated by a thrombus the lower extremity is
supplied with blood due to the surgical bypass. Name an artery that plays the main role in
reestablishment of blood flow:
A. * Deep femoral artery
B. Superficial circumflex artery of hip bone
C. Descending geniculat artery
D. Superficial epigastric artery
E. Deep external pudendal artery
180. After trauma a 44-year-old patient had a rupture of left palm muscle tendons and of the surface of
blood vessels. After operation and removal of the most part of the necrotically changed muscle tissue
the bloodstream was normalized. What vessels have helped with restoration of bloodstream?
A. * Arcus palmaris profundus
B. Arcus palmaris superficialis
C. Aa. digitales palmares communes
D. Aa. metacarpeae palmares
E. Aa. perforantes
181. During the fetal period of the development in the vascular system of the fetus large arterial (Botallo's)
duct is functioning which converts intolig. arteriosum after birth. What anatomical formations does
this duct connect between each other?
A. * Pulmonary trunk and aorta
B. Right and left auricle
C. Aorta and inferior vena cava
D. Pulmonary trunk and superior vena cava
E. Aorta and superior vena cava

182. Examination of a 2-year-old child revealed physical developmental lag, the child often has
pneumonias. The child was diagnosed with nonclosure of ductus arteriosus. Haemodynamics disorder
was caused by the intercommunication of the following vessels:
A. * Aorta and pulmonary trunk
B. Pulmonary trunk and pulmonary veins
C. Superior cava and aorta
D. Superior cava and pulmonary trunk
E. Aorta and pulmonary veins
183. On examination of a road accident victim a doctor revealed left clavicle fracture and disturbed blood
circulation in an extremity (no pulsing of radial artery). What cause of blood circulation disturbance
is the most probable?
A. * Compression of subclavian artery
B. Compression of axillary artery
C. Compression of subclavian vein
D. Compression of vertebral artery
E. Compression of axillary vein
184. The patient has come to the hospital from the smelting workshop in the condition of hyperthermia.
What is the direct cause of loss of consciousness at the heat stroke?
A. * Decreased brain blood supply
B. Arterial pressure drop
C. Increased water loss through sweating
D. Decrease of heart output
E. Dilatation of peripheral vessels
185. The process of heart transplantation determined the viability of myocardial cells. The determination
of what myocardium parameter is the most important?
A. * Rest potential of cardiomyocytes
B. Heart temperature
C. Concentration of oxygen in heart vessels
D. Concentration of calcium-ions in myofibrils
E. Concentration of Ca-ions in heart vessels
186. Victim has elbow joint trauma with avulsion of medial epicondyle of humerus. What nerve can be
damaged in this trauma?
A. * Ulnar
B. Radial
C. Musculocutaneous nerve
D. Cardiac cutaneous nerve
E. Medial cutaneous nerve of forearm
187. Where should the catheter for evacuation of the lymph from the thoracic lymph duct be inserted?
A. * To the left venous corner
B. To the right venous corner
C. To the superior vena cava
D. To the inferior vena cava
E. To the left inguinal vein
188. While palpating mammary gland of a patient a doctor revealed an induration in form of a node in the
inferior medial quadrant. Metastases may extend to the following lymph nodes:
A. * Parasternal
B. Posterior mediastinal
C. Profound lateral cervical

D. Bronchopulmonary
E. Superior diaphragmal
189. While performing an operation in the area of armpit crease a surgeon has to define an arterial vessel
surrounded by fascicles of brachial plexus. What artery is it?
A. * A. axillaris
B. A. vertebralis
C. A. transversa colli
D. A. profunda brachii
E. A. subscapularis
190. While preparing a patient to the operation the heart chambers' pressure was measured. In one of them
the pressure changed during one heart cycle from 0 to 120 mm Hg. What chamber of heart was it?
A. * Left ventricle
B. Right ventricle
C. Right atrium
D. Left atrium
E. Atrium
191. The patient has an inflammation of cervical lymph nodes. To what group they are belong ?
A. * All
B. Nodi profundi superior
C. Nodi profundi inferior
D. Nodi supraclavicularis
E. Nodi superficiales
192. The patient has bleeding from vena jugularis externa. Which veins forming it?
A. * V. occipitalis
B. V.lingualis
C. V. facialis
D. V. parotidea
E. Correct answer is missing
193. The patient has injury of common carotid artery, the direct sings of it damage is…
A. * Acute arterial insufficiency
B. Damage of the skin
C. Loss of consciousness
D. Crepitation
E. Defect of soft tissues
194. The patient has injury of common carotid artery, the direct sings of it damage is…
A. * Pulsating hematoma
B. Loss of consciousness
C. Crepitation
D. Damage of the skin
E. Defect of soft tissues
195. The patient has injury of common carotid artery, the direct sings of it damage is…
A. * Absence or impaired pulsation below the site of wound
B. Defect of soft tissues
C. Crepitation
D. Damage of the skin
E. Loss of consciousness
196. The patient has injury of vena jugularis interna, it collecting blood from the…

A. * Head and neck


B. Hands
C. Thoracic cavity
D. Abdomen
E. Correct answer is missing
197. The patient has injury of vena jugularis interna, it collecting blood from the…
A. * Correct answer is missing
B. Hands
C. Thoracic cavity
D. Abdominal
E. Spin
198. The patient has swelling of the left half of head. Where the lymph flows from this part of head?
A. * Truncus jugularis sinister
B. Truncus jugularis dexter
C. Right common carotid artery
D. Left common carotid artery
E. Correct answer is missing
199. The patient has swelling of the right half of head. Where the lymph flows from this part of head?
A. * Truncus jugularis dexter
B. Truncus jugularis sinister
C. Left common carotid artery
D. Right common carotid artery
E. Correct answer is missing
200. The patient has thrombosis of terminal branch of external carotid artery, it is…
A. Inferior tympanic artery
B. Posterior auricular artery
C. * Maxillary artery
D. Lingual artery
E. Superior thyroid artery
201. When the doctor wants to make temporary hemostasis, he uses the methods of…
A. * Maximumflexing of the extremity in joints, elevation of the extremity
B. Suturing of vessels
C. Introduction of haemostatic medicines
D. Angiorrhaphy
E. Electro-coagulation of vessels
202. When the doctor wants to make temporary hemostasis, he uses the methods of…
A. * Digital compression of the artery
B. Electro-coagulation of vessels
C. Introduction of medicines
D. Suturing of vessels
E. Angiorrhaphy
203. A man found bleeding wound in the body of the lower jaw to the front of the masseter. What artery
may cause bleeding?
A. * Facial arteries
B. Lingual arteries
C. Ascending pharyngeal arteries.

D. Superior thyroid arteries


E. Maxillary arteries
204. After a head injury in a side area appeared bleeding. What arteries blood supply of this area?
A. * Frontal, parietal arteries.
B. Ascending and descending esophageal arteries.
C. Occipital posterior, posterior auricle arteries.
D. Posterior anterior tympanic arteries.
E. Sternocleidomastoid and descending esophageal arteries
205. During the examination in patient detected bleeding from the auricle and external acoustic meatus.
Which of arteries can be damaged?
A. Superficial temporal artery
B. Superior thyroid arteries
C. Ascending pharyngeal arteries.
D. Facial arteries
E. * Maxillary arteries
206. During the examination in patient detected damaged of the vagus nerve. The patient said that several
days ago he injured occipital area of head. Name foramen through which the vagus nerve leaves the
skull.
A. * jugular;
B. round;
C. oval;
D. styloid;
E. lacerum.
207. During the examination in patient detected damaged of the vagus nerve. After the MRI examination
found pathological formation in the area of jugular foramen. Which of cranial nerves leave the skull
with vagus nerve?
A. * ІХ, ХІ pair of cranial nerves
B. ІХ, ХІІ pair of cranial nerves;
C. ІХ, ХІ, ХІІ pair of cranial nerves;
D. XII, VII pair of cranial nerves;
E. VII, IX pair of cranial nerves.
208. During the examination in patient detected external bleeding from the medial corner of eye. Which of
arteries can be damaged?
A. * Facial arteries
B. Ascending pharyngeal arteries.
C. Superficial temporal artery
D. Superior thyroid arteries
E. Maxillary arteries
209. During the examination of the woman, found bleeding wound to the back of the angle of lower jaw.
What artery may cause bleeding in the area of parotid salivary gland?
A. * Superficial temporal artery
B. Ascending pharyngeal arteries.
C. Facial arteries
D. Maxillary arteries
E. Superior thyroid arteries
210. During the MRI examination revealed pathological formation in the area of jugular foramen. Which
of cranial nerves can be damaged?
A. from IX to XII pair

B. from V to VIII pair


C. * from IX to XI pair
D. from I to IV pair
E. from I to IV pair
211. Identify the symptoms that arise when irritation of nuclei and trunk of vagus nerve?
A. * Increased peristalsis of the digestive tract
B. Increased of blood pressure;
C. Tachycardia;
D. Inhibition of secretion of stomach;
E. Increased the tension of constrictors muscles.
212. Identify the symptoms that arise when irritation of nuclei and trunk of vagus nerve?
A. * Decrease the tension of constrictors muscles;
B. Increased of blood pressure;
C. Increased the tension of constrictors muscles
D. Inhibition of secretion of stomach;
E. Tachycardia.
213. Identify the symptoms that arise when irritation of nuclei and trunk of vagus nerve?
A. * Vasorelaxation;
B. Tachycardia;
C. Increased of blood pressure;
D. Increased the tension of constrictors muscles;
E. Inhibition of secretion of stomach.
214. Identify the symptoms that arise when irritation of nuclei and trunk of vagus nerve?
A. * Slowdown and the weakening of the heart;
B. Tachycardia;
C. Increased of blood pressure;
D. Increased the tension of constrictors muscles;
E. Inhibition of secretion of stomach.
215. Identify the symptoms that arise when irritation of nuclei and trunk of vagus nerve?
A. * Bradikardiya;
B. Increased of blood pressure;
C. Tachycardia;
D. Increased the tension of constrictors muscles;
E. Inhibition of secretion of stomach.
216. In patient detected bleeding in left side area of the head. Which artery carries blood supply of
muscles and skin in this area?
A. * left superficial temporal artery
B. right anterior tympanic
C. deep temporal arteries
D. left sphenopalatine
E. all correct answers
217. In patient detected bleeding in left side area of the head. Which artery carries blood supply of
muscles and skin in this area?
A. * Frontal, parietal arteries.
B. Occipital posterior, posterior auricle arteries.
C. Ascending and descending esophageal arteries.
D. Posterior anterior tympanic arteries.

E. Sternocleidomastoid and descending esophageal arteries


218. In patient detected bleeding in left side area of the head. Superficial temporal artery is damaged.
What does it supply in head?
A. * Skin and muscles of temporal and parietal areas, parotid salivary gland, auricle, external acoustic
meatus, external corner of eye.
B. Sternocleidomastoid muscle and skin above it, trapezius muscle.
C. Auditory tube, tympanic cavity, external acoustic meatus, auricle, masseter muscle, parotid gland.
D. Back surface of auricle, skin and muscles of the back of head, submandibular gland, superior and
inferior eyelids.
E. Skin and muscles of temporal and parietal areas, submandibular gland, external acoustic meatus,
external corner of eye.
219. In tourist after falling from the mountain detected bleeding in the face. From which artery bleeding
appeared?
A. * Facial arteries
B. Lingual arteries
C. Ascending pharyngeal arteries.
D. Superior thyroid arteries
E. Maxillary arteries
220. In tourist after falling from the mountain detected bleeding in the face damaged facial arteries. Name
a place, where we can press facial artery?
A. * Basis of body of mandible, front of masseter muscle.
B. Submandibular triangle.
C. Branch of mandible, behind the edge of masseter muscle.
D. Carotid triangle.
E. Corner of mouth.
221. A man found bleeding wound in the body of the lower jaw to the front of the masseter. What artery
may cause bleeding?
A. * Facial arteries
B. Lingual arteries
C. Ascending pharyngeal arteries.
D. Superior thyroid arteries
E. Maxillary arteries
222. After a head injury in a side area appeared bleeding. What arteries supply of this area?
A. * Frontal, parietal arteries.
B. Ascending and descending esophageal arteries.
C. Occipital posterior, posterior auricle arteries.
D. Posterior anterior tympanic arteries.
E. Sternocleidomastoid and descending esophageal arteries
223. During the examination in patient detected bleeding from the auricle and external acoustic meatus.
Which of arteries can be damaged?
A. Superficial temporal artery
B. Superior thyroid arteries
C. Ascending pharyngeal arteries.
D. Facial arteries
E. * Maxillary arteries
224. During the examination in patient detected damaged of the vagus nerve. After the MRI examination
found pathological formation in the area of jugular foramen. Name which of cranial nerves leave the
skull with vagus nerve.

A. * ІХ, ХІ pair of cranial nerves


B. ІХ, ХІІ pair of cranial nerves;
C. ІХ, ХІ, ХІІ pair of cranial nerves;
D. XII, VII pair of cranial nerves;
E. VII, IX pair of cranial nerves.
225. During the examination in patient detected external bleeding from the medial corner of eye. Which of
arteries can be damaged?
A. * Facial arteries
B. Ascending pharyngeal arteries.
C. Superficial temporal artery
D. Superior thyroid arteries
E. Maxillary arteries
226. During the examination of the woman, found bleeding wound to the back of the angle of lower jaw.
What artery may cause bleeding in the area of parotid salivary gland?
A. * Superficial temporal artery
B. Ascending pharyngeal arteries.
C. Facial arteries
D. Maxillary arteries
E. Superior thyroid arteries
227. During the MRI examination revealed pathological formation in the area of jugular foramen. Which
of cranial nerves can be damaged?
A. from IX to XII pair
B. from V to VIII pair
C. * from IX to XI pair
D. from I to IV pair
E. from I to IV pair
228. Identify the symptoms that arise when irritation of nuclei and trunk of vagus nerve?
A. * Increased peristalsis of the digestive tract
B. Increased of blood pressure;
C. Tachycardia;
D. Inhibition of secretion of stomach;
E. Increased the tension of constrictors muscles.
229. Identify the symptoms that arise when irritation of nuclei and trunk of vagus nerve?
A. * Decrease the tension of constrictors muscles;
B. Increased of blood pressure;
C. Increased the tension of constrictors muscles
D. Inhibition of secretion of stomach;
E. Tachycardia.
230. Identify the symptoms that arise when irritation of nuclei and trunk of vagus nerve?
A. * Vasorelaxation;
B. Tachycardia;
C. Increased of blood pressure;
D. Increased the tension of constrictors muscles;
E. Inhibition of secretion of stomach.
231. Identify the symptoms that arise when irritation of nuclei and trunk of vagus nerve?
A. Inhibition of secretion of stomach ;
B. Tachycardia;

C. * Slowdown and the weakening of the heart;


D. Increased the tension of constrictors muscles;
E. Increased of blood pressure.
232. Identify the symptoms that arise when irritation of nuclei and trunk of vagus nerve?
A. * Bradikardiya;
B. Increased of blood pressure;
C. Tachycardia;
D. Increased the tension of constrictors muscles;
E. Inhibition of secretion of stomach.
233. In patient detected bleeding in left side area of the head. Which artery carries blood supply of
muscles and skin in this area?
A. * left superficial temporal artery
B. right anterior tympanic
C. deep temporal arteries
D. left sphenopalatine
E. all correct answers
234. In patient detected bleeding in left side area of the head. Which artery carries blood supply of
muscles and skin in this area?
A. * Frontal, parietal arteries.
B. Occipital posterior, posterior auricle arteries.
C. Ascending and descending esophageal arteries.
D. Posterior anterior tympanic arteries.
E. Sternocleidomastoid and descending esophageal arteries
235. In patient detected bleeding in left side area of the head. Superficial temporal artery is damaged.
What does it supply in head?
A. * Skin and muscles of temporal and parietal areas, parotid salivary gland, auricle, external acoustic
meatus, external corner of eye.
B. Sternocleidomastoid muscle and skin above it, trapezius muscle.
C. Auditory tube, tympanic cavity, external acoustic meatus, auricle, masseter muscle, parotid gland.
D. Back surface of auricle, skin and muscles of the back of head, submandibular gland, superior and
inferior eyelids.
E. Skin and muscles of temporal and parietal areas, submandibular gland, external acoustic meatus,
external corner of eye.
236. In tourist after falling from the mountain detected bleeding in the face. From which artery bleeding
appeared?
A. * Facial arteries
B. Lingual arteries
C. Ascending pharyngeal arteries.
D. Superior thyroid arteries
E. Maxillary arteries
237. In tourist after falling from the mountain detected bleeding in the face damaged facial arteries. Name
a place, where we can press facial artery?
A. * Basis of body of mandible, front of masseter muscle.
B. Submandibular triangle.
C. Branch of mandible, behind the edge of masseter muscle.
D. Carotid triangle.
E. Corner of mouth.
238. During operation surgeon can see that the inferior alveolar nerve and vessels are contents of…

A. * Mandibular canal
B. Angle of mandible
C. Mandibular notch
D. Mental foramen
E. None of these
239. During operation surgeon can see that the lesser palatine nerves and vessels pass through…
A. * Lesser palatine foramen
B. Palatine crest
C. Posterior nasal spine
D. Sphenopalatine foramen
E. Incisive fossa
240. During operation surgeon can see that the maxillary nerve passes through the next region and
contains…
A. * Rotundum foramen, sensory fibers
B. Oval foramen, sensory fibers
C. Rotundum foramen, motor fibers
D. Rotundum foramen, parasympathetic fibers
E. None of these
241. During the operation surgeon can see that the facial nerve pass through the next region and is
responsible for our special sense of…
A. * Internal acoustic meatus
B. Optic canal, sight
C. Cribriform plate
D. Foramen cecum
E. Jugular foramen
242. The patient has a lesion of the trigeminal nerve, it would result in weakness of…
A. * Compression the teeth
B. Closure the eyes
C. Moving the eyeball
D. Lifting the upper eyelid
E. Contraction of tongue
243. The patient has a lesion of the trigeminal nerve; it would result in weakness of…
A. * None of these
B. Contraction of sternocleidomastiod muscle
C. Compression the teeth
D. Protruding the mandibule
E. Contraction of masseter muscle
244. The patient has lesion of control facial expressions, which nerve is responsible for this function?
A. * Facial nerve
B. Trigeminal nerve
C. Accessory nerve
D. Vestibulocochlear nerve
E. Trochlear nerve
245. The patient has lesion of the ansa cerviccalis nerve, it would result in weakness of
A. * Contraction of infrahyoid muscle
B. Contraction of sternocleidomastoid muscle
C. Contraction of orbicularis oris muscle

D. Contraction of platisma
E. Contraction of mylohyoid muscle
246. The patient has the lesion of the facial nerve; it would result in weakness of…
A. * Contraction of zygomatic muscle
B. Contraction of masseter muscle
C. Compression the teeth
D. Protruding the mandible
E. None of these
247. The patient has the lesion of the facial nerve; it would result in weakness of…
A. * Contraction of orbicularis oculi muscle
B. Contraction of masseter muscle
C. Compression the teeth
D. Protruding the mandible
E. None of these
248. The patient has the lesion of the facial nerve; it would result in weakness of…
A. * Contraction of bucinator muscle
B. Contraction of masseter muscle
C. Protruding the mandible
D. Compressing the teeth
E. None of these
249. The patient has the lesion of the trigeminal nerve, it would result in weakness of…
A. * Contraction of masseter muscle
B. Contraction of platisma
C. Contraction of orbicularis oris muscle
D. Contraction of buccinator muscle
E. None of these
250. The patient has the lesion of the trigeminal nerve, it would result in weakness of…
A. * Contraction of temporal muscle
B. Contraction of platisma
C. Contraction of masseter muscle
D. Contraction of tongue muscle
E. None of these
251. Doctor dentist making local anesthesia of the inferior alveolar nerve, it is situated at…
A. * Mandibular canal
B. Mental foramen
C. Angle of mandible
D. Mandibular notch
E. None of these
252. Doctor dentist making local anesthesia of the inferior alveolar nerve, it is situated at…
A. * None of these
B. Palatine foramen
C. Angle of mandible
D. Mandibular notch
E. Infraorbital canal
253. During operation surgeon can see that the mandibular nerve passes trough the next region and
contains…

A. * Oval foramen ,sensory and motor fibers


B. Mandibular foramen, sensory and motor fibers
C. Oval foramen, sympathetic fibers
D. Oval foramen, parasympathetic fibers
E. Rotundum foramen, motor fibers
254. During operation surgeon can see that the maxillary nerve passes trough the next region and supplies
the…
A. * None of these
B. Jugular foramen, larynx
C. Orbital canal, maxillary teeth
D. Foramen rotundum, lips
E. Foramen rotundum, eyeball
255. During operation surgeon can see that the zygomaticus nerve passes trough the next region and is
responsible for the…
A. * Inferior orbital fissure, touch sensation
B. Superior orbital fissure, touch sensation
C. Superior orbital fissure, innervations of muscles
D. Inferior orbital fissure, innervations of muscles
E. Superior orbital fissure, innervations of gland
256. During the operation surgeon can see that the glosspharingeal nerves pass trough the next region and
are responsible for our special sense of…
A. * Jugular foramen, taste
B. Cribriform plate, smell
C. Optical canal, sight
D. Foramen cecum, taste
E. Internal acoustic meatus, taste
257. During the operation surgeon can see that the glossopharingeal nerves pass trough the next region
and are responsible for our special sense of…
A. * None of all
B. Jugular foramen, smell
C. Internal acoustic meatus, taste
D. Optic canal, sight
E. Foramen cecum, taste
258. During the operation surgeon can see that the СN8 nerves pass through the next region and are
responsible for our special sense of…
A. * Internal acoustic meatus, balance
B. Foramen cecum, taste
C. Optic canal, sight
D. Cribriform plate, smell
E. Jugular foramen
259. During the operation surgeon can see that the СN8 nerves pass through the next region and are
responsible for our special sense of…
A. * Internal acoustic meatus, hearing
B. Jugular foramen touch
C. Cribriform plate, smell
D. Foramen ovale
E. Optical canal, sight

260. During the operation surgeon can see that the СN8 nerves pass through the next region and are
responsible for our special sense of…
A. * None of the above
B. Cribriform plate, smell
C. Optic canal, sight
D. Jugular foramen, touch
E. Foramen cecum, taste
261. The patient has lesion of the hypoglossus nerve, it would result in weakness of…
A. * Contraction of vertical muscle
B. Contraction oforbicularis oris muscle
C. Contraction of platisma
D. Contraction of bucinator muscle
E. None of these
262. The patient has lesion of the hypoglossus nerve, it would result in weakness of…
A. * Contraction of transverse muscle
B. Contraction of orbicularis oris muscle
C. Contraction of platisma
D. Contraction of buccinators muscle
E. None of these
263. The patient has lesion of the hypoglossus nerve, it would result in weakness of…
A. * Contraction of geniohyoid muscle
B. Contraction of buccinators muscle
C. Contraction of orbicularis oris muscle
D. Contraction of platisma
E. Contraction of mylohyoid muscle
264. The patient has lesion of the hypoglossus nerve, it would result in weakness of…
A. * Contraction of infrahyoid muscle
B. Contraction of platisma muscle
C. Contraction of mylohyoid muscle
D. Contraction of buccinators muscle
E. None of these
265. The patient has lesion of control facial expressions which nerve is responsible for this function?
A. * Facial nerve
B. Trigeminal nerve
C. Trochlear nerve
D. Vestibulotrochlear nerve
E. Accessory nerve
266. The patient has lesion of hypoglossus nerve, what is it function?
A. * Supplies all the muscles of the tongue
B. Is the taste nerve of the tongue
C. Aries from the floor of the 3-d ventricle
D. Supplies the stylohyoid muscle
E. Supplies the sublingual salivary gland
267. ?A 18-year-old patient came to the out-patient department with the complaints of bleeding in the
vestibule of his nose after trauma. On examination: the mechanical injure of the mucous layer of the
nasal vestibule. What is the boundary between the vestibule and proper nasal cavity?
A. * Nasal limen

B. Nasal roller
C. Nasal septa
D. Choanes
E. Nostrils
268. A 25-year-old patient complained of the decreased vision. Accommodation disorders, dilated pupil,
not reacting on the light were revealed on examination. Which muscles are disturbed?
A. * Pupil sphincter muscle, ciliary muscle
B. Pupil dilating muscle, ciliary muscle
C. Inferior oblique muscle, ciliary muscle
D. Lateral rectus muscle, Pupil sphincter muscle
E. Pupil sphincter muscle and dilating muscle
269. In 26-year-old patient was found a big furuncle of soft tissues of face by the root of nose and inferior
eyelid. This disease can be seriously complicated by the infection spreading along veins of this region
to the sinuses of dura brain mater. Which sinus is most likely to be affected?
A. * Cavernous
B. Superior sagittal
C. Occipital
D. Sigmoid
E. Petrosal
270. A 3 year old child was admitted to the hospital with otitis. Pus is probable to spread from the
tympanic cavity. Where pus can get into?
A. * Into auditory tube
B. Into internal ear
C. Into internal acoustic meatus
D. Into external acoustic meatus
E. Into posterior cranial fossa
271. A 30-year-old patient was hospitalized due to bleeding of the facial artery. What place on the face has
to be pressed to stop bleeding?
A. * The mandible’s edge
B. The mental process
C. The mandible’s branch
D. The nose’s back
E. The molar bone
272. A 32-year-old patient has been diagnosed with bartholinitis (inflammation of Bartholin's glands ).
What part of the female urogenital system has the Bartholin's glands?
A. * The labia major
B. The labia minor
C. The clitoris
D. The vagina
E. The uterus
273. A 35 year old patient complained to a doctor about having intense rhinitis and loss of sense of smell
for a week. Objectively: nasal cavity contains a lot of mucus that covers mucous membrane and
blocks olfactory receptors. In what part of nasal cavity are these receptors situated?
A. * Superior nasal meatus
B. Meddle nasal meatus
C. Inferior nasal meatus
D. Common nasal meatus
E. Vestibule of nose

274. A 35 year old patient came to the admission department with complaints of pain and edema in the
region of floor of oral cavity. After examination he was diagnosed with inflammation in the region of
excretory duct of submandibular gland. Where does this duct open into?
A. * Caruncula sublingualis
B. Vestibulum oris
C. Foramen caecum linguae
D. Plica fimbriata
E. Recesus gingivalis
275. A 40-year-old patient complains of intensive heartbeats, sweating, nausea, visual impairment, arm
tremor, hypertension. From his anamnesis: 2 years ago he was diagnosed with pheochromocytoma.
Hyperproduction of what hormones causes the given pathology?
A. * Catecholamines
B. Aldosterone
C. Glucocorticoids
D. ACTH
E. Thyroidal hormones
276. A 40-year-old woman was admitted to the infectious diseases department with high body
temperature. Objectively: marked meningeal symptoms. A spinal cord punction was made. What
anatomic formation was examined?
A. * Spatium subarachnoideum
B. Spatium subdurale
C. Spatium epidurale
D. Cavum trigeminale
E. Cisterna cerebellomedullaris posterior
277. A 45-year-old patient was admitted to the surgical department with complaints of abrupt sharp pain in
the epigastric region. After examination it was diagnosed: perforated ulcer of the posterior wall of the
stomach. Where did content of the stomach flow out while perforation?
A. * To the omental bursa
B. To the liver bursa
C. To the preventricular sack
D. To the left mesenteric sinus
E. To the right mesenteric sinus
278. A 50 year-old patient had hemorrhage of the brain and was taken to the hospital. The place of
hemorrhage was revealed on the lateral hemispheres surfaces during the medical examination. What
artery was injured?
A. * The middle cerebral artery
B. The anterior cerebral artery
C. The posterior cerebral artery
D. The anterior communicating artery
E. The posterior communicating artery
279. A 53-year-old female patient was diagnosed with liver rupture resulting from a blunt abdominal
injury. Where blood will be collected?
A. * Rectouterine pouch
B. Vesicouterine pouch
C. Right mesenteric sinus
D. Omental bursa
E. Left mesenteric sinus

280. A 58 year old woman had her uterus and all appendages completely removed. It resulted in stoppage
of urine excretion. Cystoscopy results: bladder doesn't contain any urine; urine doesn't come also
from ureteric orifices. What part of urinary excretion system was damaged during the operation?
A. * Ureter
B. Uretra
C. Vesica urinaria
D. Pelvis renalis
E. Ren
281. A 5-year-old child was admitted to the otorhinolaryngological department with diagnosis -
inflammation of the middle ear. Disease started from the inflammation of the nasopharynx. How did
the infection get into the tympanic cavity through?
A. * Musculortubal canal
B. Tympanic Canaliculus tympanicus
C. Carotid canal
D. Canaliculus chordal tympani
E. Canaliculi caroticotympanici
282. A 6 month old baby ill with bronchitis was taken for an X-ray of chest. Apart of changes associated
with bronchi the X-ray film showed a shadow of thymus gland. What might have caused such
changes?
A. * The above-mentioned condition is a normal variant for this age
B. It's the effect of bronchitis
C. It is caused by abnormal position
D. It is caused by thymus inflammation
E. It is caused by neoplastic process
283. A 60-year-old patient has reduced perception of high-frequency sounds. What structure of auditory
analizer was damaged?
A. * Main membrane of cochlea near the oval window
B. Main membrane of cochlea near helicotrema
C. Eustachian tube
D. Muscles of middle ear
E. Tympanic membrane
284. A 6-year-old child fell on the cutting object and traumatized soft tissues between tibia and fibula.
What kind of bone connection was injured?
A. * Membrane
B. Suture
C. Ligament
D. Fontanel
E. Gomphosis
285. A 70 y.o. man has cut an abscess off in the area of mastoid process during shaving. Two days later he
was admitted to the hospital with inflammation of arachnoid membranes. How did the infection
penetrate into the cavity of skull?
A. * V.emissariaе mastoideaе
B. V.v.labyrinthi
C. V.v.tympanicae
D. V.facialis
E. V.v.auriculares

286. A 70-year-old patient is diagnosed with brainstem haemorrhage. Examination revealed increased
tonus of flexor muscles accompanied by decreased tonus of extensor muscles. Such changes in
muscle tonus can be explained by the irritation of the following brain structures:
A. * Red nuclei
B. Vestibular nuclei
C. Quadrigeminal plate
D. Black substance
E. Reticular formation
287. A foreign body (a button) closed space of the right superior lobar bronchus. What segments of the
right lung won't be supplied with air?
A. * Apical, posterior, anterior
B. Superior and inferior lingular
C. Apical and posterior basal
D. Apical and median basal
E. Medial and lateral
288. In man after loss 1,5 liter blood has suddenly reduced diuresis. What hormone after increased
secretion did cause such diuresis alteration?
A. * Vasopressin
B. Corticotropin
C. Natriuretic
D. Cortisol
E. Parathormone
289. A man with internal abdominal injury from the right side and suspicion of liver rupture was admitted
to the traumatological department. Where blood will be collected?
A. * Excavatio rectovesicalis
B. Bursa omentalis
C. Recessus intersigmoideus
D. Fossa ischio-analis
E. Recessus duodenalis inferior
290. A mother consulted the doctor about her one year old child, who has got eruption of six teeth. How
many teeth should the child of such age have?
A. * 8
B. 10
C. 7
D. 12
E. 6
291. A mother of a newborn complains of her baby's constant belching with undigested milk. Which
anomaly is it?
A. * Esophageal atresia
B. Labium leporium
C. Faux lupinum
D. Anal atresia
E. Esophageal fistula
292. A newborn didn't take his first breath. Autopsy revealed that in spite of unobstructed respiratory tracts
the baby's lungs didn't expand. What might be the cause of it?
A. * Surfactant absence
B. Bronchostenosis
C. Bronchi rupture

D. Apical cap of lung


E. Alveole enlargement
293. A patient complained about ear noise and pain sensations. Objectively: the patient has acute
respiratory disease, rhinitis. The infection penetrated into the tympanic cavity through the following
opening of the pharynx:
A. * Pharyngeal opening of auditory tube
B. Tympanic opening of auditory tube
C. Choanae
D. Fauces
E. Aperture of larynx
294. A patient complains about impaired evacuatory function of stomach (long-term retention of food in
stomach). Examination revealed a tumour of initial part of duodenum. Specify localization of the
tumour:
A. * Pars superior
B. Pars inferior
C. Pars descendens
D. Pars ascendens
E. Flexura duodeni inferior
295. A patient complains of dizziness and hearing loss. What nerve is damaged?
A. * Vestibulocochlear
B. Trigeminus
C. Sublingual
D. Vagus
E. Trochlear
296. A patient complains of having urination disorder. He is diagnosed the hypertrophy of prostate gland.
What part of gland is damaged?
A. * Median lobe
B. Left lobe
C. Right lobe
D. Base
E. Apex
297. A patient complains of headache, heavy breathing. X-ray examination confirmed the diagnosis -
frontitis. Which nasal meatus may contain purulent discharge?
A. * Middle
B. Superior
C. Inferior
D. Common
E. Above the superior nasal concha
298. A patient got a craniocerebral trauma that resulted in right-side convergent strabismus. Damage of
which craniocerebral nerve is caused such consequences?
A. * n.abducens
B. n.facialis
C. n.trigeminus
D. n.trochlearis
E. n.aculomotorius
299. A patient got a trauma that caused dysfunction of motor centres regulating activity of head muscles.
Where is localized normally the respective centre?
A. * Inferior part of precentral gyrus

B. Superior part of precentral gyrus


C. Supramarginal gyrus
D. Superior parietal lobule
E. Angular gyrus
300. A patient got an injury of spinal marrow in a road accident that caused loss of tactile sensation,
posture sense, vibration sense. What conduction tracts are damaged?
A. * Fascicle of Goll and cuneate fascicle
B. Anterior spinocerebellar tract
C. Rubrospinal tract
D. Reticulospinal tract
E. Tectospinal tract
301. A patient had his tooth extracted. The lingual surface of this tooth was smaller than the buccal one.
Masticatoty surface has oval form. Deep transverse sulcus separates buccal and lingual tubercles. The
root is strongly compressed in mesio-distal direction and has longitudinal sulci on its aproximal
surfaces, it is bifurcated. What tooth was extracted?
A. * First upper premolar
B. Upper canine
C. Lower canine
D. First lower premolar
E. Second upper premolar
302. A patient has an injury in right lateral area of abdomen. What part of large intestine is most likely
injured?
A. * Ascending colon
B. Transverse colon
C. Descending colon
D. Sigmoid colon
E. Rectum
303. A patient has lost ability to recognize the objects by the typical for them sounds (clock, bell, music).
What part of brain is most likely damaged?
A. * Lobus temporalis
B. Lobus occipitalis
C. Lobus frontalis
D. Lobus parietalis
E. Insula
304. A patient has urolithiasis that was complicated by a renal calculus passage. What level of ureter is it
most likely to stop?
A. * Between abdominal and pelvic part
B. In pelvis
C. In the middle abdominal part
D. 2 cm above flowing into urinary bladder
E. 5 cm above pelvic part
305. A patient was admitted to the surgical department with inguinal hernia. During the operation the
surgeon performs plastic surgery on posterior wall of inguinal canal. What structure forms this wall?
A. * Transverse fascia
B. Aponeurosis of abdominal external oblique muscle
C. Inguinal ligament
D. Loose inferior edge of transverse abdominal muscle
E. Peritoneum

306. A patient was admitted to the surgical department with suspected inflammation of Meckel's
diverticulum. What part of bowels should be examined in order to discover the diverticulum in course
of an operation?
A. * Ileum
B. Duodenum
C. Jejunum
D. Caecum
E. Colon ascendens
307. A patient was diagnosed with bartholinitis (inflammation of greater vulvovaginal glands). Which
organ of urogenital system does contain these glands?
A. * Large lips of pudendum
B. Small lips of pudendum
C. Clitoris
D. Vagina
E. Uterus
308. A patient was diagnosed with paralysis of facial and masticatory muscles. The haematoma is inside
the genu of internal capsule. What conduction tract is damaged?
A. * Tr. cortico-nuclearis
B. Tr. cortico-spinalis
C. Tr. cortico-thalamicus
D. Tr. cortico-fronto-pontinus
E. Tr. cortico-temporo-parieto-occipito-pontinus
309. A patient who suffers from cancer of back of tongue has an intense bleeding as a result of affection of
dorsal lingual artery by the tumour. What vessel should be ligated to stop bleeding?
A. * Lingual artery
B. Dorsal lingual artery
C. Deep lingual artery
D. Facial artery
E. Ascending pharyngeal artery
310. A patient with a stab wound of the anterior stomach wall is in surgical care. What formation of
abdominal cavity did the stomach contents get into?
A. * Pregastric bursa
B. Omental bursa
C. Hepatic bursa
D. Left mesenteric sinus
E. Right mesenteric sinus
311. A patient with cholelithiasis fell ill with mechanic jaundice. Examination revealed that the stone was
in the common bile duct. What bile-excreting ducts make up the obturated duct?
A. * Ductus hepaticus communis et ductus cysticus
B. Ductus hepaticus dexter et sinister
C. Ductus hepaticus dexter et ductus cysticus
D. Ductus hepaticus sinister et ductus cysticus
E. Ductus hepaticus communis et ductus choledochus
312. A patient's middle ear inflammation was complicated by mastoiditis. There was a threat of purulent
thrombosis of the nearest venous sinus. What sinus was under the threat?
A. * Sigmoid
B. Inferior petrosal
C. Superior saggital

D. Transverse
E. Rectus
313. A surgeon has to find the common hepatic duct during the operative intervention on account of
concrements in the gall ducts. The common hepatic duct is located between the leaves of:
A. * Hepatoduodenal ligament
B. Hepatogastric ligament
C. Hepatorenal ligament
D. Round ligament of liver
E. Venous ligament
314. A surgeon should reach the omental bursa to perform an operation on abdominal cavity. How can he
reach this part of peritoneal cavity without affecting the integrity of lesser omentum?
A. * Through the epiploic foramen
B. Through the right paracolic sulcus
C. Through the left paracolic sulcus
D. Through the right mesenteric sinus
E. Through the left mesenteric sinus
315. A three year old child was admitted to the hospital with a foreign body in bronches. What bronchus
contains most likely a foreign body?
A. * Right primary
B. Left primary
C. Right segmental
D. Left segmental
E. Lobular
316. A young man consulted a doctor about disturbed urination. Examination of his external genitals
revealed that urethra is split on top and urine runs out of this opening. What anomaly of external
genitals development is the case?
A. * Epispadia
B. Phimosis
C. Hermaphroditism
D. Paraphimosis
E. Hypospadia
317. After a 2 y.o. child has had flu, there appeared complaints about ear ache. A doctor revealed hearing
impairment and inflammation of the middle ear. How did the infection penetrate into the middle ear?
A. * Through the auditory tube
B. Through foramen jugularis
C. Through canalis caroticus
D. Through atrium mastoideum
E. Through canalis nasolacrimalis
318. An 18-year-old man was delivered to the hospital after a road accident. Examination at the
traumatological department revealed multiple injuries of soft tissues of face in the region of the
medial eye angle. The injuries caused massive haemorrhage. What arterial anastomosis might have
been damaged in this region?
A. * a. carotis externa et a. carotis interna
B. a. carotis externa et a. subclavia
C. a. carotis interna et a. subclavia
D. a. subclavia et a. ophthalmica
E. a. carotis interna et a. ophthalmica

319. An eye trauma caused soft tissues infection of eye-socket. Through what anatomical formation can
the infection penetrate into the middle cranial fossa?
A. * Through the superior orbital fissure
B. Through the anterior ethmoidal foramen
C. Through the posterior ethmoidal foramen
D. Through the inferior orbital fissure
E. Through the zygomatic orbital foramen
320. An injured person was delivered to the hospital with a penetrating wound in the left lateral region of
abdomen. What part of the large intestine is most likely damaged?
A. * Colon descendens
B. Colon ascendens
C. Colon transverses
D. Caecum
E. Rectum
321. Brain tomography revealed a tumour in the region of red nucleus. What part of brain isdamaged?
A. * Midbrain
B. Medulla oblongata
C. Cerebellum
D. Interbrain
E. Pons cerebelli
322. Child inspired button. Where is it likely to be?
A. * In the right main bronchus
B. In the left main bronchus
C. In the trachea
D. In the larynx
E. In the gullet
323. Children often have heavy nasal breathing resulting from excessive development of lymphoid tissue
of pharyngeal mucous membrane. What tonsils growth may cause this effect?
A. * Tonsilla pharyngea
B. Tonsilla palatina
C. Tonsilla lingualis
D. Tonsilla tubaria
E. All above mentioned tonsils
324. Children often have laboured nasal breathing which is caused by overdevelopment of lymphoid tissue
of the pharyngeal mucous membrane. This phenomenon may cause enlargement of the following
tonsils:
A. * Tonsilla pharyngea
B. Tonsilla palatina
C. Tonsilla lingualis
D. Tonsilla tubaria
E. All above-mentioned
325. Chronic rhinitis was complicated by inflammation of frontal sinus. What nasal meatus did the
infection get into this sinus through?
A. * Median
B. Inferior
C. Common
D. Superior

E. Nasopharyngeal
326. During complicated labour the symphysis pubis ruptured. What organ can be damaged mostly?
A. * Urinary blader
B. Rectum
C. Ovaria
D. Uterine tubes
E. Uterus
327. During investigation of patient, it was found formation in the white substance of cerebral
hemispheres with location in the knee and frontal part of posterior crus of internal capsule. Fibers of
what conductive tract of the brain will be disrupted?
A. * Tr. pyramidalis
B. Tr. frontothalamicus
C. Тr. thalamocorticalis
D. Tr. frontopontinus
E. Tr. parietooccipitopontinus
328. During the endoscopy the inflammation of a major papilla of the duodenum and the disturbances of
bile secretion were found. In which part of duodenum were the problems found?
A. * Descendent part
B. Ascendant part
C. Bulb
D. Upper horizontal part
E. Lower horizontal part
329. During the examination of patient's oral cavity a dentist found a carious cavity on the crown surface
of the second premolar tooth that was turned to the first molar tooth. Name the damaged crown
surface:
A. * Facies mesialis
B. Facies vestibularis
C. Facies lingualis
D. Facies distalis
E. Facies occlusalis
330. During the examination of patient's oral cavity a dentist noticed a slight overbite of mandibular teeth
by maxillary incisors. What occlusion belongs such position of teeth to?
A. * Orthognathic occlusion
B. Prognathism
C. Biprognathic occlusion
D. Orthogenic occlusion
E. Closed occlusion
331. Examination of a 32 year old patient revealed disproportional skeleton size, enlargement of
superciliary arches, nose, lips, tongue, jaw bones, feet. What gland's function was disturbed?
A. * Hypophysis
B. Epiphysis
C. Pancreas
D. Thyroid
E. Suprarenal
332. Examination of a newborn boy's genitals revealed a cleft of urethra that opens on the inferior surface
of his penis. What developmental anomaly is meant?
A. * Hypospadia
B. Hermaphroditism
C. Epispadia
D. Monorchism
E. Cryptorchism
333. Examination of a patient revealed abnormal development of enamel. This is caused by damage of the
following structural elements of dental germ:
A. * Internal enamel epithelium of enamel organ
B. External enamel epithelium of enamel organ
C. Intermediate layer of enamel organ
D. Pulp of enamel organ
E. Cervix of enamel organ
334. Examination of a patient revealed hypertrophy and inflammation of lymphoid tissue, edema of
mucous membrane between palatine arches (acute tonsillitis). What tonsil is normally situated in this
area?
A. * Tonsilla palatina
B. Tonsilla pharyngealis
C. Tonsilla tubaria
D. Tonsilla lingualis
E. Uvula
335. Examination of a patient with impaired blood coagulation revealed thrombosis of a branch of inferior
mesenteric artery. What bowel segment is damaged?
A. * Colon sigmoideum
B. Ileum
C. Caecum
D. Colon transversum
E. Colon ascendens
336. In case of a penetrating wound of the anterior abdominal wall the wound tract went above the lesser
curvature of stomach. What peritoneum formation is most likely to be injured?
A. * Ligamentum hepatogastricum
B. Ligamentum gastrocolicum
C. Ligamentum hepatoduoduodenale
D. Ligamentum hepatorenale
E. Ligamentum triangulare sinistrum
337. In course of a small pelvis operation it became necessary to ligate an ovarian artery. What formation
may be accidentally ligated together with it?
A. * Ureter
B. Uterine tube
C. Round ligament of uterus
D. Internal iliac vein
E. Urethra
338. In course of an operation surgeon removed a part of a lung that was ventilated by a tertiary bronchus
accompanied by branches of pulmonary artery and other vessels. What part of a lung was removed?
A. * Bronchopulmonary segment
B. Middle lobe
C. Inferior lobe
D. Superior lobe
E. Pulmonary lobule
339. In the specimen of one of the parts of respiratory system a tubular organ was found. It has low
epithelium, well developed muscular tunic, glands and cartilage are absent. Name this organ:

A. * Minor bronchs
B. Trachea
C. Larynx
D. Major bronchs
E. Median bronchs
340. Inflammation of the tympanic cavity (purulent otitis media) was complicated by inflammation of
mammillary process sockets. What wall of tympanic cavity did the pus penetrate into the sockets
through?
A. * Posterior
B. Anterior
C. Medial
D. Lateral
E. Superior
341. Inflammatory process of modified subserous layer around cervix of the uterus caused an intensive
pain syndrome. In what region of genitals does the pathological process take place?
A. * Parametrium
B. Mesometrium
C. Myometrium
D. Endometrium
E. Perimetrium
342. Micro specimen of spinal cord contains a nucleus that should be analyzed. Its neurons form motor
endings in the skeletal muscles. What nucleus of spinal cord is meant?
A. * Proper nucleus of the anterior horn
B. Thoracic nucleus
C. Intermediate lateral nucleus
D. Proper nucleus of the posterior horn
E. Proper nucleus of gray substance
343. Neurological examination of a 65 y.o. patient revealed a haemorrhage within the superior temporal
gyrus. In the blood supply area of which artery is it?
A. * Middle cerebral artery
B. Anterior cerebral artery
C. Posterior cerebral artery
D. Anterior communicating artery
E. Basilar artery
344. Obturative jaundice developed in a 60-year-old patient because of malignant tumour of the big
papillary of the duodenal. Lumen of what anatomical structure is squeezed with tumour?
A. * Hepatopancreatic ampulla
B. Cystic duct
C. Common hepatic duct
D. Right hepatic duct
E. Left hepatic duct
345. Ovarian tumour was diagnozed in the woman. Surgery was indicated. What ligament should be cut
by the surgeon to disconnect the ovary and the uterus?
A. * The ovarial ligament
B. Broad ligament of uterus
C. Lateral umbilical ligament
D. Suspensory ligament of ovary

E. Round ligament of uterus


346. Patient complains of frequent and difficult urination. Imperfection of what formation can cause it?
A. * Prostate
B. Testicles
C. Bulb-uretic glands
D. Testicle adnexa
E. Sperm bubbles
347. Preventive examination of a patient revealed an enlarged lymph node of metastatic origin on the
medial wall of the left axillary crease. Specify the most likely localization of the primary tumour:
A. * Mammary gland
B. Submandibular salivary gland
C. Lung
D. Stomach
E. Thyroid gland
348. Preventive examination of a patient revealed an enlarged lymph node of metastatic origin on the
medial wall of the left axillary crease. Specify the most likely localization of the primary tumour:
A. * Mammary gland
B. Submandibular salivary gland
C. Lung
D. Stomach
E. Thyroid gland
349. Pyeloureterography X-ray photo showed a renal pelvis with minor calyces only (major calyces were
absent). What form of urinary tracts of a kidney was revealed?
A. * Embryonal
B. Fetal
C. Mature
D. Ampullar
E. -
350. The cerebrospinal fluid is being examined for the purpose of diffrential meningitis diagnostics. At
what site is the lumbal puncture safe?
A. * L III-L IV
B. L II-L III
C. L I-L II
D. Th XII-L I
E. L V-S I
351. The electronic microphoto of kidney fragment has exposed afferent glomerular arteriole, which has
giant cells under its endothelium, containing secretory granules. Name the type of these cells:
A. * Juxtaglomerular
B. Mesangial
C. Smoothmuscular
D. Juxtavascular
E. Interstitial
352. Ultrasonic examination of a patient revealed aneurism in the area of aortic arch that caused alteration
of vocal function of larynx. What nerve was constricted?
A. * Recurrent laryngeal
B. Diaphragmatic
C. Superior laryngeal

D. Mandibular
E. Sublingual
353. Vegetative abnormalities in the sleep, heat regulation, all kinds of metabolism, diabetes insipidus are
developing in the patient due to grouth of the tumour in the III ventricle of brain. Irritation of the
nucleus of what part of the brain can cause this symptoms?
A. * Hypothalamus
B. Cerebral peduncles (cruces cerebri)
C. Mesencephalic tegmentum
D. Pons cerebelli
E. Medulla
354. While examining the oral cavity a stomatologist revealed inflammation of papillae on the border of
the median and posterior third of the back of tongue. What papillae are inflamed?
A. * Papillae vallatae
B. Papillae fungiformes
C. Papillae foliatae
D. Papillae filiformes
E. Papillae Conicae
355. While of oral cavity examination the dentist revealed the formation of the first big cheekteeth on the
lower jaw of a child. How old is this child?
A. * 6-7 years old
B. 4-5 years old
C. 8-9 years old
D. 10-11 years old
E. 12-13 years old
356. X-ray examination revealed an accumulation of suppuration in maxillary sinus. Into what nasal
meatus excretes the suppuration?
A. * Median nasal
B. Nasopharyngeal
C. Inferior nasal
D. Superior nasal
E. Common nasal
357. A 17 year old student pressed out a pustule in the medial angle of eye. In 2 days she was taken to the
institute of neurosurgery with thrombosis of cavernous sinus. Through what vein did the infection get
into this sinus?
A. * V.angularis
B. V.maxillarіs
C. V.profunda faciei
D. V.transversa faciei
E. V.diploicae frontalis
358. A 28 year old man with cut wound of frontal skin was admitted to the hospital. A vessel that supplies
blood to the frontal part of head was ligated in order to stop bleeding. What vessel was ligated?
A. * A.supraorbitalis
B. A.infraorbitalis
C. A.angularis
D. A.dorsalis nasi
E. A.temporalis superficialis
359. A 60 year old patient has problems with formation and moving of food mass, it disturbs eating
process. His tongue is stiff, speaking is impossible. What nerve is damaged?

A. * XII
B. V
C. IX
D. XI
E. VII
360. A patient complains of aching gums and maxillary teeth. What nerve is inflamed?
A. * ІI branch of the V pair
B. III branch of the V pair
C. I branch of the V pair
D. Sublingual
E. Accessory
361. A patient consulted a doctor about the inflammation of the ethmoid bone cells (ethmoiditis).
Examination revealed the disorder of blood supply to the bone. The ethmoidal cells are normally
supplied with blood by the branches of the following artery:
A. * A. ophthalmica
B. A. infraorbitalis
C. A. facialis
D. A. cerebri anterior
E. A. transversa faciei
362. A patient consulted dental surgeon about an injury of submandibular triangle. During the wound
cleansing the surgeon found that the artery leading to the soft palate is damaged. What artery is
damaged?
A. * A. palatina ascendens
B. A. palatina descendens
C. A. sphenopalatina
D. A. pharingea ascendens
E. A. facialis
363. A patient has air embolism as a result of a skin injury in the middle portion of the
sternocleidomastoid muscle. Which cervical vein was injured?
A. * External jugular vein
B. Anterior jugular vein
C. Internal jugular vein
D. Posterior auricular vein
E. Transverse cervical vein
364. A patient has assymetric face, it is especially noticeable during active muscle contraction. What nerve
may be damaged?
A. * Facial (motor unit)
B. Trigeminal, І branch
C. TrigeminaІ, IІ branch
D. Trigeminal, ІІІ branch
E. Sublingual
365. A patient has difficulties with jaw joining when he is chewing. There is partial atrophy of masticatory
muscles situated below the zygomatic arch. What nerve branches do these muscles innervate?

A. * N. mandibularis
B. N. maxillaris
C. N. alveolaris inferior
D. Nn. alveolares superiores

E. N. infraorbitalis
366. A patient has secretory dysfunction of the submandibular salivary galnd. Which nerve is responsible
for its vegetative innervation?
A. * Chorda tympani
B. N. auriculotemporalis
C. N. mandibularis
D. N. petrosus major
E. N. petrosus minor
367. A patient suffers from middle ear inflammation (otitis). He complains also of disordered test
sensation in the anterior tongue part. What nerve is damaged?
A. * N. facialis
B. N. trigeminus
C. N. vestibulo-cochlearis
D. N. vagus
E. N. glossopharyngeus
368. A patient with cancer of the back of tongue had an intensive bleeding as a result of tumor spread to
the dorsal artery of tongue. What vessel should be ligated in order to stop bleeding?
A. * Lingual artery
B. Dorsal artery of tongue
C. Deep artery of tongue
D. Facial artery
E. Ascending pharyngal artery
369. A woman in grave condition was admitted to a hospital with thediagnosis of the hemorrhagic stroke
in the region of frontal part of the right cerebral hemisphere. The damage of what artery most likely
causedthis condition?
A. * A. сerebri anterior
B. A. cerebri posterior
C. A. communicans anterior
D. A. cerebri media
E. A. communicans posterior
370. After an operation a patient's sensitivity of front and lateral surface of neck has reduced. What nerve
is damaged?
A. * N. transversus colli
B. N. auricularis magnus
C. Nn. supraclaviculares
D. N. occipitalis minos
E. N. phrenicus
371. An 18-year-old man was delivered to the hospital after a road accident. Examination at the
traumatological department revealed multiple injuries of soft tissues of face in the region of the
medial eye angle. The injuries caused massive haemorrhage. What arterial anastomosis might have
been damaged in this region?
A. * a. carotis externa et a. carotis interna
B. a. carotis externa et a. subclavia
C. a. carotis interna eta. subclavia
D. a. subclavia et a. ophthalmica
E. a. carotis interna et a. ophthalmica
372. During ablation of the nose wing lypoma a dentist injured a vessel, that caused a saphenous
hematoma. What vessel was damaged?

A. * А.facialis
B. А.maxillaris
C. А.supraorbitalis
D. А.infraorbitalis
E. А.angularis
373. In order to make a functional complete denture the left superior canine of a patient should be
extracted. After the infraorbital anesthesia the patient got a rapidly growing hematoma in the front
part of face. It was found that the injured artery is a branch of:
A. * А. maxillaris
B. А. alveolaris inferior
C. А. temporalis superficialis
D. А. ophthalmica
E. А. labialis superior
374. Surgical approach to the thyroid gland from the transverse (collar) approach involves opening of
interaponeurotic suprasternal space. What anatomic structure localized in this space is dangerous to
be damaged?
A. * Jugular venous arch
B. External jugular vein
C. Subclavicular vein
D. Inferior thyroid arthery
E. Superior thyroid arthery
375. The patient with thymoma (thymus gland tumour) has cyanosis, extention of subcutaneous venousnet
and edema of the soft tissues of face, neck, upper part of the trunk and upper extremities. What
venous trunk is pressed with tumour?
A. * Superior vena cava
B. External jugular vein
C. Clavicular vein
D. Internal jugular vein
E. Frontal jugular vein
376. When a patient puts his tongue out the tip of it deflects to the left. Motor innervation of what cranial
nerve is disturbed in this case?
A. * N. hypoglossus dexter
B. N. glossopharyngeus dexter
C. N. vagus dexter
D. N. trigeminus sinister
E. N. facialis sinister
377. A 19 year-old patient was diagnosed with appendicitis and was hospitalized. The surgical operation
on ablating appendix vermiformis is to be performed. What artery must be fixed to stop bleeding
during the surgical operation?
A. * The ileocolic artery
B. The colica dextra
C. The colica media
D. The colica sinistra
E. The iliac
378. A 50 year-old patient was admitted to the hospital with complaints about pain behind his breastbone,
asphyxia during physical activity. Angiography revealed pathological changes in the posterior
interventricular branch of the right coronary artery. What heart parts are affected?
A. * Posterior wall of the right and left ventricles

B. Left atrium
C. Anterior wall of the right and left ventricles
D. Right atrium
E. Right atrioventricular valve
379. A 54-year-old man was admitted to the hospital with complaints of pain in the right subcostal region,
vomiting with blood. Objectively: enlarged liver, varicose veins in the stomach and esophagus.
Disfunction of what vessel is likely to be?
A. * Vena porta
B. Aorta abdominalis
C. Vena hepatica
D. Vena cava superior
E. Vena cava inferior
380. A 70 year old female patient was diagnosed with fracture of left femoral neck accompanied by
disruption of ligament of head of femur. The branch of the following artery is damaged:
A. * Obturator
B. Femoral
C. External iliac
D. Inferior gluteal
E. Internal pudendal
381. A man suffering from osteochondrosis got acute pain in the abdominal muscles (lateral and anterior).
During objective examination a physician diagnosticated increased pain sensitivity of skin in the
hypogastric region. This pain might be caused by affection of the following nerve:
A. * Iliohypogastric
B. Sciatic
C. Obturator
D. Femoral
E. Genitofemoral
382. A man with a stab wound in the area of quadrilateral foramen applied to a doctor. Examination
revealed that the patient was unable to draw his arm aside from his body. What nerve is most
probably damaged?
A. * N. axillaris
B. N. medianus
C. N.radialis
D. N. ulnaris
E. N. Musculocutaneus
383. A patient complains about edema of legs, skin cyanosis, and small ulcers on one side of the lateral
condyle. Examination revealed a swelling, enlarged veins, formation of nodes. The pathological
process has started in the following vein:
A. * V. saphena parva
B. V. saphena magna
C. V. femoralis
D. V. profunda femoris
E. V. iliaca externa
384. A patient has a malignisation of thoracic part of esophagus. What lymphatic nodes are regional for
this organ?
A. * Anulus lymphaticus cardiae
B. Nodi lymphatici paratrachealis
C. Nodi lymphatici prevertebralis

D. Nodi lymphatici pericardiales laterales


E. Nodi lymphatici mediastinales posteriores
385. A patient has an exudative pleurisy. At what level should the pleural puncture along the posterior
axillary line be taken?
A. * ІХ intercostal space
B. VІІІ intercostal space
C. VІІ intercostal space
D. ХІ intercostal space
E. VІ intercostal space
386. A patient has lost skin sensitivity in the region of the medial surface of his shoulder. This is the result
of dysfunction of the following nerve:
A. * Medial brachial cutaneous nerve
B. Medial antebrachial cutaneous nerve
C. Radial nerve
D. Ulnar nerve
E. Axillary nerve
387. A patient has lost skin sensitivity in the region of the medial surface of his shoulder. This is the result
of dysfunction of the following nerve:
A. * Medial brachial cutaneous nerve
B. Medial antebrachial cutaneous nerve
C. Radial nerve
D. Ulnar nerve
E. Axillary nerve
388. A patient has tissue ischemia below the knee joint accompanied with intermittent claudication. What
artery occlusion should be suspected?
A. * Popliteal artery
B. Peroneal artery
C. Posterior tibial artery
D. Anterior tibial artery
E. Proximal part of femoral artery
389. A patient in three weeks after acute myocardial infarction has pain in the heart and joints and
pneumonia. What is the main mechanism of development of post-infarction Dressler’s syndrome?
A. * Autoimmune inflammation
B. Ischemia of myocardium
C. Resorption of enzymes from necrotized area of myocardium
D. Secondary infection
E. Vessels thrombosis
390. A patient with neuritis of femoral nerve has disturbed flexion of thigh as well as disturbed crus
extension in the knee joint. What muscle's function is disturbed?
A. * Quadriceps muscle of thigh
B. Biceps muscle of thigh
C. Triceps muscle of thigh
D. Semitendinous muscle
E. Semimembranous muscle
391. A patient's knee joint doesn't extend, there is no knee-jerk reflex, skin sensitivity of the anterior
femoral surface is disturbed. What nerve structures are damaged?
A. * Femoral nerve
B. Superior gluteal nerve

C. Big fibular nerve


D. Obturator nerve
E. Inferior gluteal nerve
392. A woman consulted a doctor about swelling and tenderness of the lower extremity, swollen veins and
nodes on the medial surface of thigh. Which vein was affected?
A. * Great saphenous
B. Small saphenous
C. Femoral
D. Popliteal
E. Tibial
393. A woman suffering from osteochondrosis felt acute pain in her humeral articulation that became
stronger when she abducted her shoulder. These symptoms might be caused by damage of the
following nerve:
A. * Axillary nerve
B. Subscapular nerve
C. Dorsal scapular nerve
D. Subclavicular nerve
E. Throracodorsal nerve
394. A woman underwent an operation on account of extrauterine (tubal) pregnancy. In course of the
operation the surgeon should ligate the branches of the following arteries:
A. * Uterine and ovarian
B. Superior cystic and ovarian
C. Inferior cystic and ovarian
D. Uterine and superior cystic
E. Uterine and inferior cystic
395. After a road accident a driver was delivered to the hospital with an injury of the medial epicondyle of
humerus. What nerve might be damaged in this case?
A. * n. ulnaris
B. n. radialis
C. n. axillaris
D. n. muscolocutaneus
E. n. medianus
396. After resection of the middle third of femoral artery obliterated by a thrombus the lower extremity is
supplied with blood due to the surgical bypass. Name an artery that plays the main role in
reestablishment of blood flow:
A. * Deep femoral artery
B. Superficial circumflex artery of hip bone
C. Descending geniculat artery
D. Superficial epigastric artery
E. Deep external pudendal artery
397. After trauma a 44-year-old patient had a rupture of left palm muscle tendons and of the surface of
blood vessels. After operation and removal of the most part of the necrotically changed muscle tissue
the bloodstream was normalized. What vessels have helped with restoration of bloodstream?
A. * Arcus palmaris profundus
B. Arcus palmaris superficialis
C. Aa. digitales palmares communes
D. Aa. metacarpeae palmares
E. Aa. perforantes

398. During the fetal period of the development in the vascular system of the fetus large arterial (Botallo's)
duct is functioning which converts intolig. arteriosum after birth. What anatomical formations does
this duct connect between each other?
A. * Pulmonary trunk and aorta
B. Right and left auricle
C. Aorta and inferior vena cava
D. Pulmonary trunk and superior vena cava
E. Aorta and superior vena cava
399. Examination of a 2-year-old child revealed physical developmental lag, the child often has
pneumonias. The child was diagnosed with nonclosure of ductus arteriosus. Haemodynamics disorder
was caused by the intercommunication of the following vessels:
A. * Aorta and pulmonary trunk
B. Pulmonary trunk and pulmonary veins
C. Superior cava and aorta
D. Superior cava and pulmonary trunk
E. Aorta and pulmonary veins
400. On examination of a road accident victim a doctor revealed left clavicle fracture and disturbed blood
circulation in an extremity (no pulsing of radial artery). What cause of blood circulation disturbance
is the most probable?
A. * Compression of subclavian artery
B. Compression of axillary artery
C. Compression of subclavian vein
D. Compression of vertebral artery
E. Compression of axillary vein
401. The patient has come to the hospital from the smelting workshop in the condition of hyperthermia.
What is the direct cause of loss of consciousness at the heat stroke?
A. * Decreased brain blood supply
B. Arterial pressure drop
C. Increased water loss through sweating
D. Decrease of heart output
E. Dilatation of peripheral vessels
402. The process of heart transplantation determined the viability of myocardial cells. The determination
of what myocardium parameter is the most important?
A. * Rest potential of cardiomyocytes
B. Heart temperature
C. Concentration of oxygen in heart vessels
D. Concentration of calcium-ions in myofibrils
E. Concentration of Ca-ions in heart vessels
403. Victim has elbow joint trauma with avulsion of medial epicondyle of humerus. What nerve can be
damaged in this trauma?
A. * Ulnar
B. Radial
C. Musculocutaneous nerve
D. Cardiac cutaneous nerve
E. Medial cutaneous nerve of forearm
404. Where should the catheter for evacuation of the lymph from the thoracic lymph duct be inserted?
A. * To the left venous corner
B. To the right venous corner

C. To the superior vena cava


D. To the inferior vena cava
E. To the left inguinal vein
405. While palpating mammary gland of a patient a doctor revealed an induration in form of a node in the
inferior medial quadrant. Metastases may extend to the following lymph nodes:
A. * Parasternal
B. Posterior mediastinal
C. Profound lateral cervical
D. Bronchopulmonary
E. Superior diaphragmal
406. While performing an operation in the area of armpit crease a surgeon has to define an arterial vessel
surrounded by fascicles of brachial plexus. What artery is it?
A. * A. axillaris
B. A. vertebralis
C. A. transversa colli
D. A. profunda brachii
E. A. subscapularis
407. While preparing a patient to the operation the heart chambers' pressure was measured. In one of them
the pressure changed during one heart cycle from 0 to 120 mm Hg. What chamber of heart was it?
A. * Left ventricle
B. Right ventricle
C. Right atrium
D. Left atrium
E. Atrium
408. A baby in the fetal position has most of its joints
A. * flexed
B. adducted
C. circumducted
D. extended
E. abducted
409. A person who is standing and bends to tie their shoe is ______ trunk.
A. * flexing
B. adducting
C. circumduction
D. extending
E. abducting
410. Kicking a football straight ahead with your toes involves what movement of your leg?
A. * extension
B. adduction
C. circumduction
D. abduction
E. flexion
411. Movement of the body part away from the main axis of the body, or away from the midsagittal plane,
is the definition of:
A. * Abduction
B. Flexion
C. Extension

D. Circumduction
E. Adduction
412. Shrugging your shoulders to indicate you don't know the answer to a question involves
A. * elevation of the scapulae
B. lateral excursion of the scapulae
C. depression of the scapulae
D. protraction of the scapulae
E. retraction of the scapulae
413. Sometimes people with TMJ disorders bruxate (grind::Sometimes people with TMJ disorders bruxate
(grind side-to-side) their teeth at night This movement is
A. * lateral excursion of the mandible
B. medial and lateral rotation of the mandible
C. circumduction of the mandible
D. flexion and extension of the mandible
E. abduction and adduction of the mandible
414. Traumatologist examines the patient with backbone pain. The anterior part of the vertebra is:
A. * Body
B. Arcus
C. Foramen
D. Superior incisura
E. Inferior incisura
415. The spinal cord located in:
A. * Vertebral canal
B. Sacral canal
C. Intervertebral foramen
D. Superior incisura
E. Inferior incisura
416. Traumatologist examines the patient with backbone pain. The spinal nerves pass through:
A. * Intervertebral foramen
B. Sacral canal
C. Vertebral canal
D. Superior incisura
E. Inferior incisura
417. Tilting the head posteriorly to look up at the sky requires
A. * extension of the neck
B. adduction of the neck
C. circumduction of the neck
D. abduction of the neck
E. flexion of the neck
418. To walk on her toes, a ballerina must be able______ to her feet for long periods of time
A. * plantarflex
B. dorsiflex
C. evert
D. invert (supinate)
E. abduct
419. Traumatologist examines the patient with backbone pain. How many cervical vertebrae are there?

A. * 7
B. 5
C. 8
D. 10
E. 12
420. When a suspect is arrested by the police, typically they are handcuffed behind their back. In this
position, the suspect's arms are
A. * extended
B. adducted
C. circumducted
D. abducted
E. flexed
421. When you are going to place something in the palm of your hand, your hand must be
A. * supinated
B. adducted
C. flexed
D. pronated
E. abducted
422. Which process take place in formation of joints with ribs?
A. * Transverse
B. Spinous
C. Articular
D. Mammilary
E. Costotransversal
423. You are sitting on a chair. To stand up, you must______your thighs and your legs
A. * extend
B. abduct
C. adduct
D. flex
E. rotate
424. At the damage of occipital region the contents of which from the adopted canal can be damaged?
A. * hypoglossal
B. tubular
C. tympanic
D. carotid
E. facial
425. At the damage of the base of skulll the contents of which from the adopted canals can be damaged?
A. * pterygoid
B. tubular
C. tympanic
D. carotid
E. facial
426. The contents of which from the adopted canals can be damaged at the trauma of orbit?
A. * optic
B. tubular
C. tympanic

D. carotid
E. facial
427. Which canal passes in occipital bone?
A. tubular
B. tympanic
C. carotid
D. facial
E. * hypoglossal
428. Which from the adopted canals is not exist?
A. * sphenoid
B. tubular
C. tympanic
D. carotid
E. facial
429. Which from the adopted canals belongs to sphenod bone?
A. * optic
B. tubular
C. tympanic
D. carotid
E. facial
430. Which from the adopted canals belongs to sphenod bone?
A. * pterygoid
B. tubular
C. tympanic
D. carotid
E. facial
431. Which from the adopted canals belongs to occipital bone?
A. * hypoglossal
B. tubular
C. tympanic
D. carotid
E. facial
432. A man with the trauma of head acted in a hospital. The break of sphenoid bone is diagnosed in the
area of basis of pterygoid process The contents of which from the adopted canals can be damaged?
A. * pterygoid
B. tubular
C. tympanic
D. carotid
E. facial
433. A man with the trauma of head acted in a hospital. The break of temporal bone is diagnosed in the
area of fossula petrosaa.The contents of which from the adopted canaliculus can be damaged?
A. * tympanic
B. tubular
C. mastoid
D. carotid
E. facial

434. A man with the trauma of head acted in a hospital. The break of sphenoid bone is diagnosed in the
area of basis of pterygoid process The table of contents of which from the adopted canals can be
damaged?
A. * pterygoid
B. tubular
C. tympanic
D. carotid
E. facial
435. As a result of tumour of hypophysis at sciagraphy the destruction and increase of pit of the Sella
turcica is exposed. What bone cavity is there?
A. * sinuses of sphenoid bon
B. the carotid canal
C. the optic canal
D. the tympanic cavity
E. the facial canal
436. As a result of tumour of middle concha at sciagraphy the destruction and increase of sinus is exposed.
What bone cavity is here staggered?
A. * the anterior ethmoidal air cells
B. the carotid canal
C. the facial canal
D. the tympanic cavity
E. sinuses of sphenoid
437. As a result of tumour of middle concha at sciagraphy the destruction and increase of sinus is exposed.
What bone cavity is here staggered?
A. * the middle ethmoidal air cells
B. the carotid canal
C. the facial canal
D. the tympanic cavity
E. sinuses of sphenoid
438. As a result of tumour of middle concha at sciagraphy the destruction and increase of sinus is exposed.
What bone cavity is here staggered?
A. * the anterior ethmoidal air cells
B. the carotid canal
C. the facial canal
D. the tympanic cavity
E. sinuses of sphenoid
439. As a result of tumour of middle concha at sciagraphy the destruction and increase of sinus is exposed.
What bone cavity is here staggered?
A. * the middle ethmoidal air cells
B. the carotid canal
C. the facial canal
D. the tympanic cavity
E. sinuses of sphenoid
440. As a result of tumour of middle concha at sciagraphy the destruction and increase of sinus is exposed.
What bone cavity is here staggered?
A. * the anterior ethmoidal air cells
B. the carotid canal
C. the facial canal

D. the tympanic cavity


E. sinuses of sphenoid
441. As a result of tumour of superior concha at sciagraphy the destruction and increase of sinus is
exposed. What bone cavity is here staggered?
A. * the posterior ethmoidal air cells
B. the carotid canal
C. the facial canal
D. the tympanic cavity
E. sinuses of sphenoid
442. As a result of tumour of superior concha at sciagraphy the destruction and increase of sinus is
exposed. What bone cavity is here staggered?
A. * sinuses of sphenoid
B. the carotid canal
C. the anterior ethmoidal air cells
D. the tympanic cavity
E. the facial canal
443. As a result of tumour the frontal lobes of the brain at sciagraphy the destruction and increase of pit of
the bone is exposed. What bone cavity is here staggered?
A. * sinuses of frontal bon
B. the carotid canal
C. the optic canal
D. sinuses of sphenoid bone
E. the facial canal
444. In flammation of tympanic cavity (festering otitis) at a patient was complicated by inflammation of
cells of mastoid process. Through what wall of tympanic cavity does pus get to the cells?
A. * posterior
B. superior
C. lateral
D. anterior
E. medial
445. A child 3 years old entered clinic with a diagnosis "festering otitis". Where one can a pus from a
tympanic cavity get?
A. * In a mastoid antrum
B. In back cranial fossa
C. In an external auditory meatus
D. In an internal ear
E. In an auditory tube
446. A child 5 years old entered of clinical hospital with a diagnosis "festering inflammation of middle ear
(timpanit)". The disease began from inflammation of nasophartynx. What canal of temporal bone did
an infection get through in a tympanic cavity?
A. * musculo-tubarius canal
B. the carotid canal
C. canaliculus of chorda tympani
D. carotico-tympanic canaliculi
E. tympanic canaliculus
447. A driver of a car got the trauma of the head because of a blow of a wheel. Which bone can be hurted
from the facial norma?
A. * Maxilla

B. Occipital bone
C. Temporal bone
D. Sphenoid bone
E. Parietal bone
448. A man with the trauma of head acted in a hospital. The break of temporal bone is diagnosed in the
area of jugular fossa. The contents of which from the adopted canaliculus can be damaged?
A. * mastoid
B. tubular
C. sphenoid
D. carotid
E. facial
449. A man with the trauma of head acted in a hospital. The break of temporal bone is diagnosed in the
area of stylomastoid foramen. The contents of which from the adopted canal can be damaged?
A. * facial
B. tubular
C. tympanic
D. carotid
E. sphenoid
450. A man with the trauma of head acted in a hospital. The break of temporal bone is diagnosed in the
area of hiatus of canal for minoris petrosal n. The contents of which from the adopted canaliculus can
be damaged?
A. * tympanic
B. tubular
C. sphenoid
D. carotid
E. facial
451. A man with the trauma of head acted in a hospital. The break of temporal bone is diagnosed in the
area of internal acoustic meatus. The contents of which from the adopted canal can be damaged?
A. * facial
B. tubular
C. tympanic
D. carotid
E. sphenoid
452. A man with the trauma of head acted in a hospital. The break of temporal bone is diagnosed in the
area of jugular fossa. The contents of which from the adopted canaliculus can be damaged?
A. * mastoid
B. tubular
C. sphenoid
D. carotid
E. facial
453. A man with the trauma of head acted in a hospital. The break of temporal bone is diagnosed in the
area of fossula petrosa. The contents of which from the adopted canaliculus can be damaged?
A. * tympanic
B. tubular
C. mastoid
D. carotid
E. facial

454. A man with the trauma of head acted in a hospital. The break of temporal bone is diagnosed in the
area of stylomastoid foramen. The contents of which from the adopted canal can be damaged?
A. * facial
B. tubular
C. tympanic
D. carotid
E. sphenoid
455. A man with the trauma of head acted in a hospital. The break of temporal bone is diagnosed in the
area of fissura petrotympanica. The contents of which from the adopted canaliculus can be damaged?
A. * chordae tympani
B. tympanic
C. mastoid
D. carotid
E. facial
456. A man with the trauma of head acted in a hospital. The break of temporal bone is diagnosed in the
area of fissura tympanomastoidea. The contents of which from the adopted canaliculus can be
damaged?
A. * mastoid
B. tubular
C. tympanic
D. carotid
E. facial
457. A man with the trauma of head acted in a hospital. The break of temporal bone is diagnosed in the
area of hiatus of canal for minoris petrosal nerves. The contents of which from the adopted
canaliculus can be damaged?
A. * tympanic
B. tubular
C. sphenoid
D. carotid
E. facial
458. A man with the trauma of head acted in a hospital. The break of temporal bone is diagnosed in the
area of internal acoustic meatus. The contents of which from the adopted canal can be damaged?
A. * facial
B. tubular
C. carotid
D. sphenoid
E. tympanic
459. A man with the trauma of head acted in a hospital. The break of temporal bone is diagnosed in the
area of stylomastoid foramen. The contents of which from the adopted canal can be damaged?
A. * facial
B. tubular
C. tympanic
D. carotid
E. sphenoid
460. A man with the trauma of head acted in a hospital. The break of sphenoid bone is diagnosed in the
area of basis of anterior clinoid process. The contents of which from the adopted canals can be
damaged?
A. * optic

B. tubular
C. tympanic
D. carotid
E. facial
461. A man with the trauma of head acted in a hospital. The break of temporal bone is diagnosed in the
area of internal acoustic meatus. The contents of which from the adopted canal can be damaged?
A. * facial
B. tubular
C. tympanic
D. carotid
E. sphenoid
462. A man with the trauma of head acted in a hospital. The break of temporal bone is diagnosed in the
area of facial canal. The contents of which from the adopted canaliculus can be damaged?
A. * chordae tympani
B. tympanic
C. mastoid
D. carotid
E. pterygoid
463. A man with the trauma of head acted in a hospital. The break of temporal bone is diagnosed in the
area of hiatus of canal for minoris petrosal n. The contents of which from the adopted canaliculus can
be damaged?
A. * tympanic
B. tubular
C. sphenoid
D. carotid
E. facial
464. A man with the trauma of head acted in a hospital. The break of temporal bone is diagnosed in the
area of fissura tympanomastoidea. The contents of which from the adopted canaliculus can be
damaged?
A. * mastoid
B. tubular
C. tympanic
D. carotid
E. facial
465. A man with the trauma of head acted in a hospital. The break of temporal bone is diagnosed in the
area of fissura petrotympanica. The contents of which from the adopted canaliculus can be damaged?
A. * chordae tympani
B. tympanic
C. mastoid
D. carotid
E. facial
466. A man with the trauma of head acted in a hospital. The break of temporal bone is diagnosed in the
area of jugular fossa. The contents of which from the adopted canaliculus can be damaged?
A. * mastoid
B. tubular
C. sphenoid
D. carotid
E. facial

467. Inflammation of tympanic cavity (festering otitis) at a patient was complicated by inflammation of
cells of mastoid process. Through what wall of tympanic cavity does pus get to the cells?
A. lateral
B. superior
C. * posterior
D. anterior
E. medial
468. Inflammation of tympanic cavity (festering otitis) at a patient was complicated by inflammation of
cells of mastoid process. Through what wall of tympanic cavity does pus get to the cells?
A. * posterior
B. superior
C. lateral
D. anterior
E. medial
469. A child 3 years old entered clinic with a diagnosis "festering otitis". Where one can a pus from a
tympanic cavity get?
A. * In a mastoid antrum
B. In back cranial fossa
C. In an external auditory passage-way
D. In an internal ear
E. In an auditory tube
470. A child 5 years old entered of clinical hospital with a diagnosis "festering inflammation of middle ear
(timpanit)". The disease began from inflammation of nasophartynx. What canal of temporal bone did
an infection get through in a tympanic cavity?
A. * musculo-tubular canal
B. the carotid canal
C. canaliculus of chorda tympani
D. carotico-tympanic canaliculi
E. tympanic canaliculus
471. A driver of a car got the trauma of the head because of a blow of a wheel. Which bone can be hurted
from the facial norma?
A. * Maxilla
B. Occipital bone
C. Temporal bone
D. Sphenoid bone
E. Parietal bone
472. A man with the trauma of head acted in a hospital. The break of temporal bone is diagnosed in the
area of jugular fossa. The contents of which from the adopted canaliculus can be damaged?
A. * mastoid
B. tubular
C. sphenoid
D. carotid
E. facial
473. A man with the trauma of head acted in a hospital. The break of sphenoid bone is diagnosed in the
area of basis of pterygoid process. The contents of which from the adopted canals can be damaged?
A. * pterygoid
B. tubular
C. tympanic

D. carotid
E. facial
474. A man with the trauma of head acted in a hospital. The break of temporal bone is diagnosed in the
area of fissura petrotympanica. The contents of which from the adopted canaliculus can be damaged?
A. * chordae tympani
B. tympanic
C. mastoid
D. carotid
E. facial
475. A man with the trauma of head acted in a hospital. The break of temporal bone is diagnosed in the
area of fissura tympanomastoidea. The contents of which from the adopted canaliculus can be
damaged?
A. * mastoid
B. tubular
C. tympanic
D. carotid
E. facial
476. 56. A man with the trauma of head acted in a hospital. The break of temporal bone is diagnosed in the
area of facial canal. The contents of which from the adopted canaliculus can be damaged?
A. * chordae tympani
B. tympanic
C. mastoid
D. carotid
E. pterygoid
477. A man with the trauma of head acted in a hospital. The break of temporal bone is diagnosed in the
area of hiatus of canal for minoris petrosal n. The contents of which from the adopted canaliculus can
be damaged?
A. * tympanic
B. tubular
C. sphenoid
D. carotid
E. facial
478. A man with the trauma of head acted in a hospital. The break of temporal bone is diagnosed in the
area of internal acoustic meatus. The contents of which from the adopted canal can be damaged?
A. * facial
B. tubular
C. tympanic
D. carotid
E. sphenoid
479. A man with the trauma of head acted in a hospital. The break of temporal bone is diagnosed in the
area of stylomastoid foramen. The contents of which from the adopted canal can be damaged?
A. * facial
B. tubular
C. tympanic
D. carotid
E. sphenoid
480. A man with the trauma of head acted in a hospital. The break of occipital bone is diagnosed in the
area of occipital condyle. The contents of which from the adopted canal can be damaged?

A. * hypoglossal
B. tubular
C. tympanic
D. carotid
E. facial
481. A man with the trauma of head acted in a hospital. The break of temporal bone is diagnosed in the
area of fossula petrosa. The contents of which from the adopted canaliculus can be damaged?
A. * tympanic
B. tubular
C. mastoid
D. carotid
E. facial
482. A man with the trauma of head acted in a hospital. The break of sphenoid bone is diagnosed in the
area of basis of pterygoid process. The contents of which from the adopted canals can be damaged?
A. * pterygoid
B. tubular
C. tympanic
D. carotid
E. facial
483. In flammation of tympanic cavity (festering otitis) at a patient was complicated by inflammation of
cells of mastoid process. Through what wall of tympanic cavity does pus get to the cells?
A. * posterior
B. superior
C. lateral
D. anterior
E. medial
484. A person who is standing and bends to tie their shoe is _____ his trunk.
A. * flexing
B. adducting
C. circumducting
D. extending
E. abducting
485. A person who is standing in the anatomic position has _______ his hands and forearms
A. * supinated
B. pronated
C. supinated
D. pronated
E. abducted
486. Executing the physical drills, a girl fell down on knee. The next day she began to feel sharp pain in
the the knee. At a review by the doctor of knee region under a skin in the its lower third haemorragia
is exposed. What bones can be damaged?
A. * the patella B. the sciatic bone
487. Kicking a ball straight ahead with your toes involves what movement of your leg?
A. * extension
B. adduction
C. circumduction
D. abduction

E. flexion
488. Movement of the body part to the main axis of the body, or around the vertical axis, is the definition
of:
A. * Supination
B. Extension
C. Adduction
D. Abduction
E. Flexion
489. Movement of the body part to the main axis of the body, or around the vertical axis, is the definition
of:
A. * Circumduction
B. Abduction
C. Adduction
D. Extension
E. Flexion
490. Movement of the body part to the main axis of the body, or around the vertical axis, is the definition
of:
A. * Pronation
B. Abduction
C. Adduction
D. Extension
E. Flexion
491. Movement of the body part to the main axis of the body, or in the frontal plane, is the definition of:
A. * Extension
B. Rotation
C. Abduction
D. Circumduction
E. Adduction
492. Movement of the body part to the main axis of the body, or in the frontal plane, is the definition of:
A. * Flexion
B. Rotation
C. Abduction
D. Circumduction
E. Adduction
493. Movement of the body part to the main axis of the body, or in the sagittal plane, is the definition of:
A. * Adduction
B. Rotation
C. Extension
D. Circumduction
E. Flexion
494. Movement of the body part to the main axis of the body, or in the sagittal plane, is the definition of:
A. * Abduction
B. Rotation
C. Extension
D. Circumduction
E. Flexion
495. A person who is standing in the anatomic position has his hands and forearms.

A. * supinated, extended
B. pronated, extended
C. supinated, flexed
D. pronated, flexed
E. abducted, extended
496. A 45 years old man addressed the traumatologist after the industrial trauma of the back. The
functions of extension of the back is absent. Which ligament covers back surface of vertebral bodies?
A. * Lig. longitudinale posterius
B. Lig. longitudinale anterius
C. Lig. flava
D. Lig. interspinalia
E. Lig. Supraspinale
497. A ligament formed predominantly by elastic fibers which joins the laminae of adjacent vertebrae.
Which ligament was injured?
A. * ligamenta flava
B. interspinous ligament
C. posterior longitudinal ligament
D. anterior longitudinal ligament
E. supraspinous ligament
498. A ligament that connects the tips of the spinous processes of thoracic and lumbar vertebrae, a
syndesmosis; begins at the C7 vertebrae and ends at the mid-sacral segmental level; it serves as a
muscle attachment site.
A. * supraspinous ligament B. nuchal ligament
499. A ligament that courses from superior to inferior along the posterior surfaces of all vertebral bodies.
It is broader at the intervertebral disks and narrow at the vertebral bodies which gives it a scalloped
edge; it is located in the vertebral canal; it is not penetrated by the needle during spinal tap.
A. * posterior longitudinal ligament
B. supraspinous ligament
C. interspinous ligament
D. anterior longitudinal ligament
E. nuchal ligament
500. A patient who suffered in a car accident was examined a fracture of the left scapula with a violation
of blood ciralation. Which articulation was injured?
A. * articul. acromioclavicularis
B. articul. sternocostalis
C. articul. costotransversaria
D. articul. intervertebralis
E. articul. capitis costae
501. A patient who suffered in a car accident was examined a fracture of the left clavice with a violation of
blood ciralation. Which articulation was injured?
A. * articul. sternoclavicularis
B. articul. sternocostalis
C. articul. costotransversaria
D. articul. intervertebralis
E. articul. capitis costae
502. Examining the patient's abdominal cavity with the wound of anterior abdominal wall the surgeon
noticed the functions of flexion of the back is absent. Which ligament was injured?
A. * Lig. longitudinale anterius
B. Lig. longitudinale posterius
C. Lig. flava
D. Lig. interspinalia
E. Lig. supraspinale
503. Movement of the body part in the vertical axis, turning the forearm to bring the back of the hand
forward is the definition of:
A. * Pronation
B. Abduction
C. Adduction
D. Extension
E. Flexion
504. Movement of the body part to the main axis of the body, or in the sagital plane, is the definition of:
A. * Adduction
B. Rotation
C. Extension
D. Circumduction
E. Flexion
505. Movement of the body part in the vertical axis, turning the forearm to bring the back of the hand
forward is the definition of:
A. * Pronation
B. Abduction
C. Adduction
D. Extension
E. Flexion
506. Movement of the body part to the main axis of the body, or in the sagital plane, is the definition of:
A. * Adduction
B. Rotation
C. Extension
D. Circumduction
E. Flexion
507. On roentgenogram of the organs of patients thorax cavity was discovered the increasing a
sternocostal joints. Which of the following structures belongs to the sternocostal joints?
A. * lig. sternocostalis intraarticulare
B. lig. capitis costae radiatum
C. lig. capitis costae intra-articulare
D. lig.transversum
E. lig. Coracoacromiale
508. On roentgenogram of the organs of patients thorax cavity was discovered the increasing a
sternocostal joints. Which of the following structures belongs to the sternocostal joints?
A. * lig. sternocostalis radiatum
B. lig. capitis costae radiatum
C. lig. capitis costae intra-articulare
D. lig.transversum
E. lig. coracoacromiale
509. On roentgenogram of the organs of patients thorax cavity was discovered the increasing a ligament
that extends along midline posteriorly from the spinous processes of cervical vertebrae and extends
from the base of skull to 7th cervical vertebra.
A. * nuchal ligament
B. interspinous ligament
C. posterior longitudinal ligament
D. anterior longitudinal ligament
E. supraspinous ligament
510. On roentgenogram of the organs of patients thorax cavity was discovered the increasing a ligament
that courses from superior to inferior along the posterior surfaces of all vertebral bodies. It is broader
at the intervertebral disks and narrow at the vertebral bodies which gives it a scalloped edge; it is
located in the vertebral canal; it is not penetrated by the needle during spinal tap.
A. * posterior longitudinal ligament
B. supraspinous ligament
C. interspinous ligament
D. anterior longitudinal ligament
E. nuchal ligament
511. On roentgenogram of the organs of patients thorax cavity was discovered the increasing of a
sternocostal joints. Which of the following structures belongs to the sternocostal joints?
A. * lig.sternocostalis radiatum
B. lig. capitis costae radiatum
C. lig. capitis costae intra-articulare
D. lig.transversum
E. lig. Coracoacromiale
512. On roentgenogram of the organs of patients thorax cavity was discovered the increasing of a
sternocostal joints. Which of the following structures belongs to the sternocostal joints?
A. * lig.sternocostalia radiata
B. lig. capitis costae radiatum
C. lig. capitis costae intra-articulare
D. lig.transversum
E. lig. coracoacromiale
513. On roentgenogram of the organs of patients thorax cavity was discovered the increasing of a
sternocostal joints. Which of the following structures belongs to the sternocostal joints?
A. * membrane sterni
B. lig. capitis costae radiatum
C. lig. capitis costae intra-articulare
D. lig.transversum
E. lig. coracoacromiale
514. On roentgenogram of the organs of patients thorax cavity was discovered the increasing of a ligament
that connects the tips of the spinous processes of thoracic and lumbar vertebrae, a syndesmosis;
begins at the C7 vertebrae and ends at the mid-sacral segmental level; it serves as a muscle
attachment site.
A. * supraspinous ligament
B. nuchal ligament
C. posterior longitudinal ligament
D. interspinous ligament
E. anterior longitudinal ligament
515. On roentgenogram of the organs of patients thorax cavity was discovered the increasing of a
sternocostal joints. Which of the following structures belongs of the sternocostal joints?
A. * synchondrosis sternocostalis costae primae
B. lig. capitis costae radiatum

C. lig. capitis costae intra-articulare


D. lig.transversum
E. lig. coracoacromiale
516. Shrugging your shoulders to indicate you don't know the answer to a question involves
A. * elevation of the scapulae
B. lateral excursion of the scapulae
C. depression of the scapulae
D. protraction of the scapulae
E. retraction of the scapulae
517. Sometimes people with TMJ disorders bruxate (grind side-to-side) their teeth at night. This
movement is
A. * lateral excursion of the mandible
B. medial and lateral rotation of the mandible
C. circumduction of the mandible
D. flexion and extension of the mandible
E. abduction and adduction of the mandible
518. Tilting the head posteriorly to look up at the sky requires
A. * extension of the neck
B. adduction of the neck
C. circumduction of the neck
D. abduction of the neck
E. flexion of the neck
519. When a suspect is arrested by the police, typically they are handcuffed behind their back. In this
position, the suspect's arms are
A. * extended
B. adducted
C. circumducted
D. abducted
E. flexed
520. When you is going to place something in the palm of your hand, your hand must be
A. * supinated
B. adducted
C. flexed
D. pronated
E. abducted
521. Kicking a football straight ahead with your toes involves what movement of your thigh?
A. * extension
B. adduction
C. circumduction
D. rotation
E. abduction
522. Kicking a football straight ahead with your toes involves what movement of your leg?
A. * extension
B. adduction
C. circumduction
D. abduction
E. flexion

523. To walk on her toes, a ballerina must be able to her feet for long periods of time and involves:
A. * plantarflex
B. dorsiflex
C. evert
D. invert (supinate)
E. abduct
524. You are sitting on a chair. To stand up, you must _____ your thighs and ______ your legs.
A. abduct, flex
B. * extend, extend
C. extend, flex
D. flex, flex
E. flex, extend
525. Which parts is the upper region of the abdomen divided into?
A. * Regiones hypochondriacae dextra et sinistra, regio epigastrica
B. Regio umbilicalis, regiones hypochondriacae dextra et sinistra
C. Regiones laterales dextra et sinistra, regio umbilicalis
D. Regiones hypochondriacae dextra et sinistra, regio umbilicalis
E. Regiones laterales dextra et sinistra, regio epigastrica
526. The doctor observes the diaphragm. Name the organ that pass through the diaphragm?
A. * The vagus nerve
B. The inferior venae cavae
C. The inferior phrenic arteries
D. The superior phrenic arteries
E. The celiac trunk
527. The doctor observes the organs of the abdominal cavity. Name the walls of the inguinal canal?
A. * Anterior, posterior, superior, inferior
B. Superficial, posterior, superior, inferior
C. External, deep, posterior, superior
D. Internal, posterior, superior, inferior
E. Deep, external, posterior, inferior
528. The doctor observes the organs of the abdominal cavity. Which parts is the lower region of the
abdomen divided into?
A. * Regiones inguinales dextra et sinistra, regio pubica
B. Regio umbilicalis, regiones hypochondriacae dextra et sinistra
C. Regiones hypochondriacae dextra et sinistra, regio epigastrica
D. Regiones hypochondriacae dextra et sinistra, regio umbilicalis
E. Regiones laterales dextra et sinistra, regio epigastrica
529. The doctor observes the organs of the abdominal cavity. Which parts is the middle region of the
abdomen divided into (Latin terms)?
A. * Regiones laterales dextra et sinistra, regio umbilicalis
B. Regio umbilicalis, regiones hypochondriacae dextra et sinistra
C. Regiones hypochondriacae dextra et sinistra, regio epigastrica
D. Regiones hypochondriacae dextra et sinistra, regio umbilicalis
E. Regiones laterales dextra et sinistra, regio epigastrica
530. A patient who suffered in a car accident was examined for fracture of the ribs. What muscles raise the
ribs?
A. * The transversus thoracis muscle

B. The subclavius muscles


C. The external intercostal muscles
D. The internal intercostal muscles
E. The subcostales muscles
531. A patient who suffered in a car accident was examined for fracture of the lower ribs. Which muscle
pulls down the costal cartilages and lower ribs?
A. * The transversus thoracis muscle
B. The subclavius muscles
C. The external intercostal muscles
D. The serratus anterior muscles
E. The pectoralis minor muscle
532. A patient who suffered in a car accident was examined for fracture of the left scapula. Which muscle
shifts the scapula to the front and the side?
A. * The serratus anterior muscle
B. The internal intercostal muscles
C. The subclavius muscle
D. The transversus thoracis muscle
E. The subcostales muscles
533. A patient who suffered in a car accident was examined for fracture of the left clavicle. Which muscle
moves the clavicle downwards and towards the front?
A. * The subclavius muscle
B. The pectoralis minor muscle
C. The pectoralis major muscle
D. The external intercostal muscles
E. The serratus anterior muscle
534. A patient who suffered in a car accident was examined for fracture of the left scapula. Which muscle
bends the scapula forward?
A. * The pectoralis minor muscle
B. The subclavius muscle
C. The pectoralis major muscle
D. The serratus anterior muscle
E. The external intercostal muscle
535. Examining the patient's abdominal cavity with the wound of anterior abdominal wall the surgeon
noticed the hurting of diaphragm. Which organs pass through the diaphragm in its lumbar part?
A. * The aorta
B. The inferior venae cavae
C. The inferior phrenic arteries
D. The superior phrenic arteries
E. The celiac trunk
536. Examining the patient's abdominal cavity with the wound of anterior abdominal wall the surgeon
noticed the hurting of diaphragm. Name the opening of the diaphragm?
A. * The esophageal hiatus
B. The opening for the inferior venae cavae
C. The opening for the inferior phrenic arteries
D. The opening for the superior phrenic arteries
E. The opening for the celiac trunk
537. Examining the patient's abdominal cavity with the wound of anterior abdominal wall the surgeon
noticed the hurting of diaphragm. Name the opening of the diaphragm?

A. * The aortic hiatus


B. The opening for the inferior venae cavae
C. The opening for the inferior phrenic arteries
D. The opening for the superior phrenic arteries
E. The opening for the celiac trunk
538. Examining the patient's abdominal cavity with the wound of anterior abdominal wall the surgeon
noticed the hurting of diaphragm. Name the opening of the diaphragm?
A. * The caval opening
B. The opening for the superior venae cavae
C. The opening for the inferior phrenic arteries
D. The opening for the superior phrenic arteries
E. The opening for the celiac trunk
539. Examining the patient's abdominal cavity with the wound of anterior abdominal wall the surgeon
noticed the hurting of diaphragm. Name the organ that pass through the diaphragm?
A. * The vagus nerve
B. The inferior venae cavae
C. The inferior phrenic arteries
D. The superior phrenic arteries
E. The celiac trunk
540. Examining the patient's abdominal cavity with the wound of anterior abdominal wall the surgeon
noticed the hurting of diaphragm. Name the organ that pass through the diaphragm?
A. * The esophagus
B. The superior venae cavae
C. The inferior phrenic arteries
D. The superior phrenic arteries
E. The celiac trunk
541. On roentgenogram of the organs of patients thorax cavity was discovered the increasing the organ of
the diaphragm. Name the organ that pass through the diaphragm?
A. * The aorta
B. The superior venae cavae
C. The inferior phrenic arteries
D. The superior phrenic arteries
E. The celiac trunk
542. Under ultrasonic examination of the abdominal cavity the doctor observes the muscle. Which muscle
brings the lower ribs down and to the front?
A. The internal oblique muscle of the abdomen
B. * The transverse abdominal muscle
C. The external oblique muscle of the abdomen
D. The pyramidalis muscle
E. The rectus abdominis muscle
543. Under ultrasonic examination of the abdominal cavity the doctor observes the muscle. Which muscle
bends the vertebral column and lowers the ribs?
A. * The internal oblique abdominis muscle
B. The phrenicus muscle
C. The transverse abdominal muscle
D. The quadratus lumborum muscle
E. The pyramidalis muscle

544. Under ultrasonic examination of the abdominal cavity the doctor observes the muscle. Which muscle
lowers the ribs and bends the back bone during bilateral contraction?
A. * The external oblique abdominis muscle
B. The serratus anterior muscle
C. The transverse abdominal muscle
D. The external intercostal muscles
E. The pyramidalis
545. Under ultrasonic examination of the abdominal cavity the doctor observes the muscle. Which muscle
pulls the ribs down, lowering the thorax when the spine and pelvic girdle are fixed?
A. * The rectus abdominis muscle
B. The quadratus lumborum muscle
C. The phrenicus muscle
D. The pyramidalis muscle
E. The transverse abdominal muscle
546. Under ultrasonic examination of the abdominal cavity the doctor observes the muscle. Which muscle
stretches the linea alba of the abdomen?
A. The internal oblique muscle of the abdomen
B. * The pyramidalis muscle
C. The external oblique muscle of the abdomen
D. The transverse abdominal muscle
E. The rectus abdominis muscle
547. Under ultrasonic examination of the abdominal cavity the doctor observes the muscle. Which muscle
helps support the trunk in an upright position during bilateral contraction?
A. * The quadratus lumborum muscle
B. The psoas major muscle
C. The psoas minor muscle
D. The transverse abdominal muscle
E. The rectus abdominis muscle
548. Under ultrasonic examination of the abdominal cavity the doctor observes the muscle. Which muscle
participates in flexion of the femur in the coxa joint?
A. * The psoas major muscle
B. The psoas minor muscle
C. The quadratus lumborum muscle
D. The internal intercostal muscles
E. The phrenicus muscle
549. Under ultrasonic examination of the vertebral column the doctor observes the diaphragm. Name the
organ that pass through the diaphragm?
A. * The inferior venae cavae
B. The superior venae cavae
C. The inferior phrenic arteries
D. The celiac trunk
E. The superior phrenic arteries
550. A 30 year old patient feels a sharp pain when he pulls his jaw to posterior position. The dentist said
that he has the inflammation of one of the masticatory muscle. Which one is damaged?
A. * musculus temporalis (posterior fibers)
B. musculus temporalis (anterior fibers)
C. medial pterygoid muscle
D. lateral pterygoid muscle

E. masseter
551. A child of 5 years old was delivered to the infections department of the hospital with the diagnosis of
diphtheria. In which triangle of the neck is it possible to find the trachea?
A. * omotracheale
B. omotrapezoideum
C. caroticum
D. omoclaviculare
E. submandibulare
552. A patient with a wound of soft tissues of the neck and the damage of the platysma muscle was
delivered to the hospital. Which neck fascia covers this muscle?
A. * fascia colli superficialis
B. fascia prevertebralis
C. fascia endocervicalis
D. lamina profunda fasciae colli propriae
E. lamina superficialis fasciae colli propriae
553. A patient with severe intoxication was admitted to the hospital. To provide treatment it is necessary to
perform catheterination of the subclavian vein. In what area is it located?
A. * spatuim anterscalenum
B. trigonum omotrapezoideum
C. spatuim interscalenum
D. spatium retrosternocleidomastoideus
E. spatuim interaponeuroticum suprasternale
554. During the dissection was discovered pus in space between superficial and deep layers of proper
cervical fascia. What name of this space?
A. * spatium interaponeuroticum suprasternale
B. previsceral space
C. retrovisceral space
D. spatium interscalenium
E. spatium anterscalenium
555. During the dissection was discovered pus in space between pharynx and deep neck muscles. What is
the name of this space?
A. * retrovisceral space
B. previsceral space
C. spatium interaponeuroticum suprasternale
D. spatium interscalenium
E. spatium anterscalenium
556. During tracheotomy arcus venosus juguli was ocasionaly cut in a 45 year old patient who was
admitted to the hospital with the larynx edema. Where is that arch located?
A. * interaponeuroticum space
B. Antercalenium space
C. pretracheale space
D. Retropharyngeal space
E. Interscalenum space
557. Patient feels a sharp pain in masticatory muscles when he compresses the molars (chewing teeth).
Which muscle develops maximal power for chewing?
A. * masseter muscle
B. musculus temporalis (posterior fibers)
C. musculus temporalis (anterior fibers)

D. medial pterygoid muscle


E. lateral pterygoid
558. Patient feels a sharp pain when he compresses the incisors (front teeth). Which muscle develops
maximal power for biting?
A. * musculus temporalis (anterior fibers)
B. musculus temporalis (posterior fibers)
C. medial pterygoid muscle
D. lateral pterygoid muscle
E. masseter
559. Patient with damaged sternocleidomastoid muscle was delivered to the hospital. Which fascia covers
this muscle?
A. * lamina superficialis fasciae colli propriae
B. fascia prevertebralis
C. fascia endocervicalis
D. lamina profunda fasciae colli propriae
E. fascia colli superficialis
560. To stop the bleeding it is necessary to press the subclavian artery. In what area is it located?
A. * spatuim interscalenum
B. trigonum omotrapezoideum
C. spatuim anterscalenum
D. spatuim retrosternocleidomastoideus
E. spatuim interaponeuroticum suprasternale
561. A 45 year old man addressed the traumatologist after the industrial trauma of the shoulder. The
functions of abduction of the arm is absent. What muscle was injured?
A. * m. supraspinatus
B. subscapular muscle
C. m. infraspinatus
D. m. teres major
E. m. teres minor
562. A child of 7 years can not lift the forearm to the shoulder. Which muscle’s active function is absent?
A. * musculus biceps brachii
B. musculus teres major
C. musculus pectoralis major
D. musculus teres minor
E. musculus deltoideus
563. A patient can not abduct the arm. Which muscle’s active function is absent?
A. * musculus deltoideus
B. musculus teres major
C. musculus pectoralis major
D. musculus biceps brachii
E. musculus teres minor
564. After falling from the height a man damaged his anterior shoulder surface. During the examination
the surgeon diagnosed the rupture of the biceps brachii. What functions of the upper extremity are
going to be disturbed that trauma?
A. * flexion of the arm and of the forearm
B. flexion of the arm and hand
C. flexion of the arm, forearm, and hand

D. flexion of the arm


E. flexion and pronation of the forearm
565. After trauma a patient cannot extend arm in the elbow joint. What muscle was injured?
A. * m. triceps brachii
B. m. brachioradialis
C. m. pronator major
D. m. subscapularis
E. m. infraspiratus
566. Extrinsic hand muscles are found in the
A. * forearm.
B. arm.
C. hand.
D. fingers.
E. Head
567. If a woman raises her arms laterally from her sides until they are level with her shoulders, she mostly
uses her
A. * deltoid muscles.
B. biceps brachii muscles.
C. latissimus dorsi muscles.
D. pectoralis major muscles.
E. all of these
568. The orchestra conductor is not able to abduct the arm. Which muscle does not operate?
A. * musculus deltoideus
B. musculus trapezius
C. musculus rhomboideus
D. musculus latissimus dorsi
E. musculus pectoralis major
569. Which muscle tendon can be injured by the bone (radius) fragment?
A. * m. biceps brachii
B. M. deltoideus
C. m. teres major
D. m. latissimus dorsi
E. m. pectoralis major
570. A patient can not flex the toes. What muscle is injured?
A. * m. flexor digitorum longus
B. m. extensor digitorum brevis
C. m. extensor hallucis longus
D. m. tibialis anterior
E. m. tibialis posterior
571. A patient can not make flexion of the toes. What muscle can be injured?
A. * m. quadratus plantae
B. m. peroneus brevis
C. m. peroneus longus
D. m. tibialis anterior
E. m. tibialis posterior
572. A patient can not to extend the foot. What muscle is injured?

A. * m. tibialis anterior
B. m. flexor digitorum longus
C. m. peroneus brevis
D. m. peroneus longus
E. m. tibialis posterior
573. A patient can not to lift lateral margin of the foot. What muscle is injured?
A. * m. peroneus brevis
B. m. extensor digitorum longus
C. m. triceps surae
D. m. tibialis posterior
E. m. tibialis anterior
574. A patient can not to lift medial margin of the foot. What muscle is injured?
A. * m. tibialis anterior
B. m. tibialis posterior
C. m. flexor digitorum longus
D. m. peroneus brevis
E. m. peroneus longus
575. A patient can not to make flexion of the foot. What muscle is injured?
A. * m. tibialis posterior
B. m. extensor digitorum longus
C. m. extensor digitorum brevis
D. m. extensor hallucis longus
E. m. tibialis anterior
576. After the trauma a patient cannot stand on tiptoes. Which muscle is damaged?
A. * m. triceps surae
B. m. fibularis longus
C. m. extensor hallucis longus
D. m. tibialis anterior
E. m. tibialis posterior
577. Doing his morning exercise a fifteen – year boy felt a sharp pain in the zone of a hip joint when the
limb is turning inside (pronation). The doctor discovered the injury of the tendon of one of the
muscles which can rotate medially leg. Which muscle is this?
A. * m. gluteus medius
B. m. piriformis
C. m. oburatorius internus
D. m. obturatorius externus
E. m. quadratus femoris
578. Motorcycle wheel hits on the leg resulted in the rupture of the deep blood vessels accompanied by a
large hematoma. Which big canal with vessels and nerves located under triceps surae muscle?
A. * cruropopliteal canal
B. superior musculoperoneal canal
C. inferior musculoperoneal canal
D. adductorial canal
E. E obturator canal
579. Motorcycle wheel hits on the thigh resulted in the rupture of the blood vessels. Which canal with
vessels and nerves located there?
A. * adductorial canal

B. superior musculoperoneal canal


C. inferior musculoperoneal canal
D. cruropopliteal canal
E. obturator canal
580. Patient felt a sharp pain in the zone of a hip joint when he flexes the thigh. The doctor discovered the
injury of the tendon of the muscle. Which muscle is this?
A. * m. rectus femoris
B. m. gluteus medius
C. m. biceps femoris
D. m. oburatorius internus
E. m. obturatorius externus
581. Patient felt a sharp pain in the zone of a hip joint when he extends the knee. The doctor discovered
the injury of the tendon of the muscle. Which muscle is this?
A. * m. quadriceps femoris
B. m. biceps femoris
C. m. rectus femoris
D. m. oburatorius internus
E. m. obturatorius externus
582. Patient with a wound in the field of the foot sole was delivered to the traumatologist. The lifting of
lateral foot edge is limited. Which muscle function is broken?
A. * m. peroneus longus
B. m. extensor digitorum longus
C. m. triceps surae
D. m. soleus
E. m. tibialis anterior
583. The medial group of thigh muscles is involved primarily in
A. * adduction of the thigh.
B. abduction of the thigh.
C. extension of the thigh.
D. flexion of the thigh.
E. flexion of the leg.
584. The patient has to make flexion of hip joint to discover pain symptom in the pelvic cavity. Which
muscle is contracted during this movement?
A. * m. iliopsoas
B. m. lumbricales
C. m. vastus lateralis
D. m. vastus lateralis
E. m. transversus abdominis
585. This muscle is used to cross the legs; it flexes the leg, and flexes and laterally rotates the thigh.
A. * sartorius
B. adductor longus
C. gracilis
D. gluteus maximus
E. vastus lateralis
586. Two thigh muscles that are antagonists are the
A. * rectus femoris and semimembranosus.
B. rectus femoris and vastus lateralis.

C. vastus medialis and vastus intermedius.


D. adductor longus and vastus lateralis.
E. gluteus maximus and semitendinosus.
587. Which of these muscles is used as an injection site?
A. * gluteus medius
B. deltoid
C. gluteus medius
D. vastus lateralis
E. gracilis
588. A 19 year-old patient was diagnosed with appendicitis and was hospitalized. The surgical operation
on ablating appendix vermiformis is to be performed. What artery must be fixed to stop bleeding
during the surgical operation?
A. * The ileocolic artery
B. The colica dextra
C. The colica media
D. The colica sinistra
E. The iliac
589. A 50 y.o. patient was admitted to the hospital with complaints about pain behind his breastbone,
asphyxia during physical activity. Angiography revealed pathological changes in the posterior
interventricular branch of the right coronary artery. What heart parts are affected?
A. * Posterior wall of the right and left ventricles
B. Left atrium
C. Anterior wall of the right and left ventricles
D. Right atrium
E. Right atrioventricular valve
590. A 54-year-old man was admitted to the hospital with complaints of pain in the right subcostal region,
vomiting with blood. Objectively: enlarged liver, varicose veins in the stomach and esophagus.
Disfunction of what vessel is likely to be?
A. * Vena porta
B. Aorta abdominalis
C. Vena hepatica
D. Vena cava superior
E. Vena cava inferior
591. A 70 year old female patient was diagnosed with fracture of neck of left femur accompanied by
disruption of ligamentum capitis femoris. The branch of the following artery is damaged:
A. * Obturator
B. Femoral
C. External iliac
D. Inferior gluteal
E. Internal pudendal
592. A man suffering from osteochondrosis got acute pain in the abdominal muscles (lateral and anterior).
During objective examination a physician diagnosticated increased pain sensitivity of skin in the
hypogastric region. This pain might be caused by affection of the following nerve:
A. * Iliohypogastric
B. Sciatic
C. Obturator
D. Femoral
E. Genitofemoral

593. A patient complains about edemata of legs, skin cyanosis, and small ulcers on one side of the lateral
condyle. Examination revealed a swelling, enlarged veins, formation of nodes. The pathological
process has started in the following vein:
A. * V. saphena parva
B. V. saphena magna
C. V. femoralis
D. V. profunda femoris
E. V. iliaca externa
594. A patient has a malignisation of thoracic part of esophagus. What lymphatic nodes are regional for
this organ?
A. * Anulus lymphaticus cardiae
B. Nodi lymphatici paratrachealis
C. Nodi lymphatici prevertebralis
D. Nodi lymphatici pericardiales laterales
E. Nodi lymphatici mediastinales posteriores
595. A patient has an exudative pleurisy. At what level should the pleural puncture along the posterior
axillary line be taken?
A. * ІХ intercostal space
B. VІІІ intercostal space
C. VІІ intercostal space
D. ХІ intercostal space
E. VІ intercostal space
596. A patient has lost skin sensitivity in the region of the medial surface of his shoulder. This is the result
of dysfunction of the following nerve:
A. * Medial brachial cutaneous nerve
B. Medial antebrachial cutaneous nerve
C. Radial nerve
D. Ulnar nerve
E. Axillary nerve
597. A patient has lost skin sensitivity in the region of the medial surface of his shoulder. This is the result
of dysfunction of the following nerve:
A. * Medial brachial cutaneous nerve
B. Medial antebrachial cutaneous nerve
C. Radial nerve
D. Ulnar nerve
E. Axillary nerve
598. A patient has tissue ischemia below the knee joint accompanied with intermittent claudication. What
artery occlusion should be suspected?
A. * Popliteal artery
B. Peroneal artery
C. Posterior tibial artery
D. Anterior tibial artery
E. Proximal part of femoral artery
599. A patient with neuritis of femoral nerve has disturbed flexion of thigh as well as disturbed crus
extension in the knee joint. What muscle's function is disturbed?
A. * Quadriceps muscle of thigh
B. Biceps muscle of thigh
C. Triceps muscle of thigh

D. Semitendinous muscle
E. Semimembranous muscle
600. A patient can not extend knee joint, there is no knee-jerk reflex, skin sensitivity of the anterior
femoral surface is disturbed. What nerve structures are damaged?
A. * Femoral nerve
B. Superior gluteal nerve
C. Big fibular nerve
D. Obturator nerve
E. Inferior gluteal nerve
601. A woman consulted a doctor about swelling and tenderness of the lower extremity, swollen veins and
nodes on the medial surface of thigh. Which vein was affected?
A. * Great saphenous
B. Lesser saphenous
C. Femoral
D. Popliteal
E. Tibial
602. A woman suffering from osteochondrosis felt acute pain in her humeral articulation that became
stronger when she abducted her shoulder. These symptoms might be caused by damage of the
following nerve:
A. * Axillary nerve
B. Subscapular nerve
C. Dorsal scapular nerve
D. Subclavicular nerve
E. Throracodorsal nerve
603. A woman underwent an operation of extrauterine (tubal) pregnancy. In course of the operation the
surgeon should ligate the branches of the following arteries:
A. * Uterine and ovarian
B. Superior cystic and ovarian
C. Inferior cystic and ovarian
D. Uterine and superior cystic
E. Uterine and inferior cystic
604. After a road accident a driver was delivered to the hospital with an injury of the medial epicondyle of
humerus. What nerve might be damaged in this case?
A. * n. ulnaris
B. n. radialis
C. n. axillaris
D. n. muscolocutaneus
E. n. medianus
605. After resection of the middle third of femoral artery obliterated by a thromb the lower extremity is
supplied with blood due to the surgical bypass. Name an artery that plays the main role in
reestablishment of blood flow:
A. * Deep femoral artery
B. Superficial circumflex artery of hip bone
C. Descending genicular artery
D. Superficial epigastric artery
E. Deep external pudendal artery

606. After trauma a 44-year-old patient had a rupture of left palm muscle tendons and of the surface of
blood vessels. After operation and removal of the most part of the necrotically changed muscle tissue
the bloodstream was normalized. What vessels have helped with restoration of bloodstream?
A. * Arcus palmaris profundus
B. Arcus palmaris superficialis
C. Aa. digitales palmares communes
D. Aa. metacarpeae palmares
E. Aa. perforantes
607. During the fetal period of the development in the vascular system of the fetus large arterial (Botalli)
duct is functioning which converts intolig. arteriosum after birth. What anatomical formations does
this duct connect between each other?
A. * Pulmonary trunk and aorta
B. Right and left auricle
C. Aorta and inferior vena cava
D. Pulmonary trunk and superior vena cava
E. Aorta and superior vena cava
608. Examination of a 2-year-old child revealed physical developmental lag, the child often has
pneumonias. The child was diagnosed with nonclosure of ductus arteriosus. Haemodynamics disorder
was caused by the intercommunication of the following vessels:
A. * Aorta and pulmonary trunk
B. Pulmonary trunk and pulmonary veins
C. Superior cava and aorta
D. Superior cava and pulmonary trunk
E. Aorta and pulmonary veins
609. On examination of a road accident victim a doctor revealed left clavicle fracture and disturbed blood
circulation in an extremity (no pulsing of radial artery). What cause of blood circulation disturbance
is the most probable?
A. * Compression of subclavian artery
B. Compression of axillary artery
C. Compression of subclavian vein
D. Compression of vertebral artery
E. Compression of axillary vein
610. The process of heart transplantation determined the viability of myocardial cells. The determination
of what myocardium parameter is the most important?
A. * Rest potential of cardiomyocytes
B. Heart temperature
C. Concentration of oxygen in heart vessels
D. Concentration of calcium-ions in myofibrils
E. Concentration of Ca-ions in heart vessels
611. Victim has elbow joint trauma with avulsion of medial epicondyle of humerus. What nerve can be
damaged in this trauma?
A. * Ulnar
B. Radial
C. Musculocutaneous nerve
D. Cardiac cutaneous nerve
E. Medial cutaneous nerve of forearm
612. Where should the cathetor for evacuation of the lymph from the thoracic lymph duct be inserted?
A. * To the left venous angle

B. To the right venous corner


C. To the superior vena cava
D. To the inferior vena cava
E. To the left inguinal vein
613. While palpating mammary gland of a patient a doctor revealed an induration in form of a node in the
inferior medial quadrant. Metastases may extend to the following lymph nodes:
A. * Parasternal
B. Posterior mediastinal
C. Profound lateral cervical
D. Bronchopulmonary
E. Superior diaphragmal
614. While performing an operation in the area of armpit crease a surgeon has to define an arterial vessel
surrounded by fascicles of brachial plexus. What artery is it?
A. * A. axillaris
B. A. vertebralis
C. A. transversa colli
D. A. profunda brachii
E. A. subscapularis
615. While preparing a patient to the operation the heart chambers' pressure was measured. In one of them
the pressure changed during one heart cycle from 0 to 120 mm Hg. What chamber of heart was it?
A. * Left ventricle
B. Right ventricle
C. Right atrium
D. Left atrium
E. Atrium
616. A 25 year old patient was examined by a medical board. Examination revealed pathology of chest.
Transverse dimensions were to small and the sternum was strongly protruding. What chest type is it?
A. * Keeled chest
B. Funnel chest
C. Flat chest
D. Cylindrical chest
E. Barrel chest
617. A 35 year old man with a trauma of his left hand was admitted to the traumatology department.
Objectively: cut wound of palmar surface of left hand; middle phalanges of II–V fingers don't bend.
What muscles are damaged?
A. * Superficial digitorum flexor
B. Profoundus finger flexor
C. Lumbrical muscles
D. Palmar interosseous muscles
E. Dorsal interosseous muscles
618. A 38-year-old patient came to a traumatology center and complained about an injury of his right
hand. Objectively: the patient has a cut wound in the region of the thenar eminence on the right hand;
distal phalanx of the I finger cannot be flexed. What muscle was injured?
A. * flexor pollicis longus
B. flexor pollicis brevis
C. abductor pollicis brevis
D. Opponens pollicis
E. Abductor pollicis

619. A 45-year-old man applied to the trauma unit because of domestic shoulder trauma. Objectively:
flexibility, reduction and pronation functions of the shoulder are absent. What muscle was injured?
A. * Teres major muscle
B. Subscapular muscle
C. Teres minor muscle
D. Infraspinous muscle
E. Supraspinous muscle
620. A 45-year-old man fell on the right knee and felt the acute pain in the joint. On examination: severe
edema on the anterior surface of the knee joint. Crunching sounds are heard while moving the joint.
Which bone is destroyed?
A. * Knee-cap
B. Neck of the femur
C. Left epicondyle of the femur
D. Right epicondyle of the femur
E. Head of the femur
621. A 5 year old child suffers from the neck deformity. Clinical examination revealed such symptoms:
apparent flexion of head to the left, his face is turned right, passive movements of the head to the
right are restricted. What muscle's development was disturbed in this case?
A. * Sternocleidomastoid
B. Trapezius
C. Splenius muscle of head
D. Sternosublingual
E. Long muscle of head
622. A 5 year old child was admitted to the hospital with suppurative inflammation of middle ear
(tympanitis). It began with the inflammation of nasopharynx. What canal of temporal bone did the
infection get into tympanic cavity through?
A. * Musculotubal canal
B. Small canal of chorda tympani
C. Caroticotympanic foramina
D. Small tympanic canal
E. Carotid canal
623. A 55-year-old patien was hospitalized in result of the trauma of the medial group of femoral muscles.
What kind of movements is the patient unable to do?
A. * Adduction of femur
B. Abduction of femur
C. Flexion of femur
D. Extension of femur
E. Suppination of femur
624. A 5-year-old child was admitted to the otorhinolaryngological department with diagnosis -
suppurative inflammation of the middle ear. Disease started from the inflammation of the
nasopharynx. Through the what canal of the temporal bone did the infection get into the tympanic
cavity?
A. * Musculortubal canal
B. Tympanic Canaliculus tympanicus
C. Carotid canal
D. Canaliculus chordal tympani
E. Canaliculi caroticotympanici

625. A 69 year old patient has got an abscess of frontal lobe as a result of purulent infection in nasal
cavity. What anatomical formation did the infection penetrate through?
A. * Foraminae cribrosae
B. Foramen ovale
C. Foramen ethmoidalae posterior
D. Foramen sphenopalatinum
E. Foramen rotundum
626. A 6-year-old child fell on the cutting object and traumatized soft tissues between tibia and fibula.
What kind of bone connection was injured?
A. * Membrane
B. Suture
C. Ligament
D. Fontanel
E. Gomphosis
627. A 7-year-old child can't abduct the shoulder, raise it to the horizontal level. He can raise the hand to
the face only with dorsal side with abduction of the shoulder (with help of supraspinous muscle ) -
"bugler" arm. Active function of what muscle is absent?
A. * Deltoid
B. Infraspinous
C. Pectoral major
D. Teres minor
E. Teres major
628. A boxer who got a punch in the region of temporomandibular joint has a traumatic dislocation of
mandible. Displacement of what articular surfaces will overstep the limits of physiological norm?
A. * Head of mandible and mandibular fossa
B. Coronoid process and pterygoid fossa
C. Coronoid process and submandibular fossa
D. Head of mandible and submandibular fossa
E. Neck of mandible and submandibular fossa
629. A man with an injury of the dorsal area of his neck was admitted to the resuscitation department.
What muscle occupies this area?
A. * M.trapezius
B. M.sternocleidomastoideus
C. M.latissimus dorsi
D. M.rhomboideus minor
E. M.scalenus anterior
630. A patient arrived to the oral surgery department with dislocation of temporomandibular joint and
injury of its main ligament. Name this ligament:
A. * Lateral
B. Mandibular
C. Styloid-mandibular
D. Pterygoid-mandibular
E. Medial
631. A patient complained about being unable to adduct and abduct fingers in the metacarpophalangeal
articulations towards and away from the 3rd finger. Which muscles' function is impaired?
A. * Interosseous muscles
B. Lumbrical muscles
C. Breviflexors of fingers

D. Long flexors of fingers


E. Extensors
632. A patient displays abnormal retrodeviation of his lower jaw as a result of trauma in the region of
mandibular coronal process. What muscle is most likely to be damaged?
A. * M.temporalis
B. M.masseter
C. M.pterygoideus lateralis
D. M.pterygoideus medialis
E. M.levator anguli oris
633. A patient has a deep cut wound on the posterior surface of his shoulder in its middle third. What
muscle might be injured?
A. * Triceps brachii muscle
B. Biceps brachii muscle
C. Anconeus muscle
D. Brachial muscle
E. Coracobrachial muscle
634. A patient has an inflammation in the pterygopalatine fossa. The infection has spread into the nasal
cavity. Which anatomical structure has the infection spread through?
A. * Foramen sphenopalatinum
B. Foramen rotundum
C. Canalis palatinus major
D. Canalis palatinus minor
E. Canalis ptherygoideus
635. A patient has been diagnosed with a compressive fracture of a lumbar vertebra. As a result he has a
considerable increase in curvature of the lumbar lordosis. Which ligament damage can induce
such changes in the spine curvature?
A. * Anterior longitudinal ligament
B. Posterior longitudinal ligament
C. Iliolumbar ligament
D. Interspinous ligament
E. Yellow ligament
636. A patient has difficulties with hand movement. Examination revealed inflammation of common
synovial sheath of flexor muscles. It is known from the patient's anamnesis that he got a stab wound
of finger a week ago. Which finger was most probably damaged?
A. * Digitus minimus
B. Pollex
C. Digitus medius
D. Index
E. Digitus anularis
637. A patient presents with dysfunction of shin muscles. He cannot raise his body by standing on tiptoe.
Which muscle is affected?
A. * M. triceps surae
B. M. tibialis posterior
C. M. extensor digitorum longus
D. M. flexor digitorum longus
E. M. tibialis anterior
638. A patient was admitted to the surgical department with inguinal hernia. During the operation the
surgeon performs plastic surgery on posterior wall of inguinal canal. What structure forms this wall?

A. * Transverse fascia
B. Aponeurosis of abdominal external oblique muscle
C. Inguinal ligament
D. Loose inferior edge of transverse abdominal muscle
E. Peritoneum
639. A patient with a knife wound in the left lumbal part was delivered to the emergency hospital. In
course of operation a surgeon found that internal organs were not damaged but the knife injured one
of muscles of renal pelvis. What muscle is it?
A. * Psoas major muscle
B. Iliac muscle
C. Erector muscle of spine
D. Abdominal internal oblique muscle
E. Abdominal external oblique muscle
640. A patient with neuritis of femoral nerve has disturbed flexion of thigh as well as disturbed crus
extension in the knee joint. What muscle's function is disturbed?
A. * Quadriceps femoris muscle
B. Biceps femoris muscle
C. Triceps femoris muscle
D. Semitendinous muscle
E. Semimembranous muscle
641. A victim of a road accident has an abruption of a part of mandibular angle, displacement of fragment
backwards and upwards. What ligament is responsible for this displacement?
A. * Styloid-mandibular
B. Intraarticular
C. Lateral
D. Sphenoid-mandibular
E. Pterygoid-mandibular
642. After a trauma a patient lost ability of elbow extension. This might have been caused by dysfunction
of the following main muscle:
A. * m. triceps brachii
B. m. subscapularis
C. m. teres major
D. m. infraspinatus
E. m. levator scapulae
643. An eye trauma caused soft tissues infection of orbit. Through what anatomical formation can the
infection penetrate into the middle cranial fossa?
A. * Through the superior orbital fissure
B. Through the anterior ethmoidal foramen
C. Through the posterior ethmoidal foramen
D. Through the inferior orbital fissure
E. Through the zygomatic orbital foramen
644. An old woman was hospitalized with acute pain, edema in the right hip joint; the movements in the
joint are limited. Which bone or part of it was broken?
A. * The neck of the femur
B. The body of the femur
C. Condyle of the femur
D. Pubic bone

E. Ischial bone
645. As a result of an accident a patient has intense painfullness and edema of the anterior crus surface;
dorsal flexion of foot is hindered. Function of which crus muscle is most likely to be disturbed?
A. * M.tibialis anterior
B. M.flexor digitorum longus
C. M.flexor hallucis longus
D. M.peroneus longus
E. M.peroneus brevis
646. Chronic rhinitis was complicated by inflammation of frontal sinus. What nasal meatus did the
infection get into this sinus through?
A. * Median
B. Inferior
C. Common
D. Superior
E. Nasopharyngeal
647. During manipulations aimed at treatment of mandible dislocation a physician should pay particular
attention to a muscle that pulls a capsule and interarticular disc of temporomandibular articulation
exteriorly. What muscle is it?
A. * M. pterygoideus lateralis
B. M. masseter
C. M. pterygoideus medialis
D. M. temporalis
E. M. mylohyoideus
648. During the operation on the hip joint of a 5-year-old child her ligament was damaged which caused
bleeding. What ligament was damaged?
A. * Capitis femoris
B. Perpendicular of the acetabule
C. Iliofemoral
D. Pubofemoral
E. Ischiofemoral
649. Examination of a 6-month-old child revealed a delay in closure of the occipital fontanelle. When
should it normally close?
A. * Until 3 months
B. Before the child is born
C. Until 6 months
D. Until the end of the first year of life
E. Until the end of the second year of life
650. Examination of a patient revealed an abscess of pterygopalatine fossa. Where can the infection spread
to unless the disease is managed in time?
A. * To the orbit
B. To the interpterygoid space
C. To the frontal sinus
D. To the temporal space
651. It is necessary to take the cerebrospinal fluid from a patient with suspected inflammation of brain
tunics. Diagnostic puncture was performed between the arches of the lumbar vertebras. During the
puncture the needle went through the following ligament:
A. * Yellow (flava)
B. Iliolumbar

C. Anterior longitudinal
D. Posterior longitudinal
E. Intertransverse
652. The operative dentistry department admitted a newborn girl who choked during sucking.
Examination revealed cleft palate arising from non-union of the middle frontal process and maxillary
process of the I-st branchial arch. The cleft was located in the palate between:
A. * Os incisivum et processus palatinus maxillae
B. Processus palatinus maxillae dextrae et sinistrae
C. Lamina horizontalis os palatinum dextrum et sinistrum
D. Processus palatinus maxillae et lamina horizontalis os palatinum
E. In the region of canalis incisivus
653. Three separate bones connected with cartilage in the area of pelvis cavity are noticed on the X-ray of
the pelvis. What are these bones?
A. * Iliac, pubic, sciatic
B. Pubic, sciatic, femoral
C. Sciatic, femoral, sacral
D. Iliac, sacral, coccyx
E. Sacral, pubic, coccyx
654. Trauma of occipital region of head resulted in crack fracture in the region of transverse sinus. What
part of occipital bone is damaged?
A. * Squama
B. Left lateral
C. Right lateral
D. Proximal
E. Condyle
655. Under development of which parts of facial skeleton in the embryonal period is the reason for such a
malformation as cleft palate?
A. * Palatine processes
B. Frontal processes
C. Frontal and maxillary processes
D. Mandibular processes
E. Mandibular and palatine processes
656. Usually the intravenous injection is done into median cubital vein because it is slightly movable due
to fixation by the soft tissues. What does it fix in the cubital fossa?
A. * Aponeurosis of biceps muscle
B. Tendon of the triceps muscle
C. Brachial muscle
D. Brachioradial muscle
E. Anconeus muscle
657. When processing a molar tooth with a dental cutter the dentist has by accident deeply wounded the
patient's cheek and damaged not only the mucosa but also a muscle. Which muscle was hurt?
A. * Buccinator muscle
B. Zygomatic major muscle
C. Masetter muscle
D. Orbicularis oris muscle
E. Mylohyoid muscle
658. X-ray examination revealed an accumulation of suppuration in maxillary sinus. Into what nasal
meatus excretes the suppuration?

A. * Median nasal
B. Nasopharyngeal
C. Inferior nasal
D. Superior nasal
E. Common nasal
659. Young man felt sharp pain in the back during active tightening on the horizontal bar. Objectively:
pain while moving of upper extremity, limited pronation and adduction functions. Sprain of what
muscle is presented?
A. * М. latissimus dorsi
B. М. levator scapulae
C. М. romboideus major
D. М. trapezius
E. М. subscapularis
660. ?The patient has compressive fracture at the beginning of spinal cord .At what vertebra is it?
A. * C1
B. L4
C. C3
D. T11
E. T7
661. Between what vertebras may a needle be inserted during a spinal tap?
A. * L3-L4
B. L4-L5
C. L2-L3
D. L1-L2
E. T12-L1
662. During general anesthesia doctor blocking the basic functional unit of the nervous system it is…
A. * Neuron
B. Glial cell
C. White mater and gray mater
D. Portion of spinal cord with pair of spinal nerves
E. Anterior and posterior horns
663. During general anesthesia doctor blocking the brain it is the part of…
A. * Central nervous system
B. Somatic nervous system
C. Autonomic nervous system
D. Portion of spinal cord with pair of spinal nerves
E. Splanchnic nervous system
664. During spinal anesthesia doctor blocking segment of spinal cord it is…
A. * Portion of spinal cord with pair of spinal nerves
B. Portion of spinal cord with membranes
C. White mater and gray mater
D. Portion of spinal cord between upper and lower spinal roots
E. Anterior and posterior horns
665. On the CT picture doctor can indicate position of the end of spinal cord on adults in the…
A. * L2 vertebrae
B. S1 vertebrae
C. Th 12 vertebrae

D. Th11 vertebrae
E. L1 vertebrae
666. On the CT picture doctor can indicate position of the end of spinal cord on child in the…
A. * L3 vertebrae
B. Th 12 vertebrae
C. L1 vertebrae
D. S1 vertebrae
E. L2 vertebrae
667. On the CT picture doctor can indicate position of the terminal ventricle in the…
A. * Spinal cord
B. Brain steam
C. Midbrain
D. Diencephalon
E. Hind brain
668. The child 5 years old diagnosed meningitis. At what level punctuate must be done?
A. * L3-L4
B. L2-L3
C. L4-L5
D. L1-L2
E. T12-L1
669. The doctor neuropathology examine knee reflex the sensory nerve fibers carry information…
A. * Toward the CNS
B. Away from the ANS
C. Away from the CNS to the ANS
D. Away from the visceral nervous system
E. Away from the CNS
670. The doctor want punctuate spinal canal. At what level it must be done?
A. * L2
B. T4
C. C5
D. T12
E. T8
671. The patient cannot detect the amount of stretch or tension in skeletal muscles, what receptors not
functioned?
A. * Proprioceptors
B. Pain receptors
C. Heat receptors
D. Cutaneous receptors
E. Away from the CNS
672. The patient has fever and weakness in foots, doctor suspect meningitis and examine…
A. * Cerebral spinal fluid
B. The subarachnoid space
C. Extra cerebral spinal fluid
D. Fat
E. Blood
673. The patient has not temperature sensitivity of the legs skin. What is the minimum number of neurons
is blocked?
A. * -2
B. -4
C. -5
D. -1
E. -3
674. What functions of the central nervous system examine the doctor neuropathology?
A. * All of the above
B. Sending out messages to glands and organs
C. Sending out messages to muscles
D. None of the above
E. Receiving, processing and interpreting incoming information
675. You see CT picture indicate position of filum terminale in the…
A. * L2-Co2
B. C1-S1
C. C1-L2
D. C2-L2
E. L1-Co1
676. You see CT picture indicate position of lumbosacral enlargement in the…
A. * Th 12 vertebrae
B. L1 vertebrae
C. S1 vertebrae
D. L2 vertebrae
E. Th11 vertebrae
677. You see CT picture what you can indicate in central canal of vertebrae?
A. * Spinal cord
B. Brain steam
C. Midbrain
D. Diencephalon
E. Hind brain
678. After auto accident to the neurological department of hospital delivered the man. Found traumatic
brain injury .Suspected lesion of the hindbrain. Which parts of the brain can be damaged?
A. * pons
B. none of the above
C. hypophysis
D. corpus callosum
E. all correct answers
679. After auto accident to the neurological department of hospital delivered the man. Found traumatic
brain injury .Suspected lesion of the hindbrain. Which parts of the brain can be damaged?
A. * vermis of cerebellum
B. frontal lobe
C. corpus callosum
D. hypophysis
E. temporal lobe
680. After auto accident to the neurological department of hospital delivered the man. Found traumatic
brain injury .Suspected lesion of the hindbrain. Which parts of the brain can be damaged?
A. * decussation of pyramids
B. hypophysis
C. frontal lobe
D. corpus callosum
E. temporal lobe
681. After traumatic brain injury of occipital area of the head to the neurological department of hospital
delivered the man. Suspected lesion of the medulla oblongata. What disorders of the central nervous
system can be detected?
A. * cardiovascular disorders
B. speech defect
C. auditory loss
D. a loss of coordination and balance
E. distorted pain perception
682. After traumatic brain injury of occipital area of the head to the neurological department of hospital
delivered the man. Suspected lesion of the medulla oblongata. What disorders of the central nervous
system can be detected?
A. * cardiac and vasomotor disorders
B. auditory loss
C. speech defect
D. a loss of coordination and balance
E. distorted pain perception
683. After traumatic brain injury of occipital area of the head to the neurological department of hospital
delivered the man. Suspected lesion of the medulla oblongata. What disorders of the central nervous
system can be detected?
A. * respiratory rhythmicity disorders
B. auditory loss
C. a loss of coordination and balance
D. speech defect
E. distorted pain perception
684. After traumatic brain injury of occipital area of the head to the neurological department of hospital
delivered the man. Suspected lesion of the cerebellum. What disorders of the central nervous system
can be detected?
A. * a loss of coordination
B. auditory loss
C. speech defect
D. respiratory rhythmicity disorders
E. distorted pain perception
685. After traumatic brain injury of occipital area of the head to the neurological department of hospital
delivered the man. Suspected lesion of the cerebellum. What disorders of the central nervous system
can be detected?
A. * a loss of balance
B. speech defect
C. auditory loss
D. respiratory rhythmicity disorders
E. distorted pain perception
686. After traumatic brain injury patient complains of loss of visual motor response. What part of the brain
may be damaged?
A. * corpora quadrigemina
B. Corpus callosum.
C. cerebellum

D. none of the above


E. medulla oblongata
687. After traumatic brain injury to the neurological department of hospital delivered the man. Suspected
lesion of the hindbrain. Which parts of the brain can be damaged?
A. * all correct answers
B. cerebellar hemispheres
C. vermis of cerebellum
D. decussation of pyramids
E. pons
688. After traumatic brain injury to the neurological department of hospital delivered the man. Suspected
lesion of the hindbrain. Which parts of the brain can be damaged?
A. * cerebellar hemispheres
B. hypophysis
C. none of the above
D. corpus callosum
E. all correct answers
689. During operation neurosurgeon remove some parts of the pons of the brain. What structures may be
destroyed?
A. * All of the above
B. Processes of visual and auditory information
C. Connections of the cerebellum to the brain stem
D. None of the above
E. Processing centers for sensory information
690. During the examination of the patient of neurological department found violations of hypoglossal
nerve. Indicate the level of location of the nucleus of hypoglossal nerve?
A. * Medulla oblongata
B. The upper segments of the spinal cord;
C. The lower part of the pons;
D. Legs brain;
E. Top of the pons.
691. During the MRI examination revealed pathological formation in the area of the lower triangle of
rhomboid fossa. What nucleus of cranial nerves can be damaged?
A. * from IX to XII pair
B. from V to VIII pair
C. from V to XII pair
D. from I to IV pair
E. from I to IV pair
692. During the MRI examination revealed pathological formation in the area of the upper triangle of
rhomboid fossa. What nucleus of cranial nerves can be damaged?
A. * from V to VIII pair
B. from IX to XII pair
C. from V to XII pair
D. from I to IV pair
E. from I to IV pair
693. During the MRI examination revealed pathological formation in the area of the lateral triangle of
rhomboid fossa. What can be damaged?
A. * Vestibular field
B. Triangle pneumogastric

C. Triangle hyoid nerve


D. Caudal pole
E. Cranial pole
694. During the MRI examination revealed pathological formation in the area of superior colliculus and
brachium of superior colliculus. The function of what centers can be damaged?
A. * subcortical visual centers
B. olfactory centers
C. subcortical motor centers
D. subcortical auditory centers
E. centers of stereognozy
695. During the MRI examination revealed pathological formation in the area of trapezoid body. Which
parts of the brain can be damaged?
A. * Pons
B. Rhomboid fossa
C. Cerebellum
D. Isthmus rhomboencephalon
E. Medulla oblongata
696. During the MRI examination revealed pathological formation in the area of interpeduncular fossa.
Which nerve can be damaged?
A. * 3d cranial nerve
B. 2nd cranial nerve
C. 4th cranial nerve
D. 5th cranial nerve
E. 1st cranial nerve
697. Following an auto accident, a man with an obvious head injury was observed stumbling about the
scene. An inability to walk properly and loss of balance were quite obvious. What brain area was
involved?
A. * cerebellum
B. occipital lobe of cerebrum
C. temporal lobe of cerebrum
D. medulla oblongata
E. corpus callosum
698. Following an auto accident, a man with an obvious head injury was observed stumbling about the
scene. An inability to walk properly and loss of balance were qute obvious. What brain area was
involved?
A. * Cerebellum
B. Occipital lobe of cerebrum
C. Corpus callosum
D. Medulla oblongata
E. Temporal lobe of cerebrum
699. In patient heart rhythm irregularities. What part of the brain may be damaged?
A. * medulla oblongata
B. cerebrum.
C. superior coliculi.
D. none of the above
E. cerebellum
700. The CT picture indicates hemorrhage in the fourth ventricle. Where it locate?

A. * Rhomboencephalon
B. Mesencephalon
C. Diencephalon
D. Medulla oblongata
E. Isthmus rhomboencephalon
701. The CT picture indicates hemorrhage in the pyramids region. Where it locate?
A. * Isthmus rhomboencephalon
B. Rhomboid fossa
C. Medulla oblongata
D. Cerebellum
E. Pons
702. The CT picture show small hemorrhage to the reflex center of the medulla. What center not involve
to the process?
A. Digestive centers
B. Cardiac and vasomotor centers
C. * Respiratory rhythmicity center
D. Cardiovascular center
E. None of the above
703. The CT picture shows that patient has closed connection between 3-d and 4-th ventricles. What
structure does not function correctly?
A. * Cerebral aqueduct
B. Lateral ventricle
C. Amiculum olive
D. Corpus callosum
E. Tractus pyramidalis
704. The doctor diagnosed pathology in the cerebellum. What functions will be damage?
A. * Balance
B. Visual activity
C. Speech
D. Binocular vision
E. Emotional behavior
705. The doctor on CT picture diagnosed pathology in the medulla oblongata. What control functions
except heart rate, blood pressure will be damage?
A. * Rate and depth of breathing
B. Balance
C. B and D
D. Body temperature
E. Emotions
706. The patient has hemorrhage to the corpora quadrigemina ,it would most like result in…
A. * Loss of visual motor response
B. Speech defect
C. Auditory loss
D. A loss of coordination and balance
E. Distorted pain perception
707. The patient has hemorrhage to the region of the brain that regulates heart rate I t is located…
A. * Medulla oblongata
B. Cerebrum

C. Cerebullum
D. Superior coliculi
E. Non of the above
708. The patient has less quantity of cerebrospinal fluid. What structure is response for it production?
A. * Chroid plexus
B. Dura mater
C. Pia mater
D. Dural sinus
E. Arachnoid granulation
709. The patient has less quantity of cerebrospinal fluid. What functional process will be break?
A. * All of the above
B. Transports nutrients, chemical messengers and waste products
C. Supports of the brain
D. None of the above
E. Cushions neural structures
710. The patient has partial loss of control wake. Which area of the brain is damaged?
A. * Reticular activating system (RAS)
B. Midbrain
C. All of the above
D. Cerebellum
E. Non of the above
711. The patient in ICU has problems with respiratory rhythm. Where can be damage of the brain locate?
A. * Mesencephalon
B. Rhomboencephalon
C. Diencephalon
D. Medulla oblongata
E. Isthmus rhomboencephali
712. The surgeon during operation cut nerve in the interpeduncular fossa. Which number of cranial nerve
it was?
A. * 3-d cranial nerve
B. 2-nd cranial nerve
C. 4-th cranial nerve
D. 5-th cranial nerve
E. 1-st cranial nerve
713. To the emergency department brought the patients after auto accident during the examination in
patient found heart rhythm irregularities. What part of the brain may be damaged?
A. * medulla oblongata
B. cerebellum
C. cerebrum.
D. superior coliculi.
E. none of the above
714. After injury of head MRI picture indicate, that patient has some adhesions in the middle frontal
gyrus. What center is located here?
A. * Writing center
B. Hearing area
C. Speech area
D. Visual cortex

E. Sensory area
715. After injury of the head, subdural hemorrhage and compression of the occipital lobe was detected by
CT investigation. With what this region is associated?
A. * Are concerned with vision
B. Are the most posterior part of the cerebellum
C. Help regulate gross movements performed unconsciously
D. Are located within the white matter of the cerebrum
E. Are the part of peripheral nervous system
716. After scanning, tumor was detected at the Brocar’s Area in the brain of patient. Abnormalities of
which function may be observe?
A. * Speak
B. Interpret sounds
C. Maintain a state of wakefulness
D. Remember
E. Play a musical instrument
717. At the clinical researches scientists explored, that anterior pituitary gland receives hypothalamic
relising factors via…
A. * System of vessels in the infundibulum
B. Transport along nerve axons to the adenohypophysis
C. The systemic arterial system
D. Transport through the lymphatic system
E. Direct diffusion through the interstitial spaces
718. By clinical researches it was explored, that the part of the brain concerned with water balance,
appetite and regulating body temperature is the…
A. * Hypothalamus
B. Medulla oblongata
C. Cerebral cortex
D. Cerebellum
E. Thalamus
719. During clinical scientific researches of the brain, it was explored ,that general sensory center is
located at…
A. * Thalamus
B. Epithalamus
C. Metathalamus
D. Subthalamic area
E. Hypothalamus
720. During clinical scientific researches of the brain, it was explored, that the cortical hearing area is
located at…
A. * Superior temporal gyrus
B. Unculus
C. Superior frontal sulcus
D. Insula
E. Calcarine sulcus
721. During clinical scientific researches of the brain, it was explored, that cortical smell area is locate
at…
A. * Uncus
B. Superior temporal gyrus
C. Insula

D. Superior frontal sulcus


E. Calcarine sulcus
722. During clinical researches on volunteers it was explored, that next structure plays an important role in
regulating of voluntary movements. It is…
A. * Primary motor gyrus of frontal lobe
B. Basal ganglias
C. Cerebral peduncles
D. Hypotalamus
E. Postcentral gyrus of parietal lobe
723. During operation it was diagnosed that free circulation of the intracerebral liquid is blocked at the
aqueduct. What formations it communicate?
A. * Fourth and third ventricles
B. Third and second ventricles
C. Central canal of fourth ventricle
D. Central canal and subarachnoid space
E. Fourth ventricle and subarachnoid space
724. On the MRI picture it is indicate small hemorrhage in the pineal body. Where it is locate?
A. * Epithalamus
B. Thalamus
C. Metathalamus
D. Subthalamic araea
E. Hypothalamus
725. On the MRI picture of brain doctor can indicate the four distinct lobes of the cortex, they are…
A. * Occipital, parietal, temporal and frontal lobes
B. Front, back, side and top lobes
C. Hind, mid, fore and association lobes
D. Sensory, auditory, visual and motor lobes
E. None of the above
726. On the MRI picture of brain doctor can indicate the largest part of the diencephalon, it is…
A. * Thalamus
B. Pineal body
C. Hypothalamus
D. Intermediate mass
E. Pituitary gland
727. On the picture of brain, made by electronic microscope, we can see the most common type of
neurons, which have several dendrites and one axon extending from the cell body .They are…
A. * Multipolar neurons
B. Afferent neurons
C. Pseudounipolar neurons
D. None of the above
E. Interneurons
728. The patient has concussion of the brain, he complain about dysfunction of verbal communication,
higher intellectual processes, voluntary control of skeletal muscles. Which cerebral lobes are injured?

A. * Frontal
B. Occipital
C. Temporal

D. Parietal
E. Non of the above
729. The patient has some problems with regulating sensory activity. What region of brain may be
involved in pathologic process?
A. * Somatic ,parietal
B. Voluntary motor, occipital
C. Somatic, temporal
D. Autonomic, parietal
E. Autonomic, temporal
730. The patient has some problems with regulating consciously movements of skeletal muscles. What
region of brain may be involved in pathologic process?
A. * Primary motor, frontal lobe
B. Primary motor, temporal
C. Primary motor parietal
D. Somatic sensory, parietal
E. Somatic sensory frontal
731. The scientist explored by clinical researches of the brain, that autonomic center is located at…
A. * Hypothalamus
B. Epithalamus
C. Thalamus
D. Optic chiasma
E. Subthalamic area
732. The scientist explored by clinical researches of the brain, that visual cortex is located at…
A. * Calcarine sulcus
B. Uncus
C. Insula
D. Superior temporal gyrus
E. Superior frontal sulcus
733. After clinical scientific researches it was explored that “Pleasure center” and “Punishment center” of
the brain are located in the…
A. * Limbic system
B. Prefrontal area
C. Intermediate mass
D. Precentral gyrus
E. Mammilliary bodies
734. After examination of CT image of the brain structures, can you say what sentence is true?
A. * The lateral ventricles are two cavities, one in each cerebral hemisphere
B. The fourth ventricle is diamond-shaped expansion of the central canal within the brainstem
C. The third ventricle is a thin chamber lying below the corpus callosum and septum pellucidum
D. None of the above
E. All of the above
735. After examination of CT image indicate structure that represent structure of lateral ventricle, it is…
A. * Corn anterior
B. Mesencephalon
C. Pons
D. Medulla oblongata
E. Spinal cord

736. After examination of CT image indicates structures that represent structure of lateral ventricle, it is…
A. * Foramen interventriculare
B. Pons
C. Mesencephalon
D. Spinal cord
E. Medulla oblongata
737. After examination of CT image indicates structure that represents structure of lateral ventricle, it is…
A. * Cornu frontale
B. Pons
C. Medulla oblongata
D. Mesencephalon
E. Spinal cord
738. After examination of CT image indicate structure that represent structure of lateral ventricle, it is…
A. * Stria terminalis
B. Pons
C. Spinal cord
D. Medulla oblongata
E. Mesencephalon
739. After examination of CT image indicate structure that represent structure of lateral ventricle, it is…
A. * Pars centralis
B. Spinal cord
C. Pons
D. Mesencephalon
E. Medulla oblongata
740. After examination of CT image indicate structure that represent structure of lateral ventricle, it is…
A. * Cornu temporal
B. Mesencephalon
C. Pons
D. Medulla oblongata
E. Spinal cord
741. After examination of CT image indicate structure that represent structure of lateral ventricle, it is…
A. * Cornu inferius
B. Pons
C. Medulla oblongata
D. Mesencephalon
E. Spinal cord
742. Indicate, after examination of the MRI picture of the brain, what structures connect corpus callosum?
A. * Right and left cerebral hemispheres
B. Lateral lobes of the cerebellum
C. The calluses on the palms of your hand
D. Cerebellum to the pons and medulla
E. The calluses on the palm of your hands
743. Indicate, after examination of the MRI picture of the brain, what is the principal connection between
cerebral hemispheres?
A. * The corpus callosum
B. The cerebral peduncles

C. Cerebellum to the pons and medulla


D. The brain system
E. The fornix
744. Indicate, after examination of the MRI picture of the brain, what structure doesn’t take part in
formation of corpus callosum?
A. * Cortex cerebelli
B. Rostrum
C. Splenium corporis callosi
D. Truncus corpora callosi
E. Radiatio corporis callosi
745. The patient has problems with motor function of left hand. Where the first neuron (axon) of lateral
corticospinal tract is passes through?
A. * Internal capsule
B. Lentiform nucleus
C. External capsule
D. Extrema capsule
E. Claustrum
746. The patient has significations of high intracranial pressure. What structure of brain has concern to
production of cerebrospinal fluid?
A. * Choroid plexus
B. Meninges
C. Corpora quadrigmina
D. Crura cerebri
E. Adenohypophysis glands
747. At patient was diagnosed extra production of cerebrospinal fluid. What structure response about it?
A. * Choroid plexus
B. Arachnoid granulations
C. Spinal cord
D. Pia mater
E. Dural sinus
748. At the patient was diagnosed “nearsightedness”, it means:
A. * Myopia
B. Presbiopia
C. Hemianopia
D. Hyperopia
E. Emmetropia
749. During microscopic investigation of eyeball doctor can see that photoreceptors are found in the..
A. * Retina
B. Cornea
C. Choroid coat
D. All the above
E. Sclera
750. During microscopic investigation of retina doctor can see that ora serrata separate…
A. * Parts of retina
B. Retina and chroidea
C. Chroidea and cornea
D. Iris and ciliary body

E. Choroidea and sclera


751. During neurosurgical operation doctor cut three connective tissue membranes covering and
protecting CNS structures are the…
A. * Meninges, dura mater
B. Periosteum, dura mater
C. Meninges, arachnoid
D. Endosteum, pia mater
E. Meninges, pia mater
752. The doctor diagnosed that the patient has blocked drainage of aqueous humor at the canal of
Schlemm, what condition results?
A. * Glaucoma
B. Cataract
C. Iritis
D. Conjunctivitis
E. Blefaritis
753. The patient cannot see clear picture, focusing the light rays involves which of the following?
A. * Change in the sharp of the lens
B. Change in the sharp of the cornea
C. Change in the relative position of the lens
D. Change in the location of the optic disc
E. Change in the density of the aqueous humor
754. The patient has an inflammation of delicate membrane that lines the eyelids and covers part of outer
surface of the eyeball, it is…
A. * Conjunctiva ,cornea
B. Conjunctiva, ciliary glands
C. Arachnoid, cornea
D. Conjunctiva, lacrimal apparatus
E. Endothelium, cornea
755. The patient has diagnosed “glaucoma”. It is a condition:
A. * Characterized by abnormally high intraocular pressure
B. Results eventually in opacity of the lens or cornea
C. Which may be caused by exposure to ultraviolet light
D. In which the conjunctiva is inflamed due to irritants such as dust or smoke
E. Characterized by encrustations on the eyelids
756. The patient has inflammation of the transparent anterior portion of the sclera, it is…
A. * Cornea
B. A and B
C. Retina
D. Choroid
E. Ciliary body
757. The patient has problems with accommodation of the lens. What is responsible for this function?
A. * Ciliary muscle
B. Pupil
C. Pectinate ligament
D. Dilator muscle
E. Sphincter muscle

758. The patient has problems with production of the aqueous humor of the eye, what structure responds
of it?
A. * Ciliary process
B. Pectinate ligaments
C. Iridocorneal angle
D. Ora serrata
E. Zonula ciliaris
759. The patient has problems with vision. Which way of light within the eyeball?
A. * Cornea, anterior chamber, pupil, posterior chamber, lens, vitreous body
B. Pupil, cornea, anterior chamber, posterior chamber, lens, vitreous body
C. Cornea, pupil, lens, anterior chamber, posterior chamber, vitreous body
D. Cornea, anterior chamber, pupil, lens, posterior chamber, vitreous body
E. Pupil, anterior chamber, cornea, lens, vitreous body, posterior chamber
760. At the newborn child some special sense is NOT function, it is…
A. * Balance
B. Vision
C. Taste
D. Smell
E. Hearing
761. Doctor explore reaction of patient’s eye on bright light. Which muscle is response for this reaction?
A. * Sphincter pupillae
B. Ciliary muscle
C. Dilator pupilae
D. Oblique superior
E. Rectus superior
762. Doctor explores reaction of patients’ eye on darkness. Which muscle is response for this reaction?
A. * Dilator papillae
B. Oblique superior
C. Sphincter papillae
D. Ciliary muscle
E. Rectus superior
763. It was diagnosed that patient has problems with accommodation. What is response about thigh
function?
A. * Ciliary muscle
B. Pupil
C. Sphincter muscle
D. Pectinate ligament
E. Dilator muscle
764. It was diagnosed that patient has problems with chemoreceptors, they are…
A. * Taste, smell
B. Sight, smell
C. Smell, hearing
D. Pain, pressure
E. Proprioception, pain
765. The doctor diagnosed no mobile the malleolus, incus and stapes they…
A. * Form a bridge between the tympanum and the oval window
B. Are parts of the organ of Corti

C. Both A and B are correct


D. Are surrounded by perilymph
E. Are locate in the inner ear
766. The doctor diagnosed that patient has blocked Eustachian tube. With which structure they
communicate?
A. * Nasopharynx and tympanic cavity
B. Larynx and pharynx
C. Laryngeal pharynx and oropharynx
D. Larynx and tympanic cavity
E. Oropharynx and tympanic cavity
767. The doctor diagnosed that the patient has an inflammation of the auditory (or Eustachian) tube, which
is locate between the…
A. * Throat and middle ear
B. Throat and inner ear
C. Throat and trachea
D. Middle ear and inner ear
E. Throat and larynx
768. The patient after car accident has injured of auditory bones, they are associated with the…
A. * Middle ear
B. Semicircular canals
C. External ear
D. Internal ear
E. Cochlear duct
769. The patient has an infection at cochlea, it is associate with the…
A. * Internal ear
B. Eustachian tube
C. Semicircular canals
D. External ear
E. Middle ear
770. The patient has an infection in the part of ear which locates between tympanic membrane and oval
window, they associate with the…
A. * Middle ear
B. Cochlear duct
C. External ear
D. Inner ear
E. Semicircular canals
771. The patient has an inflammation of membrane that in lay the eyelids and covers part of the outer
surface of the eyeball, it is…
A. * Conjunctiva, cornea
B. Conjunctiva,cilary glands
C. Endothelium,cornea
D. Arachnoid,cornea
E. conjunctiva,lacrimal apparatus
772. The patient has blocked structure that provides a means of equalizing the air pressure in the middle
ear chamber it is…
A. * Auditory tube
B. Mastoid sinus

C. Oval window
D. Seminiferous tube
E. Round window
773. When patient crying where do tears flow from lacrimal gland?
A. * Conjunctiva sac
B. Lacrimal sac
C. Inferior nasal meatus
D. Superior nasal meatus
E. Nasolacrimal canal
774. When patient crying where do tears flow from nasolacrimal canal?
A. Lacrimal sac
B. Superior nasal meatus
C. Excretory ducts
D. Conjunctival sac
E. * Inferior nasal meatus
775. When patient has dysfunction of the taste, what cranial nerves may be injured?
A. * Facial, vagus and glossopharyngeal
B. Trigeminal, facial, and glossopharyngeal
C. Trigeminal, hypoglossal and glossopharyngeal
D. Hypoglossal, vagus and glossopharyngeal
E. Olfactory, trigeminal and glossopharyngeal
776. When the doctor suddenly exposed to bright light the eyes of patient, the pupils immediately
constrict. This is called the…
A. * Photo pupillary reflex
B. Stretch reflex
C. Nociception reflex
D. Crossed-extensor reflex
E. Accommodation pupillary reflex
777. During central line catheterization doctor making puncture of subclavian vein.Where it is locate?
A. * Neck
B. Abdomen
C. Thoracic cavity
D. Pelvis
E. Correct answer is missing
778. In arterial bleeding a tourniquet is imposed to…
A. * Proximal to the side of damage
B. Distal to the side of damage
C. Directly on the wound
D. On opposite extremity
E. Simultaneously proximal and distal to the site of damage
779. In venous bleeding a tourniquet is imposed to…
A. * Distal to the side of damage
B. Proximal to the side of damage
C. Directly on the wound
D. On opposite extremity
E. Simultaneously proximal and distal to the site of damage

780. Movement of the head make stimulations of hears cells in the semicircular canals it is results from
the movement of…
A. * Endolymph
B. Cerebrospinal fluid
C. Perilymph
D. Otoliths
E. Aqueous humor
781. The doctor explore pupillary reflex at the patient, pupils will be dilate in response to the contraction
of the…
A. * Radial iris muscle
B. Ciliary muscle
C. Orbicularis oculi muscle
D. Ciliar iris muscle
E. Levator palpebrae superior muscle
782. The doctor put on the tourniquet on patients extremity. What time it is imposed in summer?
A. * To 1-2 hours
B. To 15-30 min
C. To 30-60min
D. To 4-6 hours
E. To 12 hours
783. The doctor put on the tourniquet on patients extremity. What time it is imposed in winter?
A. * T0 30-60 min
B. To 15-30 min
C. To 1-2 hours
D. To 4-6 hours
E. To 12 hours
784. The doctor put on tourniquet at the patients extremity. What are the criteria of efficiency of
imposition of it?
A. * Stop bleeding
B. Pale extremity absence of pulsation
C. Coldness of the extremity
D. Disappearance of sensation
E. Absence of pulsation
785. The patient has an inflammation of cervical lymph nodes. To what group they are belong?
A. * All
B. Nodi profundi superior
C. Nodi profundi inferior
D. Nodi supraclavicularis
E. Nodi superficiales
786. Dentist introduces the cotton wool into the lumen between the cheek and an alveolar process of the
maxilla. Which orifice of gland duct does he close?
A. * parotid gland
B. thyroid gland
C. submandibular gland
D. all above mentioned
E. sublingual gland
787. The patient has the inflammation of tooth alveola. What part of the tooth can be injured?

A. * tooth root
B. tooth crown
C. tooth cervix
D. pulp cavity
E. canal of tooth root
788. The patient has the inflammation of tooth alveola. What part of the tooth can be injured?
A. * tooth root
B. tooth cervix
C. pulp cavity
D. canal of tooth root
E. tooth crown
789. Which duct opens on the cheeks into vestibule of mouth cavity opposite the second superior molar ?
A. * Parotid
B. Palatine
C. Sublingual
D. Submandibular
E. Buccal
790. A patient 18 years old got symptoms of acute appendicitis - acute pain in right lumbar area. What
type of placing of appendix can be supposed?
A. * Retrocecal retroperitoneal.
B. Descending.
C. Ascending.
D. Medial.
E. Lateral.
791. A patient 40 years old, appealed to the doctor with complaints about frequent heartburn of esophagus.
After examination insufficiency of what sphincter is exposed?
A. * Cardiac.
B. Pyloric.
C. Phalangeal.
D. Esophageal.
E. Duodenal.
792. A patient 60 years old because of malignant tumor of papilla duodenum major had an obstructive
icterus. Deleting a tumor a surgeon must remember, that on the papilla duodenum major is opened:
A. * Hepatoduodenal ampule.
B. Common hepatic duct.
C. Cystic duct.
D. Right hepatic duct.
E. Left hepatic duct.
793. A patient 65 years old was hospitalized with suspicion on the tumor of superior part of esophagus.
But after X-ray examination a tumor process is exposed on the border of esophagus and pharynx. On
level of what cervical vertebra was the tumor locate?
A. * 6.
B. 5.
C. 4.
D. 3.
E. 2.

794. A patient complains on pain in a right iliac area. During the palpation a mobile, soft, sickly intestine
was determined. What part of intestine did the doctor palpate?
A. * Cecum.
B. Sigmoid colon.
C. Transverse colon.
D. Ascending colon.
E. Jejunum.
795. A patient complaints on disturbance of evacuative function of stomach (the protracted delay of food
in stomach). After the examination, the tumor of initial part of duodenum was detected. What is the
localization of tumor?
A. * Pars superior.
B. Pars inferior.
C. Pars descendens.
D. Pars ascendens.
E. Flexura duodeni inferior.
796. A patient complaints on pain in the superior part of umbilical area. During the palpation a mobile,
soft, sickly intestine is determined. What part of intestine does the doctor palpate?
A. * Transverse colon.
B. Jejunum.
C. Duodenum.
D. Ileum
E. Sigmoid.
797. A patient has acute appendicitis, which clinically is similar a hepatic colic. What position of
vermiform appendix makes it possible?
A. * Ascending.
B. Medial.
C. Descending.
D. Lateral.
E. Retrocecal.
798. A patient was hospitalized with the ulcer of esophagus. After some time the symptoms of
inflammation of peritoneum appeared. In what part of esophagus the perforation happened?
A. * Abdominal.
B. Thoracic.
C. Cervical.
D. Stomach.
E. Pharyngeal.
799. At 4e child 8 years old, the foreign body of esophagus at the level of a 10 thoracic vertebra was
revealed by X-ray. At the area of what esophageal narrowing had the foreign body stopped?
A. * Diaphragmatic narrowing.
B. Pharyngeal narrowing.
C. Bronchial narrowing.
D. Aortal narrowing.
E. Abdominal narrowing.
800. At children often possible to observe difficulty of nasal breathing. This is related to extra
development of lymphoid tissue in pharyngeal mucous membrane. Growth of what tonsil can cause
this phenomenon?
A. * Tonsilla pharyngea.
B. Tonsilla palatine.

C. Tonsilla lingualis.
D. Tonsilla tubaria.
E. All of these tonsils.
801. Dentist introduces the cotton wool into the lumen between the cheek and an alveolar process of the
maxilla. Which orifice of gland duct does he close?
A. * parotid gland
B. thyroid gland
C. submandibular gland
D. all above mentioned
E. sublingual gland
802. During endoscope examination of small intestine on a mucous membrane longi tudinal folds were
visible. What part ol what intestine did a doctor see?
A. * Initial part (ampoule) of duodenum.
B. Horizontal part of duodenum.
C. Ascending part of duodenum.
D. Initial part of jejunum.
E. Final part of ileum.
803. During the first feeding of newborn, milk began to flow out of a nasal cavity. What anomaly or
abnormally got a newborn?
A. * Cleft palate.
B. Cleft lip.
C. Atrezy of esophagus.
D. Atrezy of anal canal.
E. Esophageal-tracheal fistula.
804. During the operation on a small intestine a doctor found out the area of mucous membrane, where on
base of circular folds the longitudinal fold was present. What part of small intestine does such
structure have?
A. * Pars descendens duodeni.
B. Pars horizontal is duodeni.
C. Pars ascendens duodeni.
D. Initial department of jejunum.
E. Distal department of ileum.
805. During the operation on the strangulated umbilical hernia in a hernia sack a part of intestine was
revealed with the appendixes of serous, which contain fat tissue. What part of intestine was
strangulated?
A. * Transverse colon.
B. Duodenum.
C. Ileum.
D. Jejunum.
E. Cecum.
806. In patient follicle angina was complicated by severe otitis (inflammation of middle ear). What
anatomical precondition does exist for this?
A. * Eustachian tube.
B. Fallopian tube.
C. Anomalies of pharynx.
D. Lymphoepythelial ring of Valdeer- Pirogov.
E. Presence of piriform recess.

807. In the mucous of intestine a surgeon saw group of lymphoid follicles (Peyer's patches). What
department of intestine it is?
A. * Ileum.
B. Jejunum.
C. Cecum.
D. Duodenum.
E. Rectum.
808. Mother of 8 years old child appealed to doctor with complaints on pain during swallowing and high
temperature. During examination a doctor saw an edema and hyperemia of lymphoid tissue between
the arches of soft palate. What tonsil does situate in this place?
A. * Tonsilla palatina.
B. Tonsilla pharyngealis.
C. Tonsillatubaria.
D. Tonsillalingualis.
E. Tonsilla nasalis.
809. Parents asked the dentist about the delay change of teeth in a child 9 years old, in whose mouth is the
only milk teeth. At what age begin erupting permanent teeth?
A. * Since 6-7 years
B. Since 10 -12 years
C. Since 2 years
D. Since 16 years
E. Since 5-6 years
810. The patient erroneously drank the solution of the acetic acid. Which membrane of the esophagus was
damaged mostly?
A. * mucous.
B. serous
C. adventitia
D. muscle and serous
E. muscular
811. The patient has the inflammation of tooth alveola. What part of the tooth can be injured?
A. * tooth root
B. tooth cervix
C. pulp cavity
D. canal of tooth root
E. tooth crown
812. The patient was hospitalized in surgical department with the penetrable wound of anterior abdominal
wall. A traumatic canal passed superiorly to the small curvature of stomach. Through what formation
of peritoneum did atraumatic channel passed?
A. * Ligamentum hepatogastricum.
B. Ligamentum gastrocolicum.
C. Ligamentum hepatoduoduodenale.
D. Ligamentum hepatorenale.
E. Ligamentum triangulare sinistrurn.
813. The pregnant women 30 years old got the raised doze of radiation. Because of this, the process of
intrauterine development of fetus was disordered. At formation of an oral cavity did not take a place a
fusing of the lateral nasal proc¬ess and maxillary process. Occurrence of what anomaly is supposed?

A. * Cleft palate.

B. Atrezy of esophagus.
C. Microstoma.
D. Makrostoma.
E. Cleft lip.
814. The X-ray examination of stomach of the patient 37 years old with brachymorphic constitution with a
conical chest was provided. What form of stomach most probably will be determined?
A. * Form of horn.
B. Form hook.
C. Form of stocking.
D. The lengthened stomach.
E. Form of the extended hook.
815. The X-ray examination of stomach of the patient of 42 years old with mesomorphic constitution with
a cylindrical chest was provided. What form of stomach most probably be determined?
A. * Form hook.
B. Form of horn.
C. Form of stocking.
D. Form of the extended hook.
E. The lengthened stomach.
816. The X-ray examination of stomach of the woman is 28 years old, dolychomorphyc constitution with
a flat thorax was provided. What form of stomach most probably will show the pathology?
A. * Form of stocking.
B. Form hook.
C. Form of horn.
D. Form of the extended hook.
E. Ovalfonn.
817. To a hospital the child was delivered with complaints to a retrosternal pain, appearing after
swallowing and accompanying with cough. After X-ray research, the foreign body in thickness of a
wall of esophagus at a level 5 thoracic vertebra was revealed. In the level of what narrowing of the
esophagus was a damage of its wall occurred?
A. * In the place of crossing with the left main bronchus.
B. In the place of adjoining of arch of aorta.
C. In the place of passing through a dia¬phragm.
D. In the place of passing to the stom¬ach.
E. In the place of transition of pharynx to esophagus.
818. To a hospital the child was delivered with complaints to a retrosternal pain,appearing after
swallowing and accompanying with cough. After X-ray research, the foreign body in thickness of a
wall of esophagus at a level 4 thoracic vertebra was revealed. In the level of what narrowing of the
esophagus was a damage of its wall occurred?
A. * In the place of adjoining of arch of aorta.
B. In the place of decussating with the left main bronchus.
C. In the place of passing through a diaphragm.
D. In the place of passing to the stomach.
E. In the place of transition of pharynx in esophagus.
819. To patient the endoscopy of duodenum was provided. The examination revealed an inflammation of
large duodenal papil¬la and disturbances of bile secretion to the intestine. In which part of duodenum
the disturbances were revealed?
A. * Descending part.
B. Ascending part.

C. Bulb.
D. Superior part.
E. Inferior horizontal part.
820. To the child 6 years old Meckel diverticle was diagnosed. In what department of gastro-intestinal
tract is it necessary to search it during operation?
A. * In the area of ileum.
B. In the area of jejunum.
C. In the area of colon.
D. In the area of duodenum.
E. In the area of sigmoid colon.
821. Which duct opens on the cheeks into vestibule of mouth cavity opposite the second superior molar ?
A. * Parotid
B. Buccal
C. Palatine
D. Sublingual
E. Submandibular
822. A 40 years old man was hospitalized to surgical department with a diagnosis - splenic rupture. In
what anatomical formation will the blood accumulate?
A. * Bursa pregastrica.
B. Bursa hepatica.
C. Bursa omental is.
D. Excavatio rectovesicalis.
E. Right lateral canal.
823. A man with closed trauma of the right part of abdomen and suspicion on the liver rupture was
delivered to traumatic department. In what formations of peritoneum it is necessary to expect the
collection of blood?
A. * In the rectovesical excavation.
B. In the left lateral canal.
C. In the superior ileocecal angle.
D. In an intersigmoid corner.
E. In the omental bursa.
824. A patient 18 years old got symptoms of acute appendicitis - acute pain in right lumbar area. What
type of placing of appendix can be supposed?
A. * Retrocecal retroperitoneal.
B. Descending.
C. Ascending.
D. Medial.
E. Lateral.
825. A patient 27 year’s old entered clinic with complaints on pain in region of stomach, nausea. After the
palpation of abdomen a surgeon found a point of pain on crossing of right costal arc and lateral
margin t of right rectus abdominis muscle (point of Kehr). What preliminary diagnosis [ probably
will the doctor do?
A. * Pancreatitis.
B. Nephritis.
C. Cholecystitis.
D. Gastritis.
E. Duodenitis.

826. A patient 27 year’s old entered clinic with complaints on pain in region of stomach, nausea. After the
palpation of abdomen a surgeon found a point of pain on crossing of right costal arc and lateral
margin t of right rectus abdominis muscle (point of Kehr). What preliminary diagnosis [ probably
will the doctor do?
A. * Duodenitis.
B. Pancreatitis.
C. Cholecystitis.
D. Gastritis.
E. Nephritis.
827. A patient has a perforation and inflammation of vermiform appendix. Revision of what recess of
peritoneum must a doctor provide first?
A. * Recessus retrocecalis.
B. Recessus duodenalis superior.
C. Recessiis duodenalis inferior.
D. Recessus ileocecal is superior.
E. Recessus ileocecalis inferior.
828. A patient has acute appendicitis, which clinically is similar a hepatic colic. What position of
vermiform appendix makes it possible?
A. * Ascending.
B. Medial.
C. Descending.
D. Lateral.
E. Retrocecal.
829. A patient was operated on indication of a trauma of liver with a bleeding to a hepatic bursa. What
border prevents entering the blood in the pregastric bursa?
A. * Falciform ligament.
B. Round ligament.
C. Coronary ligament.
D. Right triangulate ligament.
E. Right triangulate ligament.
830. A patient with the traumatic splenic rupture, which needs an urgent operation, was delivered to a
surgical department. In what formation of peritoneum is it located?
A. * Bursa pregastrica.
B. Bursa hepatica.
C. Burs omentalis.
D. Sinus mesentericus sinister.
E. Sulcus paracolicus sinister.
831. After the revision of peritoneal cavity with indication on peritonitis, the local abscess at the root of
mesentery was found. In what formation of peritoneum it situate?
A. * Intersigmoid fossa.
B. Right mesenteric sinus.
C. Right lateral canal.
D. Left lateral canal.
E. Left mesenteric sinus.
832. At a patient after an operation (stitching of penetrate wound of small intestine) an interintestinal
abscess was formed, which drainage to right mesenteric sinus. Where can a purulent effluent spread
in?
A. * Stay in bounds of sinus.

B. To a small pelvis.
C. To the retrocecal recess.
D. Right lateral cannel.
E. Intersigmoid recess.
833. At acute destructive pancreatitis usually the inspection of omental burse is carry out because the
pancreas is one of its wall. What wall?
A. * Posterior one.
B. Superior one.
C. Anterior one.
D. Inferior one.
E. Left one.
834. At operative interference in an abdominal region a surgeon must get the omental bursa. How can a
surgeon get in this part of cavity of peritoneum, without violating integrity of lesser omentum?
A. * Through the omental foramen.
B. Through right paracolic sulcus.
C. Through left paracolic sulcus.
D. Through the right mesenteric sinus.
E. Through the left mesenteric sinus.
835. At patient one of gangrenous form of acute pancreatitis is diagnosed. To what peritoneal spaces from
below does the exudate spread?
A. * Bursa omentalis.
B. Infrahepatic fissure.
C. Bursa pregastrica.
D. Left lateral canal.
E. Between the layers of anterior and posterior omental laminas.
836. At the patient of 52 years old a chronic stone cholecystitis is diagnosed. At retrograde
cholecystectomy a surgeon checks hepatoduodenal ligament. What elements of this ligament is it
necessary to select and bandage?
A. * Portal vein, proper hepatic artery, common bile duct.
B. Common bile duct, common hepatic duct.
C. Proper hepatic artery, duct of gall bladder.
D. Portal vein, artery of gallbladder.
E. Cystic duct, artery of gallbladder.
837. At the patient of 52 years old a chronic stone cholecystitis is diagnosed. At retrograde
cholecystectomy a surgeon checks hepatoduodenal ligament. What elements of this ligament is it
necessary to select and bandage?
A. * Portal vein, proper hepatic artery, common bile duct.
B. Common bile duct, common hepatic duct.
C. Cystic duct, artery of gallbladder.
D. Portal vein, artery of gallbladder.
E. Proper hepatic artery, duct or urinary bladder.
838. Doctors during surgical operation examine the organs of the abdominal cavity. Pancreatic cyst was
found. What relation of the peritoneum and pancreas?
A. * Extraperitoneal position
B. Mesoperitoneal position
C. Peritoneum does not cover the pancreas
D. Peritoneum forms mesentery for pancreas
E. Intraperitoneal position

839. Doctors during surgical operation examine the organs of the abdominal cavity. Pancreatic cyst was
found. What relation of the peritoneum and pancreas?
A. * Peritoneum does not form mesentery for pancreas
B. Mesoperitoneal position
C. Peritoneum forms sheaths of pancreas
D. Peritoneum forms mesentery for pancreas
E. Intraperitoneal position
840. Doctors during surgical operation examine the organs of the abdominal cavity. Pancreatic cyst was
found. What relation of the peritoneum and pancreas?
A. * Peritoneum covers one side the pancreas
B. Mesoperitoneal position
C. Peritoneum does not cover the pancreas
D. Peritoneum does not form mesentery for pancreas
E. Intraperitoneal position
841. Doctors during surgical operation examine the organs of the abdominal cavity. Pancreatic cyst was
found. What relation of the peritoneum and pancreas?
A. * Extraperitoneal position
B. Peritoneum forms mesentery for pancreas
C. Intraperitoneal position
D. Mesoperitoneal position
E. Peritoneum does not cover the pancreas
842. Doctors during surgical operation examine the organs of the abdominal cavity. Pancreatic cyst was
found. What relation of the peritoneum and pancreas?
A. * Pancreas is positioned in retroperitoneal space
B. Peritoneum forms mesentery for pancreas
C. Intraperitoneal position
D. Mesoperitoneal position
E. Peritoneum does not cover the pancreas
843. During operation on indication the bile stones a surgeon must find a common hepatic duct. Between
the layers of what ligament does it situate?
A. * Hepatoduodenal.
B. Hepatogastric.
C. Hepatorenal.
D. Round ligament of the liver.
E. Ligamentum venosum.
844. During operation on indication the bile stones a surgeon must find a common hepatic duct. Between
the layers of what ligament does it situate?
A. * Hepatoduodenal.
B. Hepatogastric.
C. Hepatorenal.
D. Round ligament of the liver.
E. Ligamentum venosum.
845. During the cholecystectomy (removing of a gall bladder), which performed from a fundus,
concrements (bile stones) can move through wide cystic duct to the next departments of the bile
ducts. What place a surgeon must check in?
A. * Ductus choledochus.
B. Ductulus billifer.
C. Ductus hepaticus dexter.

D. Ductus hepaticus sinister.


E. Ductus hepaticus communis.
846. During the cholecystectomy (removing of a gall bladder), which performed from a fundus,
concrements (bile stones) can move through wide cystic duct to the next departments of the bile
ducts. What place a surgeon must check in?
A. * Ductus choledochus.
B. Ductus hepaticus communis.
C. Ductus hepaticus dexter.
D. Ductus hepaticus sinister.
E. Ductulus billifer.
847. During the operation on the strangulated umbilical hernia in a hernia sack a part of intestine was
revealed with the appendixes of serous, which contain fat tissue. What part of intestine was
strangulated?
A. * Transverse colon.
B. Duodenum.
C. Ileum.
D. Jejunum.
E. Cecum.
848. In a patient with destructive appendicitis, as complication a subdiaphragmatic abscess appeared. In
what formation of peritoneum does it locate?
A. * Bursa hepatica.
B. Bursa pregastrica.
C. Bursa omentalis.
D. Right lateral canal.
E. Left lateral channel.
849. In man the perforation of ulcer of horizontal part of duodenum was revealed. In what area the
inflammatory process will situate?
A. * Right mesenteric sinus.
B. Left mesenteric sinus.
C. Right lateral channel.
D. Left lateral channel.
E. Omental bursa.
850. The abscess of the left lobe of liver in patient 48 years old was diagnosed. The generalization of
inflammation can cause the peritonitis. Inflammation of what part of peritoneum will be observed?
A. * Pregastric bursa.
B. Left mesenteric sinus.
C. Left mesenteric sinus.
D. Omental bursa.
E. Hepatic bursa.
851. The injured by firearms was delivered to hospital with the strong bleeding. During the examination
the surgeon revealed, that the bullet canal passed through the anterior wall of stomach, fornix of
stomach and went out at the level of the X rib on the left middle axillary line. What organ is damaged
with the stomach?
A. Transverse colon.
B. Left kidney.
C. Pancreas.
D. * Spleen.
E. Left lobe of liver.
852. The injured by firearms was delivered to hospital with the strong bleeding. During the examination
the surgeon revealed, that the bullet canal passed through the anterior wall of stomach, fornix of
stomach and went out at the level of the X rib on the left middle axillary line. What organ is damaged
with the stomach?
A. * Spleen.
B. Left kidney.
C. Pancreas.
D. Transverse colon.
E. Left lobe of liver.
853. The patient complains on a pain in the right hypohondriac area. After the palpation in this area doctor
found that liver has increased. What are biggest parts of the liver?
A. * Lobes
B. Nephrons
C. Acynus
D. Segments
E. Lobules
854. The perforation of ulcer of posterior wall of stomach was diagnosed at a patient 40 years old. To what
anatomical formation the blood and content of stomach will get?
A. * Bursa omentalis.
B. Bursa pregastrica.
C. Bursa omentalis
D. Left lateral canal.
E. Bursa hepatica.
855. The tumor of ascending colon in woman 65 years old was diagnosed. Peritonitis of what area of
peritoneum will be observed in the case of destruction of tu¬mor?
A. * Right mesenteric sinus.
B. Left mesenteric sinus.
C. Hepatic bursa.
D. Pregastric bursa.
E. Omental bursa.
856. The tumor of descending colon in man 69 years old was diagnosed. Peritonitis of what area of
peritoneum will be observed in the case of destruction of tumor?
A. * Left mesenteric sinus.
B. Right mesenteric sinus.
C. Omental bursa.
D. Pregastric bursa.
E. Hepatic bursa.
857. The ulcer of anterior wall of stomach was diagnosed. Inflammation of what part of peritoneum is
most possible?
A. * Bursa pregastrica.
B. Bursa hepatica.
C. Bursa omentalis.
D. Right mesenteric sinus.
E. Left mesenteric sinus.
858. The victim got a knife wound in right lumbar area. The damage of right kidney takes a place. What
organs of retroperitoneal space can be damaged here?
A. * Duodenum.
B. Transverse colon.
C. Left curvature of transverse colon.
D. Descending colon.
E. Initial part of ileum.
859. To the patient 50 years concerning a pancreatitis the resection of tail of pancreas is performed. It is
necessary to know, that a pancreas situate\:
A. * Extraperitoneal.
B. Mesoperitoneal.
C. Intraperitoneal.
D. Parenteral.
E. Intramuraly.
860. To victim with the punctured wound of anterior wall of stomach appealed for surgical help. To what
formation of peritoneal cavity did the content of stomach get?
A. * Pregastric bursa.
B. Omental bursa.
C. Hepatic bursa.
D. Left mesenteric sinus.
E. Right mesenteric sinus.
861. A patient was hospitalized in a clinic with complaints of cuffing with blood, hyperhydrosis. The X-
ray revealed the tuberculosis focus in the superior lobe of the left lung. An operation was
recommended. What is greatest quantity of segments can be removed in structure of the superior lobe
of left lung?
A. * 5.
B. 6.
C. 3.
D. 2.
E. l.
862. A pathologist made dissection of patient, dyed from long chronic disease of kidney. The dissection
revealed congenital absence of one kidney. What diagnosis did a doctor?
A. * Agenesia renis.
B. Ren duplex.
C. Distopia renis.
D. Ren arcuata.
E. Ren anularis.
863. A patient 28 years old appealed to nephrology with a diagnosis a falling of the right kidney
(nephroptosis). X-ray contrast urography confirmed a preliminary diagnosis. What is norm position
of the right kidney according to 12th rib?
A. * 12 ribs cross a kidney in the superior third.
B. 12 ribs cross a kidney in the inferior third.
C. 12 ribs cross a kidney in the middle.
D. 12 ribs are projected on the superior pole.
E. 12 ribs are project on the inferior pole.
864. A patient 37 years old appealed to urology with a diagnosis a falling of the left kidney (nephroptosis).
X-ray-contrast urography confirmed a preliminary diagnosis. What is the position of the left kidney
concerning 12th rib in norm?
A. * 12 ribs cross a kidney in the middle.
B. 12 ribs cross a kidney in the superior third.
C. 12 ribs cross a kidney in the inferior third.
D. 12 ribs are projected on the superior pole.

E. 12 ribs are project on the inferior pole.


865. A patient 37 years old was delivered to pulmonology department with a diagnosis left-side exsudative
pleuritis. What anatomical formation of pleura the inflammatory exudates will be collected more
credible?
A. * Costodiaphragmatic sinus.
B. Costomediastinal sinus.
C. Phrenicomediastinal sinus.
D. All enumerated.
E. Pleural cupula.
866. A patient 43 years old appealed to urologist with the preliminary diagnosis floating kidney (ren
mobile). At X-ray contrast urography in vertical and lying position the displacement of the right
kidney has been marked. At what level in norm there is a right kidney according a vertebral column?
A. * From inferior edge of 11 thoracic to the middle of 3 lumbar vertebras.
B. From inferior edge of 12 thoracic to the middle of 4 lumbar vertebras.
C. From inferior edge of 11 thoracic to the superior edge of 1 lumbar vertebra.
D. From inferior edge of 12 thoracic to the superior edge of 2 lumbar vertebras.
E. From the middle of 11 thoracic to the superior edge of 3 lumbar vertebras.
867. A patient 65 years old was operated on indications of the tumor of ureter. During operation it is
revealed, that the tumor extended to mesentery of small in\: testine. What part of ureter is affected
with a tumor?
A. * Abdominal.
B. Pelvic.
C. Intraorganic.
D. Intrarenal.
E. Thoracic
868. A patient appealed in clinic with trauma of a kidney. During a surgical operation from the posterior
approach, it is necessary to press a renal artery. What is the sequence of the elements of renal leg in
its hilus from posterior to anterior one?
A. * Ureter, artery, vein.
B. Artery, ureter, vein.
C. Artery, vein, ureter.
D. Vein, ureter, artery.
E. Vein, artery, ureter.
869. A patient appealed to urology with the preliminary diagnosis floating kidney (ren mobile). At X-ray
contrast urography in vertical and lying position the displacement of the left kidney has been marked.
At what level in norm there is a left kidney according to a vertebral column?
A. * From the middle of 11 thoracic to the superior edge of 3 lumbar vertebras.
B. From inferior edge of 11 thoracic to the middle of 4 lumbar vertebras.
C. From the middle of 11 thoracic to the superior edge of 4 lumbar vertebras.
D. From inferior edge of 12 thoracic to the middle of 3 lumbar vertebras.
E. From inferior edge of 12 thoracic to the superior edge of 2 lumbar vertebras.
870. A patient died from acute renal insufficiency (because of edema of kidney's parenchyma). What layer
of a kidney will be separated hardly from renal parenchyma quring pathological anatomical
dissection?
A. * Adipose capsule.
B. Fibrous capsule.
C. Renal fascia.
D. Retroperitoneal fascia.

E. Preperitoneal fascia.
871. A patient has a urolithiasis. During re moving the concrement from the right ureter a surgeon cut the
wall of ureter. In what anatomical formation the urine will get?
A. * Retroperitoneal space.
B. Right mesemteric sinus.
C. Rectovesical excavation.
D. Prevesical space.
E. Right lateral canal.
872. A patient was hospitalized in clinic with a tumor of middle lobe of right lung. How many segments
can be removed during this operation?
A. * 2.
B. 3
C. 4.
D. 5.
E. l.
873. After a patient examination was revealed, that his right kidney is situated in iliac fossa (iliac kidney).
This congenital dystopia is differentiated from the nephroptosis (falling kidney)\:
A. * Decreasing of pararenal adipose body.
B. Absence of fibrous capsule.
C. Shape of kidney.
D. Anomalistic low position of kidney.
E. More caudal arising of renal artery from aorta.
874. At the sick child RDS (respiratory distress syndrome), connected with disturbances of surfactant
production was revealed. The surfactant covers:
A. * Alveolar walls.
B. Walls of trachea.
C. Walls of larynx.
D. Bronchi.
E. Bronchioles.
875. Because of illness a middle-aged woman had acute fall thin. After some period a pain appeared in a
lumbar area. A doctor diagnosed nephroptosis. Weakening of what fixative factors of kidneys led to
this pathology.
A. * Capsula adiposa.
B. Arteriae et venae renalis.
C. Capsula fibrosa.
D. Perinephrium.
E. Fascia endoabdominalis.
876. Doctors during surgical operation review the organs of the abdominal cavity. On posterior wall of
abdomen ureter was found. What relation of the peritoneum and ureter?
A. * Retroperitoneal position
B. Mesoperitoneal position
C. Peritoneum does not cover the ureter
D. Peritoneum forms mesentery for ureter
E. Intraperitoneal position
877. Doctors during surgical operation review the organs of the abdominal cavity. On posterior wall of
abdomen ureter was found. What relation of the peritoneum and ureter?
A. * Peritoneum does not form mesentery for ureter
B. Mesoperitoneal position

C. Peritoneum does not cover the ureter


D. Peritoneum forms mesentery for ureter
E. Intraperitoneal position
878. Doctors during surgical operation review the organs of the abdominal cavity. On posterior wall of
abdomen ureter was found. What relation of the peritoneum and ureter?
A. * Peritoneum does not cover the ureter
B. Mesoperitoneal position
C. Peritoneum covers the ureter
D. Peritoneum does not form mesentery for ureter
E. Intraperitoneal position
879. Doctors during surgical operation review the organs of the abdominal cavity. On posterior wall of
abdomen ureter was found. What relation of the peritoneum and ureter?
A. * Peritoneum does not cover the ureter
B. Peritoneum forms mesentery for ureter
C. Intraperitoneal position
D. Mesoperitoneal position
E. Extraperitoneal position
880. Doctors during surgical operation review the organs of the abdominal cavity. On posterior wall of
abdomen ureter was found. What relation of the peritoneum and ureter?
A. * Peritoneum does not cover the ureter
B. Peritoneum forms mesentery for ureter
C. Peritoneum cover all surfaces of the ureter
D. Peritoneum cover three surfaces of the ureter
E. Peritoneum covers one surface of the ureter
881. Doctors during surgical operation review the organs of the abdominal cavity. On posterior wall of
abdomen ureter was found. What relation of the peritoneum and ureter?
A. * Ureter is positioned in retroperitoneal space
B. Peritoneum forms mesentery for ureter
C. Intraperitoneal position
D. Mesoperitoneal position
E. Peritoneum cover all surfaces of the ureter
882. Doctors during surgical operation review the organs of the abdominal cavity. On posterior wall of
abdomen to the both sites of vertebrae kidneys was found. What relation of the peritoneum and
kidneys?
A. * Retroperitoneal position
B. Mesoperitoneal position
C. Extraperitoneal position
D. Peritoneum forms mesentery for kidneys
E. Intraperitoneal position
883. Doctors during surgical operation review the organs of the abdominal cavity. On posterior wall of
abdomen to the both sites of vertebrae kidneys was found. What relation of the peritoneum and
kidneys?
A. * Peritoneum does not form mesentery for kidneys
B. Mesoperitoneal position
C. Extraperitoneal position
D. Peritoneum forms mesentery for kidneys
E. Intraperitoneal position

884. Doctors during surgical operation review the organs of the abdominal cavity. On posterior wall of
abdomen to the both sites of vertebrae kidneys was found. What relation of the peritoneum and
kidneys?
A. * Peritoneum does not cover the kidneys
B. Mesoperitoneal position
C. Peritoneum covers the kidneys
D. Peritoneum does not form mesentery for kidneys
E. Intraperitoneal position
885. Doctors during surgical operation review the organs of the abdominal cavity. On posterior wall of
abdomen to the both sites of vertebrae kidneys was found. What relation of the peritoneum and
kidneys?
A. * Retroperitoneal position
B. Peritoneum forms mesentery for kidneys
C. Intraperitoneal position
D. Mesoperitoneal position
E. Peritoneum covers the kidneys
886. Doctors during surgical operation review the organs of the abdominal cavity. On posterior wall of
abdomen to the both sites of vertebrae kidneys was found. What relation of the peritoneum and
kidney?
A. * Peritoneum does not cover the kidney
B. Peritoneum forms mesentery for kidney
C. Peritoneum cover all surfaces of the kidney
D. Peritoneum cover three surfaces of the kidney
E. Peritoneum covers one surface of the kidney
887. Doctors during surgical operation review the organs of the abdominal cavity. On posterior wall of
abdomen to the both sites of vertebrae kidneys was found. What relation of the peritoneum and
kidneys?
A. * Kidneys are positioned in retroperitoneal space
B. Peritoneum forms mesentery for kidneys
C. Intraperitoneal position
D. Mesoperitoneal position
E. Peritoneum covers the kidneys
888. During auscultation (hearing) of lungs of a patient 37 years old the "Vesicular breathing" was
marked. It is normal noise which listened to above the thorax of healthy people. This noise arisen up
in a bronchial tree and passed through a normally functioning alveolar tree. What anatomical
structures do not concern to elements of a bronchial tree?
A. * Respiratory bronchioles.
B. Terminal bronchioles.
C. Lobular bronchi.
D. Lobar bronchi.
E. Segmental bronchi.
889. During the operation a stone 3x4 cm in size and having on one side 3 horn- shaped outgrowths (coral
calculus) was removed from renal pelvis. What anatomic formations do these processes correspond?
A. * Lobules.
B. Nephrons.
C. Greater calices.
D. Renal pelvis.
E. Lesser calices.

890. During the operation on lungs a surgeon removed a clot of blood from a horizontal fissure. What
lobes of lung are separated by this fissure?
A. * Superior and middle lobes of right lung.
B. Superior and inferior lobes of right lung.
C. Superior and inferior lobes of left lung.
D. Inferior and middle lobes of right lung.
E. Inferior lobes of right and left lungs.
891. Excretory urogramma (contrasting of urinoexcretory ways) revealed wide sac-shaped renal pelvis, in
which lesser calices empties, greater calices are absent. Specify the form excretory ways of a kidney.
A. * Embryonic.
B. Fetal.
C. Mature.
D. Phylogenetic.
E. Ontogenetic.
892. In 3 years old patient the lobectomy of middle lobe of right lung was provided. What segments were
damaged?
A. * Lateral and medial.
B. Apical, posterior, anterior.
C. Medial and anterior.
D. Posterior and lateral basal.
E. Posteroapical.
893. In patient left renal infarct was diagnosed. The damaging of parenchyma of kidney was caused a
revascularization in the system of arteries, passing through renal column. Name these arteries...
A. * Segmental.
B. Interlobular.
C. Arcuate.
D. Interlobar.
E. Renal.
894. In patient right renal infarct was diagnosed. The damaging of parenchyma of kidney was caused by
revascularization in the system of segmental artery. How many segmental arteries are present in right
kidney in a norm?
A. * 6.
B. 3.
C. 5.
D. 7.
E. 4
895. In patient the stone in the place of transition of the left renal pelvis to ureter was revealed. What
structure situate anteriorly to the initial part of the left ureter?
A. * Pancreas.
B. Stomach.
C. Ileum.
D. Spleen.
E. Sigmoid colon.
896. In the short-winded patient X-ray exposed an exudation in a pleural cavity. To prevent injuring
intercostal arteries, the puncture of pleural cavity should be provided (taking into account the level of
exudation)
A. * To the upper edge of subjacent rib.
B. On the lower edge of overlying rib.

C. In the middle point between ribs.


D. At the head of rib.
E. In the place of transition of bone part of rib in cartilaginous.
897. Mother of 3-years-old child appealed to the clinic. During conversation doctor clarified that child
played with a small metallic object (a cuff-link) and inserted it in mouth. The child had swallowed or
had inhaled this subject. To find out it was not possible. At X-ray examination the foreign body was
revealed at a level of 6-th thoracic vertebra on a middle line. Where, more probably than all, is there
this foreign body?
A. * In an esophagus.
B. In a trachea.
C. In a pharynx.
D. In the left main bronchus.
E. In the right main bronchus.
898. The auscultation (hearing) of lungs revealed the segment with «bronchial breathing» in a patient 46
years old. Such respiratory noise is not listened at healthy people. The doctor made a conclusion that
in the given segment there was a destruction of an alveolar tree. What anatomical structure doesn't
belong to the alveolar tree?
A. * Intrasegmental bronchi.
B. Alveolar saccules.
C. Alveolar ducts.
D. Alveola.
E. Respiratory bronchioles.
899. The left-sided segmental pneumonia is diagnosed at the patient. An additional respiratory noise
(crepitation) is listened at the left side from the back surface, on the level of VII-X ribs. In what
segment of lungs is the pathological process locates?
A. * Posterior basal.
B. Medial basal.
C. Lateral basal.
D. Anterior basal.
E. Inferior uvular.
900. The parents of new-born child appealed to pediatrics with complaints of excretions a liquid (urines)
in the area of navel. What congenital abnormality did the child get?
A. * Umbilical cyst.
B. Non-closed urachus.
C. Splintering urethra.
D. Meckel divertikulum.
E. Inguinal cyst.
901. The patient appealed to the surgery with a tumor in lower third of esophagus. What serous layer of
thoracic cavity can be damaged during the operation in this area?
A. * Right mediastinal pleura.
B. Left mediastinal pleura.
C. Right costal pleura.
D. Diaphragmatic pleura.
E. Left costal pleura.
902. The patient with knife wound of thorax on the right and pneumothorax (entering of air in a pleural
cavity) was delivered to surgical department. By percussion was found the lower margin of right lung
on a midclavicular line lifted up to level of the III rib. Where in a norm must it be?
A. * VI rib.

B. VII rib.
C. VIII rib.
D. IX rib.
E. V rib.
903. To a patient 60 years old the lobectomy of superior lobe of right lung was provided. What segments
were damaged?
A. * Apical, posterior, anterior.
B. Lateral, middle, superior.
C. Middle, basal and lateral basal.
D. Posterior basal, superior and inferior lingual.
E. Posteroapical.
904. To patient with diagnose a cancer of left lung was performed an operation pneumonectomy
(removing of the lung). One of stages of the operation is ligation and cutting a root of the lung, which
includes arteries, veins and bronchus. In what order are these structures disposed in a root of left lung
from up to down?
A. * Pulmonary artery, main bronchus, pulmonary veins.
B. Pulmonary artery, pulmonary veins, main bronchus.
C. Main bronchus, pulmonary artery, pulmonary veins.
D. Main bronchus, pulmonary veins, pulmonary artery.
E. Pulmonary veins, main bronchus, pulmonary artery.
905. To the patient 50 years old the lobectomy of superior lobe of right lung was provided. How many
segments were removed?
A. * 3.
B. 5.
C. 2.
D. 4.
E. O.
906. A 65 years old man appealed to doctor with acute retention of urine. The examination revealed an
obstruction of urethra, which was caused by pathology of organ surrounding it. What is this organ?
A. * Prostate gland.
B. Spermatic cord.
C. Testicle.
D. Seminal vesicles.
E. Epidydimis.
907. A man 40 years old appealed a doctor with complaints of the presence of painful tumor in area of
scrotum. A doctor diagnosed is hydrocele of testicle. Be¬tween what suits of testicle does a liquid
accumulate at this disease?
A. * Between the parietal and visceral lamina of vaginal tunic.
B. Between a skin and tunica dartos.
C. Between a tunica dartos and external seminal fascia.
D. Between external seminal fascia and fascia of cremaster muscle.
E. Between internal seminal fascia and vaginal tunic.
908. A man 75 years old entered urology department with complaints of the expressed pain at the
hypogastric region, absence of urine, impossibility of urination. At examination an urologist revealed
a diagnosis\: adenoma (benign tumor of prostate gland). The catheterization of urinary bladder was
indicated. What is the sequence of passing of catheter through the parts of urethra?
A. * Spongy, membranous, prostatic.
B. Spongy, prostatic, membranous.

C. Membranous, spongy, prostatic.


D. Prostatic, membranous, spongy.
E. Membranous, prostatic, spongy.
909. A man has a tumor of posterior wall of urinary bladder. What organs can be engaged in a process?
A. * Rectum, seminal vesicles, ampule of ejaculatory duct.
B. Prostate gland.
C. Rectum, prostate gland.
D. Male urethra.
E. Cowper's (bulbourethral) glands.
910. A patient 20 years old appealed to the policlinic with complaints of burning and purulent excretions
during urination. For confirmation of diagnosis the bacterial inoculation from a urethra is appointed.
Doctor doing this inoculation, takes mucus from a navicular fossa. In what part of urethra is this fossa
located?
A. * Spongy.
B. Cavernous.
C. Prostatic.
D. Membranous.
E. Bulbar.
911. A patient 35 years old entered obstetric department with the symptoms of "acute abdomen" and
suspicion on extrauterine pregnancy. Where will blood accumulate at the rupture of salpinx?
A. * In a rectouterine cavity.
B. In a vesicouterine cavity.
C. In a rectovesical cavity.
D. In a right lateral canal.
E. In the left lateral canal.
912. A patient 65 years old was hospitalized with suspicion on the tumor of prostate. During an operation
it is revealed, that the tumor «germinated» to a urinary bladder. What department of urinary bladder
suffered?
A. * Neck.
B. Apex.
C. Fundus.
D. Body.
E. Triangle.
913. A patient appealed to the doctor with a complaint of sharp attacks of a pain in a right lumbar area.
The examination revealed obstruction of right ureter by stone between its abdominal and pelvic parts.
What is anatomical border between these two parts?
A. * Linea terminalis.
B. Linea semilunaris.
C. Linea arcuata.
D. Linea transversa.
E. Linea inguinalis.
914. A patient has inflammation of bulbourethral glands. Between what perineal fascia are these organs?
A. * Fasciae diaphragmatis urogenitalis superior et inferior.
B. Fasciae perinei superficialis et diaphragmatic urogenitalis inferior.
C. Fasciae diaphragmatis urogenitalis superior et diaphragmatis pelvis inferior.
D. Fasciae diaphragmatis pelvis inferior et superior.
E. Fasciae perinei superficialis et diaphragmatis pelvis inferior.

915. A patient was hospitalized with suspicion on the tumor of prostate. During an operation it was
exposed, that a tumor «germinated» to the middle department of urethra. What is it a department?
A. * Membranous.
B. Prostatic.
C. Spongy.
D. Cavernous.
E. Lacunar.
916. A young man appealed to the hospital with complaints of disturbances of urination. It was revealed,
that a urethra is split above and urine flows out through this opening. What type of anomaly of
development is observed in this case?
A. * Epispadiya.
B. Phimosis.
C. Hermaphroditism.
D. Paraphimosis.
E. Hypospadiya.
917. After trauma of testicle and hemorrhage the coiled seminal tubuli in a parenchyma are damaged.
What function of testicle is destroyed?
A. * Spermatogenesis.
B. Erection of penis.
C. Excretion of sperms.
D. Production of liquid part of sperm.
E. All mentioned.
918. An urolithiasis was complicated by the exit of concrement from the kidney. On what level of ureter
can it stop most probably?
A. * On the border between abdominal and pelvic parts.
B. In a renal pelvis.
C. In middle abdominal part.
D. 2 cm superiorly to the inflow to a urinary bladder.
E. 5 cm superiorly to pelvic part.
919. At a woman extrauterine pregnancy was found out. In what organ did the impregnation of ovule and
its development happen?
A. * In a salpinx.
B. In an ovary.
C. In the body of uterus.
D. In the neck of uterus.
E. Vagina.
920. At a woman the tumor of ovary was found out. An operation is indicated. What ligament must a
surgeon cut to separate an ovary from uterus?
A. * Lig. ovarium proprium.
B. Lig. suspensorium ovarium.
C. Lig. umbilicalis medialis.
D. Lig. umbilicalis lateralis.
E. Rounded ligament of uterus.
921. At examination of a newborn boy a surgeon exposed absence of the left testicle in a scrotum
(monorchizm). In what age must a testicle be in a scrotum in norm?
A. * To moment of birth.
B. Till one year.
C. Till three years.

D. Till five years.


E. Till seven years.
922. During a childbirth at difficult exiting of head of child to avoid the perineal rupture they usually
make the section of vagina opening at base of labia majora. What perineal muscle is dissected here?
A. * M. bulbospongiosus.
B. M. ischiocavernosus.
C. M. transversus perinei superficialis.
D. M. sphincter ani externus.
E. M. transversus perinei profundus.
923. During a cystoscopy a doctor revealed changes of mucous membrane of urinary bladder in the area of
«triangle». I n what part of urinary bladder is this triangle?
A. * Fundus.
B. Isthmus.
C. Apex.
D. Collumn.
E. Body.
924. During examination in a newborn boy was found that the orifice of urethra is open on the lower
surface of penis. What anomaly of development is it?
A. * Hypospady.
B. Hermaphroditism.
C. Epispady.
D. Monorchism.
E. Cryptorchism.
925. During operation of deleting of uterus with ovaries and ovarian tubes a doctor bandages Ligaments
which suspend ovaries. What vessels were bandaged by a doctor in this ligament?
A. * Ovarian artery and vein.
B. Uterine artery and vein.
C. Tubular artery and vein.
D. Internal iliac artery.
E. Internal iliac vein.
926. During operation on an ovary vessels were bandaged in area of hilus. In what department of ovary
does conduct manipulation?
A. * Margo mesovaricus.
B. Margo liber.
C. Facies medialis.
D. Facies lateralis.
E. Extremitas uterinus.
927. During the catheterization of male urethra a mucous membrane was injured. In what part of urethra is
the damage of it most possible?
A. * Pars membranacea.
B. Pars spongiosa.
C. Pars cavernosa.
D. Pars prostatica.
E. Pars bulbaris.
928. During the removing of right ovary a surgeon instead of ovarian artery bandaged a near located
organ. Which one?
A. * Ureter.
B. Thoracic duct.

C. Aorta.
D. Common iliac artery.
E. Internal iliac artery.
929. For a man 40 years inflammation of testicle was complicated by hydrocele. Operative treatment is
needed. What from the suits of testicle does a surgeon cut the last during an operation?
A. * Lamina parietalis tunicae vaginalis testis.
B. Tunica dartos.
C. Fascia spermatica interna.
D. Musculus cremaster.
E. Fascia spermatica externa.
930. For a newborn boy at examination in a right half of scrotum a testicle it is not found. What anomaly
of development does speech go about?
A. * Monorchizm.
B. Hermaphroditism.
C. Hydrocele.
D. Hypospadia.
E. Epispadia.
931. For the boy of 2 years a scrotal hernia is diagnosed. Abnormality of what testicular membrane does
lead to this pathology?
A. * Tunica vaginalis testis.
B. Fascia cremasterica.
C. Fascia spermatica externa.
D. Tunica dartos.
E. Fascia spermatica interna.
932. For the patient of 53 years old after a wound of perineal region involuntary urination is marked. What
of muscles is damaged?
A. * M. sphincter urethrae.
B. M. ischiocavernosus.
C. M. bulbospongiosus.
D. M. transversus perinei superficialis.
E. M. transversus perinei profundus.
933. In patient the presence of stone in the place of transition of right renal pelvis to the ureter was
revealed. Behind what structure does the initial part of the right ureter situate?
A. * Pars descendens duodeni.
B. Flexura coli dextra.
C. Colon ascendens.
D. Pars superior duodeni.
E. Pars horizontalis duodeni.
934. It is possible to perform a puncture of urinary bladder through anterior abdominal wall, not affecting
a peritoneum. When?
A. * When the bladder is full (stretched), when it is covered mesoperitoneally.
B. When the bladder is empty.
C. Only in male.
D. Only in female.
E. Only in children.
935. Several times after the trauma of perineal region a patient have the phenomena of impotence. What
muscle was damaged?
A. * M. ischiocavernosus.

B. M. sphincter urethrae internus.


C. M. sphincter urethrae externus.
D. M. bulbospongiosus.
E. M. levator ani.
936. Ultrasonic research of organs of lesser pelvis is carried out at a full urinary bladder. What does
perineal muscle retain the urine?
A. * M. sphincter urethrae.
B. M. bulbospongiosus.
C. M. ischiocavernosus.
D. M. transversus perinei superficialis.
E. M. transversus perinei profundus.
937. ?Position of the most joints of baby in the fetal position is
A. abducted
B. adducted
C. circumducted
D. extended
E. * flexed
938. Position of backbone if the person who is standing and bends to tie the shoe is ______ of the trunk.
A. abducting
B. adducting
C. circumduction
D. extending
E. * flexing
939. Kicking a football straight ahead with your toes involves the movement of your lower leg
A. abduction
B. adduction
C. circumduction
D. * extension
E. flexion
940. Lateral movement of the body part away from the main axis of the body, or away from the
midsagittal plane, is the definition of:
A. * Abduction
B. Flexion
C. Extension
D. Circumduction
E. Adduction
941. Shrugging your shoulders to indicate somebody ‘doesn't know’ the answer to a question involves
A. depression of the scapulae
B. lateral excursion of the scapulae
C. * elevation of the scapulae
D. protraction of the scapulae
E. retraction of the scapulae
942. Traumatologist examines the patient with backbone pain. The anterior part of the vertebra is:
A. * Body
B. Arcus
C. Foramen
D. Superior incisura

E. Inferior incisura
943. Meningitis was diagnosed in 5-year-old child. Where the spinal puncture should be performed?
A. * L2-L3
B. L4-L5
C. L3-L4
D. L1-L2
E. L12-L1
944. Traumatologist examines the patient with backbone pain. The rear part of the vertebra is:
A. * Arcus
B. Body
C. Foramen
D. Superior incisura
E. Inferior incisura
945. The spinal cord located in:
A. * Vertebral canal
B. Sacral canal
C. Intervertebral foramen
D. Superior incisura
E. Inferior incisura
946. Traumatologist examines the patient with backbone pain. The spinal nerves pass through:
A. * Intervertebral foramen
B. Sacral canal
C. Vertebral canal
D. Superior incisura
E. Inferior incisura
947. Tilting the head backward to look up at the sky requires
A. * extension of the neck
B. adduction of the neck
C. circumduction of the neck
D. abduction of the neck
E. flexion of the neck
948. To walk on her toes, a ballerina must be able to ______ her feet for long periods of time
A. * plantarflex
B. dorsiflex
C. evert
D. invert (supinate)
E. abduct
949. Traumatologist examines the patient with backbone pain. How many cervical vertebrae are there?
A. * 7
B. 5
C. 8
D. 10
E. 12
950. When a suspect is arrested by the police, typically they are handcuffed behind their back. In this
position, the suspect's arms are _____ in shoulder joints
A. * extended

B. bended
C. circumducted
D. abducted
E. flexed
951. When you are going to place something in the palm of your hand, your hand must be
A. * supinated
B. adducted
C. flexed
D. abducted pronated
952. Which process of the vertebrae take place in formation of joints with ribs?
A. * Transverse
B. Spinous
C. Articular
D. Mammilary
E. Accessory
953. You are sitting on a chair. To stand up, you must ______ your thighs and your legs
A. * extend
B. abduct
C. adduct
D. rotate
E. flex
954. Which canal can be damaged in case of the trauma of occipital bone?
A. * hypoglossal
B. musculotubarius
C. tympanic
D. carotid
E. facial
955. Which canal can be damaged in case of the trauma of sphenoid bone?
A. * pterygoid
B. hypoglossal
C. tympanic
D. carotid
E. facial
956. Which canal can be damaged in case of the trauma of sphenoid bone?
A. * optic
B. hypoglossal
C. greater tympanic
D. carotid
E. facial
957. Which canal passes in occipital bone?
A. * hypoglossal
B. musculotubarius
C. tympanic
D. carotid
E. facial
958. Which canal does not exist?

A. * sphenoid
B. musculotubarius
C. tympanic
D. carotid
E. facial
959. Which canal belongs to sphenoid bone?
A. * optic
B. musculotubarius
C. tympanic
D. carotid
E. facial
960. Which canal is situated in sphenoid bone?
A. * pterygoid
B. musculotubarius
C. tympanic
D. carotid
E. facial
961. Which canal is situated in occipital bone?
A. * hypoglossal
B. musculotubarius
C. tympanic
D. carotid
E. facial
962. A man got a the trauma of head. The fracture of sphenoid bone is diagnosed in the area of basis of
pterygoid process. The contents of which canal can be damaged?
A. * pterygoid
B. musculotubarius
C. tympanic
D. carotid
E. facial
963. The fracture of temporal bone is diagnosed in the area of fossula petrosa. Which canal’s content can
be damaged?
A. * tympanic
B. musculotubarius
C. mastoid
D. carotid
E. facial
964. The tumor of hypophysis causes the destruction of sella turcica. Which cavity can be damaged?
A. the facial canal
B. the carotid canal
C. the optic canal
D. the tympanic cavity
E. * sinuses of sphenoid bone
965. The tumor of middle nasal concha causes the destruction and increasing of sinus. Which cavity can
be damaged?
A. the facial canal
B. the carotid canal

C. * the anterior ethmoidal cells


D. the tympanic cavity
E. the sphenoid sinus
966. The tumor of middle nasal concha causes the destruction and increasing of sinus. Which cavity can
be damaged?
A. the facial canal
B. the carotid canal
C. * the middle ethmoidal cells
D. the tympanic cavity
E. the sphenoid sinus
967. The tumor of superior nasal concha causes the destruction and increasing of sinus. Which cavity can
be damaged?
A. the facial canal
B. the carotid canal
C. * the posterior ethmoidal cells
D. the tympanic cavity
E. the maxillary sinus
968. The tumor of superior nasal concha causes the destruction and increasing of sinus. Which cavity can
be damaged?
A. the facial canal
B. the carotid canal
C. the anterior ethmoidal cells
D. the tympanic cavity
E. * the sphenoid sinus
969. The tumor of the frontal lobes of the brain causes the destruction and increasing of some parts of
bones. Which cavity can be damaged?
A. the infraorbital canal
B. the facial canal
C. foramen magnum
D. sinuses of sphenoid bone
E. * sinuses of frontal bone
970. Inflammation of tympanic cavity (festering otitis) in a patient was complicated by inflammation of
cells of the mastoid process. Which wall of the tympanic cavity can pus get to the cells?
A. lateral
B. superior
C. * posterior
D. anterior
E. medial
971. A child 3 years old entered the clinic with a diagnosis "festering otitis". Where pus can pass from a
tympanic cavity through its rear wall?
A. In an external auditory meatus
B. In posterior cranial fossa
C. * In a mastoid antrum
D. In an internal ear
E. In an auditory tube
972. A child 3 years old entered the clinic with a diagnosis "festering otitis". Where pus can pass from a
tympanic cavity through its lateral wall?
A. * In an external auditory meatus

B. In posterior cranial fossa


C. In a mastoid antrum
D. In an internal ear
E. In an auditory tube
973. A child 3 years old entered the clinic with a diagnosis "festering otitis". Where pus can pass from a
tympanic cavity through its anterior wall?
A. In an external auditory meatus
B. In posterior cranial fossa
C. In a mastoid antrum
D. In an internal ear
E. * In an auditory tube
974. A child 3 years old entered the clinic with a diagnosis "festering otitis". Where pus can pass from a
tympanic cavity through its medial wall?
A. In an external auditory meatus
B. In posterior cranial fossa
C. In a mastoid antrum
D. * In an internal ear
E. In an auditory tube
975. A child 3 years old entered the clinic with a diagnosis "festering otitis". Where pus can pass from a
tympanic cavity through its anterior wall?
A. In an external auditory meatus
B. In posterior cranial fossa
C. In a mastoid antrum
D. In an internal ear
E. * In an auditory tube
976. A child 5 years old entered the hospital with a diagnosis "festering inflammation of middle ear
(tympanitis)". The disease was caused by inflammation of nasopharynx. What canal of temporal bone
can be infected from tympanic cavity?
A. tympanic canaliculus
B. carotid canal
C. canaliculus of chorda tympani
D. carotico-tympanic canaliculi
E. * musculotubarius canal
977. A patient got the trauma of occipital condyle. Which canal or canaliculus content can be damaged?
A. sphenoid
B. musculotubarius
C. mastoid
D. * hypoglossal
E. facial
978. A patient got the injury of the clivus. Which bone can be damaged?
A. * Occipital bone
B. Temporal bone
C. Zygomatic bone
D. Vomer
E. Maxilla
979. Executing the physical exercise, a girl fell down on the knee. The next day she began to feel a sharp
pain of the knee and hemorrhage in lower part of the knee. Which bone can be damaged?

A. the sciatic bone


B. * the patella
C. the coccyx bone
D. popliteal surface
E. the pubic bone
980. Movement of the arm to the trunk around sagittal axis is:
A. Extension
B. Abduction
C. * Adduction
D. Circumduction
E. Flexion
981. Movement of the arm when we rotate it medially around the vertical axis is:
A. Extension
B. Abduction
C. Adduction
D. * Pronation
E. Flexion
982. Movement of the arm when we rotate it laterally around the vertical axis is:
A. Extension
B. Abduction
C. Adduction
D. * Supination
E. Flexion
983. Movement of the arm when we lift it anteriorly around the frontal axis is:
A. Extension
B. Abduction
C. Adduction
D. Supination
E. * Flexion
984. Movement of the arm when we move it posteriorly around the frontal axis is:
A. * Extension
B. Abduction
C. Adduction
D. Supination
E. Flexion
985. Movement of the arm when we move it in different directions in such order – to the front, laterally,
posteriorly and medially is termed as
A. Rotation
B. * Circumduction
C. Adduction
D. Supination
E. Flexion
986. A person who is standing in the anatomic position holds the hands and forearms
A. pronated, flexed
B. pronated, extended
C. supinated, flexed

D. * supinated, extended
E. abducted, extended
987. A 45 years old man addressed the traumatologist after the industrial trauma of the back. The function
of extension of the back is absent. Which ligament covers back surface of vertebral bodies?
A. Lig. longitudinale anterius
B. * Lig. longitudinale posterius
C. Lig. flava
D. Lig. interspinalia
E. Lig. Supraspinale
988. A ligament formed predominantly by elastic fibers which connect the arcs of adjacent vertebrae.
Which ligament was injured?
A. interspinous ligament
B. * ligamenta flava
C. posterior longitudinal ligament
D. anterior longitudinal ligament
E. supraspinous ligament
989. A patient who suffered in a car the fracture of the left scapula was diagnosed. Which articulation
connects the scapula to the adjacent bone?
A. articul. sternocostalis
B. articul. costotransversaria
C. * articul. glenohumeralis
D. articul. intervertebralis
E. articul. capitis costae
990. Which ligament connects the tips of the spinous processes of thoracic and lumbar vertebrae, a
syndesmosis; begins at the C7 vertebrae and ends at the sacral segmental level?
A. posterior longitudinal ligament
B. nuchal ligament
C. * supraspinous ligament
D. interspinous ligament
E. anterior longitudinal ligament
991. Which ligament passes downward along the posterior surfaces of all vertebral bodies?
A. * posterior longitudinal ligament
B. supraspinous ligament
C. interspinous ligament
D. anterior longitudinal ligament
E. nuchal ligament
992. A patient who suffered in a car the fracture of the left scapula was diagnosed. Which articulation
connects the scapula to the adjacent bone?
A. articul. sternocostalis
B. * articul. acromioclavicularis
C. articul. costotransversaria
D. articul. intervertebralis
E. articul. capitis costae
993. A patient who suffered in a car the fracture of the left clavicle was diagnosed. Which articulation
connects the clavicle to the adjacent bone?
A. articul. sternocostalis
B. * articul. sternoclavicularis

C. articul. costotransversaria
D. articul. intervertebralis
E. articul. capitis costae
994. A patient who suffered in a car the fracture of the left clavicle was diagnosed. Which articulation
connects the clavicle to the adjacent bone?
A. articul. sternocostalis
B. * articul. acromioclavicularis
C. articul. costotransversaria
D. articul. intervertebralis
E. articul. capitis costae
995. Movement of the forearm around the vertical axis, turning the forearm to put the dorsal surface of the
hand anteriorly is the definition of:
A. Extension
B. Abduction
C. Adduction
D. * Pronation
E. Flexion
996. Movement of the forearm around the vertical axis, turning the forearm to put the palmar surface of
the hand anteriorly is the definition of:
A. Extension
B. Abduction
C. Adduction
D. * Supination
E. Flexion
997. Injury of the sternocostal joint is diagnosed during medical examination of the chest after the car
accident. Which of the listed below structures belong to the sternocostal joint?
A. * lig. sternocostalis radiatum
B. lig. capitis costae radiatum
C. lig. capitis costae intraarticulare
D. lig. transversocostalis
E. lig. coracoacromiale
998. Damage of the ligament that extends along the spinous processes of cervical vertebrae and extends
from the base of the skull to 7th cervical vertebra is diagnosed during medical examination after the
car accident. Which ligament can be injured?
A. * nuchal ligament
B. interspinous ligament
C. posterior longitudinal ligament
D. anterior longitudinal ligament
E. supraspinous ligament
999. Damage of the ligament that extends downward along the posterior surfaces of vertebral bodies is
diagnosed during medical examination after the car accident. Which ligament can be injured?
A. * posterior longitudinal ligament
B. supraspinous ligament
C. interspinous ligament
D. anterior longitudinal ligament
E. nuchal ligament

1000 Damage of the ligament that connects the tips of the spinous processes of thoracic and lumbar
vertebrae, begins at the C7 vertebrae and ends at the mid sacral level is diagnosed during medical
examination after the car accident. Which ligament is injured?
A. posterior longitudinal ligament
B. * supraspinous ligament
C. interspinous ligament
D. anterior longitudinal ligament
E. nuchal ligament
1001 Damage of the ligament that connects the spinous processes near arcs of thoracic and lumbar
vertebrae is diagnosed during medical examination after the car accident. Which ligament was
injured?
A. posterior longitudinal ligament
B. nuchal ligament
C. supraspinous ligament
D. * interspinous ligament
E. anterior longitudinal ligament
1002 Which parts of the upper region of the abdomen are divided into?
A. * Regiones hypochondriacae dextra et sinistra, regio epigastrica
B. Regio umbilicalis, regiones hypochondriacae dextra et sinistra
C. Regiones laterales dextra et sinistra, regio umbilicalis
D. Regiones hypochondriacae dextra et sinistra, regio umbilicalis
E. Regiones laterales dextra et sinistra, regio epigastrica
1003 The doctor observes the diaphragm. What does pass through the muscular part of the diaphragm?
A. * The vagus nerve
B. The inferior vena cava
C. The inferior phrenic arteries
D. The superior phrenic arteries
E. The celiac trunk
1004 The doctor observes the diaphragm. What does pass through the muscular part of the diaphragm?
A. The inferior vena cava
B. * The esophagus
C. The inferior phrenic arteries
D. The superior phrenic arteries
E. The celiac trunk
1005 The doctor observes the diaphragm. What does pass through the muscular part of the diaphragm?
A. The vagus nerve
B. The inferior vena cava
C. The inferior phrenic arteries
D. * The aorta
E. The celiac trunk
1006 The doctor examines anterior abdominal wall. Which walls form the inguinal canal?
A. * Anterior, posterior, superior, inferior
B. Superficial, posterior, superior, inferior
C. External, deep, posterior, superior
D. Internal, posterior, superior, inferior
E. Deep, external, posterior, inferior

1007 The doctor examines anterior abdominal wall. Which parts are situated is the lower region of the
abdomen?
A. Regiones hypochondriacae dextra et sinistra, regio epigastrica
B. Regio umbilicalis, regiones hypochondriacae dextra et sinistra
C. * Regiones inguinales dextra et sinistra, regio pubica
D. Regiones hypochondriacae dextra et sinistra, regio umbilicalis
E. Regiones laterales dextra et sinistra, regio epigastrica
1008 The doctor examines anterior abdominal wall. Which parts are situated is the middle area of the
abdomen?
A. Regiones hypochondriacae dextra et sinistra, regio umbilicalis
B. Regio umbilicalis, regiones hypochondriacae dextra et sinistra
C. Regiones hypochondriacae dextra et sinistra, regio epigastrica
D. * Regiones laterales dextra et sinistra, regio umbilicalis
E. Regiones laterales dextra et sinistra, regio epigastrica
1009 A patient who suffered in a car accident was examined for fracture of the ribs. What muscles raise the
ribs?
A. The subcostales muscles
B. The subclavius muscles
C. * The external intercostal muscles
D. The internal intercostal muscles
E. The transversus thoracis muscle
1010 A patient who suffered in a car accident was examined for fracture of the lower ribs. Which muscle
pulls the ribs down?
A. * The transversus thoracis muscle
B. The subclavius muscles
C. The external intercostal muscles
D. The serratus anterior muscles
E. The pectoralis minor muscle
1011 A patient who was suffered in a car accident was examined for fracture of the left scapula. Which
muscle can move the scapula to the front and laterally?
A. The internal intercostal muscles
B. * The serratus anterior muscle
C. The subclavius muscle
D. The transversus thoracis muscle
E. The subcostales muscles
1012 A patient who suffered in a car accident was examined for fracture of the left clavicle. Which muscle
moves the clavicle downwards and towards the front?
A. The pectoralis minor muscle
B. * The subclavius muscle
C. The pectoralis major muscle
D. The external intercostal muscles
E. The serratus anterior muscle
1013 A patient who suffered in a car accident was examined for fracture of the left scapula. Which muscle
draws the scapula inferiorly?
A. The subclavius muscle
B. * The pectoralis minor muscle
C. The pectoralis major muscle
D. The serratus posterior inferior muscle

E. The external intercostal muscle


1014 The doctor examines the abdominal wall. Which muscle pulls the lower ribs downward and to the
front and consists of horizontal fibres?
A. The internal oblique muscle of the abdomen
B. * The transverse abdominal muscle
C. The external oblique muscle of the abdomen
D. The pyramidalis muscle
E. The rectus abdominis muscle
1015 The doctor examines the abdominal wall. Which muscle carries the tendinous intersections?
A. The internal oblique muscle of the abdomen
B. The transverse abdominal muscle
C. The external oblique muscle of the abdomen
D. The pyramidalis muscle
E. * The rectus abdominis muscle
1016 The doctor examines the abdominal wall. Which muscle has a most important function as flexing the
lumbar spine?
A. The internal oblique muscle of the abdomen
B. The transverse abdominal muscle
C. The external oblique muscle of the abdomen
D. The pyramidalis muscle
E. * The rectus abdominis muscle
1017 The doctor examines the abdominal wall. Which muscle rotates the backbone to the opposite side
(right muscle rotates to the left side) at unilateral contraction?
A. * The external oblique abdominis muscle
B. The transverse abdominal muscle
C. The internal abdominis oblique muscle
D. The pyramidalis muscle
E. The rectus abdominis muscle
1018 The doctor examines the abdominal wall. Which muscle rotates the backbone to the same side (right
muscle rotates to the right side) at unilateral contraction?
A. The external oblique abdominis muscle
B. The transverse abdominal muscle
C. * The internal abdominis oblique muscle
D. The pyramidalis muscle
E. The rectus abdominis muscle
1019 The doctor examines the abdominal wall. Which muscle stretches the linea alba of the abdomen?
A. The internal oblique muscle of the abdomen
B. * The pyramidalis muscle
C. The external oblique muscle of the abdomen
D. The transverse abdominal muscle
E. The rectus abdominis muscle
1020 Physician examines the abdominal wall. Which muscle participates in flexion of the femur in the
coxal joint?
A. The psoas minor muscle
B. * The psoas major muscle
C. The quadratus lumborum muscle
D. The internal intercostal muscles

E. The phrenic muscle


1021 A 30-year-old patient feels a sharp pain when he pulls the lower jaw to the posterior position. The
dentist diagnosed the inflammation of one of the masticatory muscle. Which one is damaged?
A. * musculus temporalis (posterior fibers)
B. musculus temporalis (anterior fibers)
C. medial pterygoid muscle
D. lateral pterygoid muscle
E. masseter
1022 A child of 5 years old was delivered to the infections department of the hospital with the diagnosis of
diphtheria. In which triangle of the neck is it possible to find the trachea?
A. * omotracheal
B. omotrapezoideum
C. carotic
D. omoclavicular
E. submandibulare
1023 A patient with a wound of soft tissues of the neck and the damage of the platysma muscle was
delivered to the hospital. Which neck fascia covers this muscle?
A. * fascia colli superficialis
B. fascia prevertebralis
C. fascia endocervicalis
D. lamina profunda fasciae colli propriae
E. lamina superficialis fasciae colli propriae
1024 A patient with severe intoxication was admitted to the hospital. To provide treatment it is necessary to
make injection into subclavian vein. In what area is it located?
A. * spatuim anterscalenum
B. trigonum omotrapezoideum
C. spatuim interscalenum
D. spatium retrosternocleidomastoideus
E. spatuim interaponeuroticum suprasternale
1025 During the dissection was discovered pus in space between superficial and deep layers of proper
cervical fascia. What is the term of this space?
A. * spatium interaponeuroticum suprasternale
B. previsceral space
C. retrovisceral space
D. spatium interscalenium
E. spatium anterscalenium
1026 During the dissection was discovered pus in space between pharynx and prevertebral muscles. What
is the name of this space?
A. * retrovisceral space
B. previsceral space
C. spatium interaponeuroticum suprasternale
D. spatium interscalenium
E. spatium anterscalenium
1027 During tracheotomy arcus venosus juguli was observed in a 45 year old patient who was admitted to
the hospital with the laryngeal edema. Where is that arch located?
A. * interaponeuroticum space
B. Antercalenium space
C. pretracheale space

D. Retropharyngeal space
E. Interscalenum space
1028 Patient feels a sharp pain in masticatory muscles when he compresses the molars (chewing teeth).
Which muscle develops maximal power for chewing?
A. * masseter muscle
B. musculus temporalis (posterior fibers)
C. musculus temporalis (anterior fibers)
D. medial pterygoid muscle
E. lateral pterygoid
1029 Patient feels a sharp pain when he compresses the incisors (front teeth). Which muscle develops
maximal power for biting?
A. * musculus temporalis (anterior fibers)
B. musculus temporalis (posterior fibers)
C. medial pterygoid muscle
D. lateral pterygoid muscle
E. masseter
1030 Patient with damaged sternocleidomastoid muscle was delivered to the hospital. Which fascia covers
this muscle?
A. * lamina superficialis fasciae colli propriae
B. fascia prevertebralis
C. fascia endocervicalis
D. lamina profunda fasciae colli propriae
E. fascia colli superficialis
1031 To stop the bleeding it is necessary to press the subclavian artery. In what area is it located?
A. * spatuim interscalenum
B. trigonum omotrapezoideum
C. spatuim anterscalenum
D. spatuim retrosternocleidomastoideus
E. spatuim interaponeuroticum suprasternale
1032 A 45 year old man addressed the traumatologist after the industrial trauma of the shoulder. The
functions of abduction of the arm is absent. What muscle was injured?
A. * m. supraspinatus
B. subscapular muscle
C. m. infraspinatus
D. m. teres major
E. m. teres minor
1033 A child of 7 years can not lift the forearm to the shoulder. Which muscle’s active function is absent?
A. * musculus biceps brachii
B. musculus teres major
C. musculus pectoralis major
D. musculus teres minor
E. musculus deltoideus
1034 A patient can not abduct the arm. Which muscle’s active function is absent?
A. * musculus deltoideus
B. musculus teres major
C. musculus pectoralis major
D. musculus biceps brachii
E. musculus teres minor
1035 After falling from the height a man damaged his anterior surface of arm. During the examination the
surgeon diagnosed the rupture of the biceps brachii. What functions of the upper extremity can be
disturbed by the trauma?
A. * flexion of the arm and of the forearm
B. flexion of the arm and hand
C. flexion of the arm, forearm, and hand
D. flexion of the arm
E. flexion and pronation of the forearm
1036 After trauma a patient cannot extend arm in the elbow joint. What muscle was injured?
A. * m. triceps brachii
B. m. brachioradialis
C. m. pronator major
D. m. subscapularis
E. m. infraspiratus
1037 Extrinsic (long fexors and extensors) hand muscles are found in the
A. * forearm.
B. arm.
C. hand.
D. fingers.
E. head.
1038 If a woman raises her arms laterally till the level with her shoulders, she mostly uses her
A. * deltoid muscles.
B. biceps brachii muscles.
C. latissimus dorsi muscles.
D. pectoralis major muscles.
E. all of these
1039 The orchestra conductor is not able to abduct the arm. Which muscle does not operate?
A. * musculus deltoideus
B. musculus trapezius
C. musculus rhomboideus
D. musculus latissimus dorsi
E. musculus pectoralis major
1040 Which muscle tendon can be injured after fracture of the radius bone?
A. * m. biceps brachii
B. M. deltoideus
C. m. teres major
D. m. latissimus dorsi
E. m. pectoralis major
1041 A patient can not flex the toes. What muscle is injured?
A. * m. flexor digitorum longus
B. m. extensor digitorum brevis
C. m. extensor hallucis longus
D. m. tibialis anterior
E. m. tibialis posterior
1042 A patient can not make flexion of the toes. What muscle can be injured?
A. * m. quadratus plantae
B. m. peroneus brevis
C. m. peroneus longus
D. m. tibialis anterior
E. m. tibialis posterior
1043 A patient can not make flexion of the toes. What muscle can be injured?
A. * m. quadratus plantae
B. m. peroneus brevis
C. m. peroneus longus
D. m. tibialis anterior
E. m. tibialis posterior
1044 A patient can not to extend the foot. What muscle is injured?
A. * m. tibialis anterior
B. m. flexor digitorum longus
C. m. peroneus brevis
D. m. peroneus longus
E. m. tibialis posterior
1045 A patient can not to lift lateral margin of the foot. What muscle is injured?
A. * m. peroneus brevis
B. m. extensor digitorum longus
C. m. triceps surae
D. m. tibialis posterior
E. m. tibialis anterior
1046 A patient can not to lift medial margin of the foot. What muscle is injured?
A. * m. tibialis anterior
B. m. quadratus plantae
C. m. extensor digitorum brevis
D. m. peroneus brevis
E. m. peroneus longus
1047 A patient can not to make flexion of the foot. What muscle is injured?
A. * m. tibialis posterior
B. m. extensor digitorum longus
C. m. extensor digitorum brevis
D. m. extensor hallucis longus
E. m. tibialis anterior
1048 After the trauma a patient cannot stand on tiptoes. Which muscle is damaged?
A. * m. triceps surae
B. m. fibularis longus
C. m. extensor hallucis longus
D. m. tibialis anterior
E. m. tibialis posterior
1049 Doing his morning exercise a fifteen – year boy felt a sharp pain in the zone of a hip joint when the
limb is turning inside (pronation). The doctor discovered the injury of the tendon of one of the
muscles which can rotate medially the leg. Which muscle is this?
A. * m. gluteus medius
B. m. piriformis
C. m. oburatorius internus
D. m. obturatorius externus

E. m. quadratus femoris
1050 A patient with neuritis of femoral nerve has disturbed flexion of thigh as well as disturbed crus
extension in the knee joint. What muscle's function is disturbed?
A. * Quadriceps muscle femoris
B. Biceps muscle femoris
C. Triceps muscle surae
D. Semitendinous muscle
E. Semimembranous muscle
1051 A patient can not extend knee joint, there is no knee-jerk reflex, skin sensitivity of the anterior
femoral surface is disturbed. What nerve structures are damaged?
A. * Femoral nerve
B. Superior gluteal nerve
C. Tibial nerve
D. Obturatory nerve
E. Inferior gluteal nerve
1052 A woman consulted a doctor about swelling and tenderness of the lower extremity, swollen veins and
nodes on the medial surface of thigh. Which vein was affected?
A. * Greater saphenous
B. Lesser saphenous
C. Femoral
D. Popliteal
E. Tibial
1053 A woman suffering from osteochondrosis felt acute pain in her humeral articulation that became
stronger when she abducts the arm. These symptoms might be caused by damage of the following
nerve:
A. * Axillary nerve
B. Subscapular nerve
C. Dorsal scapular nerve
D. Subclavicular nerve
E. Throracodorsal nerve
1054 A woman underwent an operation of ovaries. In course of the operation the surgeon should ligate the
branches that supply the ovary of the following arteries:
A. * Uterine and ovarian
B. Superior cystic and ovarian
C. Inferior cystic and ovarian
D. Uterine and superior cystic
E. Uterine and inferior cystic
1055 After a road accident a driver was delivered to the hospital with an injury of the medial epicondyle of
humerus. What nerve might be damaged in this case?
A. * n. ulnaris
B. n. radialis
C. n. axillaris
D. n. muscolocutaneus
E. n. medianus
1056 After resection of the middle third of femoral artery obliterated by a thrombus, the lower extremity is
supplied with blood due to the surgical bypass. Name an artery that is the biggest branch of femoral
artery:
A. * Deep femoral artery

B. Superficial circumflexa ilei artery


C. Descending genicular artery
D. Superficial epigastric artery
E. Deep external pudendal artery
1057 After trauma, a 44-year-old patient had a rupture of left palmar interossei muscles. After operation
and removal of the most part of the necrotically changed muscular tissue, the bloodstream was
normalized. Which vessels can help for the restoration of bloodstream?
A. * Arcus palmaris profundus
B. Arcus palmaris superficialis
C. Aa. digitales palmares communes
D. Aa. metacarpeae palmares
E. Aa. perforantes
1058 During the fetal period of the development, the vascular system of the fetus contains large arterial
(Botali) duct that converts into lig. arteriosum after birth. What anatomical objects does this duct
communicate?
A. * Pulmonary trunk and aorta
B. Right and left auricles
C. Aorta and inferior vena cava
D. Pulmonary trunk and superior vena cava
E. Aorta and superior vena cava
1059 Examination of a 2-year-old child revealed physical developmental violation, the child often suffered
from pneumonia. The child was diagnosed with nonclosure of ductus arteriosus. This hemodynamic
disorder was caused by the communication of the following vessels:
A. * Aorta and pulmonary trunk
B. Pulmonary trunk and pulmonary veins
C. Superior cava and aorta
D. Superior cava and pulmonary trunk
E. Aorta and pulmonary veins
1060 On examination of a road accident victim a doctor revealed left clavicle fracture and disturbed blood
circulation in an extremity (no pulsing of radial artery). What cause of blood circulation disturbance
is the most probable?
A. * Compression of subclavian artery
B. Compression of axillary artery
C. Compression of subclavian vein
D. Compression of vertebral artery
E. Compression of axillary vein
1061 The victim has elbow joint trauma with avulsion of medial epicondyle of the humerus. What nerve
can be damaged in this trauma?
A. * Ulnar
B. Radial
C. Musculocutaneous nerve
D. Cardiac cutaneous nerve
E. Medial cutaneous nerve of forearm
1062 Where should the catheter be inserted for evacuation of the lymph from the thoracic lymph duct?
A. * To the left venous angle
B. To the right venous angle
C. To the superior vena cava
D. To the inferior vena cava

E. To the left inguinal vein


1063 While palpating mammary gland of a patient a doctor revealed an induration in form of a node in the
inferior medial quadrant of the gland. Metastases may extend to the following lymph nodes:
A. * Parasternal
B. Posterior mediastinal
C. Deep lateral cervical
D. Bronchopulmonary
E. Superior phrenic
1064 During an operation in the area of the armpit, a surgeon has to define an arterial vessel surrounded by
three fascicles of brachial plexus. Which artery is it?
A. * A. axillaris
B. A. vertebralis
C. A. transversa colli
D. A. profunda brachii
E. A. subscapularis
1065 A 25 year old patient was examined by a medical board. Examination revealed pathology of chest.
Transverse dimensions were to small and the sternum was strongly protruding. What chest type is it?
A. * Keeled chest
B. Funnel chest
C. Flat chest
D. Cylindrical chest
E. Barrel chest
1066 A 35 year old man with a trauma of his left hand was admitted to the traumatologic department.
Objectively: cut wound of palmar surface of left hand; patient can not bend the middle phalanges of
II–V fingers. Which muscles are damaged?
A. * Flexor digitorum superficialis
B. Flexor digitorum profundus
C. Lumbrical muscles
D. Palmar interossei muscles
E. Dorsal interossei muscles
1067 A 38-year-old patient came to a traumatologic center and complained for an injury of his right hand.
Objectively the patient has a cut wound and can not flex the distal phalanx of the I finger. What
muscle was injured?
A. * flexor pollicis longus
B. flexor pollicis brevis
C. abductor pollicis brevis
D. Opponens pollicis
E. Abductor pollicis
1068 A 5-year-old child suffers from the neck deformity. Clinical examination revealed such symptoms:
apparent flexion of the head to the left, his face is turned right, passive bending of the head to the
right is limited. What muscle's development was disturbed in this case?
A. * Sternocleidomastoid
B. Trapezius
C. Splenius muscle of head
D. Sternohyoid
E. Long muscle of head

1069 A 5-year-old child was admitted to the hospital with suppurative inflammation of middle ear
(tympanitis). It began with the inflammation of nasopharynx. Which canal of temporal bone did the
infection get into tympanic cavity?
A. * Musculotubal canal
B. Small canal of chorda tympani
C. Caroticotympanic foramina
D. Small tympanic canal
E. Carotid canal
1070 A 55-year-old patient was hospitalized in result of the trauma of the medial group of femoral
muscles. What kind of movements is the patient unable to do?
A. * Adduction of femur
B. Abduction of femur
C. Flexion of femur
D. Extension of femur
E. Suppination of femur
1071 A 69 year old patient has got an abscess of frontal lobe as a result of purulent infection in nasal
cavity. What anatomical formation did the infection penetrate through?
A. * Foraminae cribrosae
B. Foramen ovale
C. Foramen ethmoidalae posterior
D. Foramen sphenopalatinum
E. Foramen rotundum
1072 A 6-year-old child fell on the cutting object and traumatized soft tissues between shafts of the tibia
and fibula. What kind of bone connection was injured?
A. * Membrane
B. Suture
C. Ligament
D. Fontanel
E. Gomphosis
1073 A boxer who got a punch in the region of temporomandibular joint has a traumatic dislocation of
mandible. Displacement of which articular surfaces overstep the limits of anatomical norm?
A. * Head of mandible and mandibular fossa
B. Coronoid process and pterygoid fossa
C. Coronoid process and submandibular fossa
D. Head of mandible and submandibular fossa
E. Neck of mandible and submandibular fossa
1074 A man with an injury of the dorsal area of his neck was admitted to the resuscitation department.
What muscle occupies this area?
A. * M.trapezius
B. M.sternocleidomastoideus
C. M.latissimus dorsi
D. M.rhomboideus minor
E. M.scalenus anterior
1075 A patient was arrived to the surgery department with dislocation of temporomandibular joint and
injury of its main ligament that covers the joint externally. Name this ligament:
A. * Lateral
B. Mandibular
C. Styloid-mandibular

D. Pterygoid-mandibular
E. Medial
1076 A patient complained about being unable to adduct and abduct fingers in the metacarpophalangeal
articulations towards and away from the 3rd finger. Which muscles' function is impaired?
A. * Interossei muscles
B. Lumbrical muscles
C. Brevis flexor of fingers
D. Long flexor of fingers
E. Extensor digitorum
1077 A patient displays abnormal posterior position of his lower jaw as a result of trauma in the region of
the coronal process of the mandible. What muscle is most likely to be damaged?
A. * M.temporalis
B. M.masseter
C. M.pterygoideus lateralis
D. M.pterygoideus medialis
E. M.levator anguli oris
1078 A patient has a deep cut wound on the posterior surface of his arm in middle third portion. Which
muscle might be injured?
A. * Triceps brachii muscle
B. Biceps brachii muscle
C. Anconeus muscle
D. Brachial muscle
E. Coracobrachial muscle
1079 A patient has an inflammation in the pterygopalatine fossa. The infection has spread into the nasal
cavity. Which anatomical structure has the infection spread through?
A. * Foramen sphenopalatinum
B. Foramen rotundum
C. Canalis palatinus major
D. Canalis palatinus minor
E. Canalis pterygoideus
1080 A patient has been diagnosed with a compressive fracture of a lumbar vertebra. As a result he has a
considerable increase in curvature of the lumbar lordosis. Which ligament weakness can induce such
changes in the spine curvature?
A. * Anterior longitudinal ligament
B. Inguinal ligament
C. Flava ligament
D. Iliolumbar ligament
E. Interspinous ligament
1081 A patient has difficulties with the movement of the hand. Examination revealed inflammation of
common synovial sheath of flexor muscles. It is known from the patient's anamnesis that he got a
stab wound of the finger a week ago. Which finger was most probably damaged?
A. * Digitus minimus
B. Pollex
C. Digitus medius
D. Index
E. Digitus anularis
1082 A patient presents with dysfunction of shin muscles. He cannot raise his body by standing on tiptoe.
Which muscle is affected?

A. * M. triceps surae
B. M. tibialis posterior
C. M. extensor digitorum longus
D. M. flexor digitorum longus
E. M. tibialis anterior
1083 A patient was admitted to the surgical department with inguinal hernia. During the operation the
surgeon performs plastic surgery on posterior wall of inguinal canal. What structure forms this wall?
A. * Transverse fascia
B. Aponeurosis of abdominal external oblique muscle
C. Inguinal ligament
D. Loose inferior edge of transverse abdominal muscle
E. Peritoneum
1084 A patient with a knife wound in the left lumbar part was delivered to the emergency hospital. In
course of operation, a surgeon found that internal organs were not damaged but the knife injured one
of the muscles of renal muscular bed. What muscle is it?
A. * Psoas major muscle
B. Iliac muscle
C. Erector spine
D. Rectus abdominis muscle
E. Abdominal external oblique muscle
1085 A patient with neuritis of femoral nerve has disturbed flexion of thigh as well as disturbed leg
extension in the knee joint. What muscle's function is disturbed?
A. * Quadriceps femoris muscle
B. Biceps femoris muscle
C. Triceps femoris muscle
D. Semitendinous muscle
E. Semimembranous muscle
1086 A victim of a road accident has an abruption of a part of the angle, displacement of fragment
backward and upwards. What ligament is responsible for this displacement?
A. * Stylo-mandibular
B. Intraarticular
C. Lateral
D. Spheno-mandibular
E. Pterygo-mandibular
1087 After a trauma a patient lost ability of elbow extension. This might have been caused by dysfunction
of the following muscle:
A. * m. triceps brachii
B. m. subscapularis
C. m. teres major
D. m. infraspinatus
E. m. levator scapulae
1088 An eye trauma caused soft tissues infection of orbit. Which anatomical formation can the infection
penetrate into the middle cranial fossa?
A. * Through the superior orbital fissure
B. Through the anterior ethmoidal foramen
C. Through the posterior ethmoidal foramen
D. Through the inferior orbital fissure

E. Through the zygomatic orbital foramen


1089 An old woman was hospitalized with acute pain, edema in the right hip joint; the movements in the
joint are limited. Which bone or part of it was broken?
A. * The neck of the femur
B. The body of the femur
C. Condyle of the femur
D. Pubic bone
E. Ischial bone
1090 As a result of an accident, a patient has intense painfulness and edema of the anterior leg surface;
dorsal flexion of the foot is hindered. Function of which muscle of the leg is most likely be
disturbed?
A. * M.tibialis anterior
B. M.flexor digitorum longus
C. M.flexor hallucis longus
D. M.peroneus longus
E. M.peroneus brevis
1091 Chronic rhinitis was complicated by inflammation of frontal sinus. What nasal meatus did the
infection get into frontal sinus?
A. * Middle
B. Inferior
C. Common
D. Superior
E. Nasopharyngeal
1092 During manipulations aimed at treatment of mandible dislocation a physician should pay particular
attention to a muscle that pulls a capsule and interarticular disc of temporomandibular articulation
anteriorly. Which muscle is it?
A. * M. pterygoideus lateralis
B. M. masseter
C. M. pterygoideus medialis
D. M. temporalis
E. M. mylohyoideus
1093 During the operation on the hip joint of a 5-year-old child her ligament was damaged which caused
bleeding. What ligament was damaged?
A. * Capitis femoris
B. Perpendicular of the acetabule
C. Iliofemoral
D. Pubofemoral
E. Ischiofemoral
1094 Examination of a 6-month-old child revealed a delay in closure of the occipital fontanelle. When
should it normally close?
A. * Until 3 months
B. Before the child is born
C. Until 6 months
D. Until the end of the first year of life
E. Until the end of the second year of life
1095 Examination of a patient revealed an abscess of pterygopalatine fossa. Where can the infection
spread?
A. * To the orbit

B. To the ethmoid cells


C. To the frontal sinus
D. To the tympanic cavity
1096 It is necessary to take the cerebrospinal fluid from a patient with suspected inflammation of brain
tunics. Diagnostic puncture was performed between the arches of the lumbar vertebras. During the
puncture the needle went through the following ligament:
A. * Flava
B. Iliolumbar
C. Anterior longitudinal
D. Posterior longitudinal
E. Intertransverse
1097 The operative dentistry department admitted a newborn girl who choked during sucking.
Examination revealed cleft palate arising from non-union of the middle frontal process and maxillary
process of the I-st branchial arch. The cleft was located in the palate between:
A. * Os incisivum et processus palatinus maxillae
B. Processus palatinus maxillae dextrae et sinistrae
C. Lamina horizontalis os palatinum dextrum et sinistrum
D. Processus palatinus maxillae et lamina horizontalis os palatinum
E. In the region of canalis incisivus
1098 Three separate bones connected with cartilage in the area of pelvis cavity are noticed on the X-ray of
the pelvis. What are these bones?
A. * Iliac, pubic, sciatic
B. Pubic, sciatic, femoral
C. Sciatic, femoral, sacral
D. Iliac, sacral, coccyx
E. Sacral, pubic, coccyx
1099 Trauma of occipital region of head resulted in crack fracture in the region of transverse sinus. What
part of occipital bone is damaged?
A. * Squama
B. Left lateral
C. Right lateral
D. Proximal
E. Condyle
1100 Under development of which parts of facial skeleton in the embryonal period is the reason for such a
malformation as cleft palate?
A. * Palatine processes
B. Frontal processes
C. Frontal and maxillary processes
D. Mandibular processes
E. Mandibular and palatine processes
1101 Usually the intravenous injection is done into median cubital vein because it is slightly movable due
to fixation by the soft tissues. What does it fix in the cubital fossa?
A. * Aponeurosis of biceps muscle
B. Tendon of the triceps muscle
C. Brachial muscle
D. Brachioradial muscle
E. Anconeus muscle

1102 When processing a molar tooth with a dental cutter the dentist has by accident deeply wounded the
patient's cheek and damaged not only the mucosa but also a muscle. Which muscle was hurt?
A. * Buccinator muscle
B. Zygomatic major muscle
C. Masetter muscle
D. Orbicularis oris muscle
E. Mylohyoid muscle
1103 X-ray examination revealed an accumulation of suppuration in maxillary sinus. Into what nasal
meatus excretes the suppuration?
A. * Median nasal
B. Nasopharyngeal
C. Inferior nasal
D. Superior nasal
E. Common nasal
1104 Young man felt sharp pain in the back during active tightening on the horizontal bar. Objectively:
pain while moving of upper extremity, limited pronation and adduction functions. Sprain of what
muscle is presented?
A. * М. latissimus dorsi
B. М. levator scapulae
C. М. romboideus major
D. М. trapezius
E. М. subscapularis
1105 ?Doctor analised X-ray picture of the heart. He found pathology on the right contour of the cardio-
vessel shadow. What parts of the heart formed it?
A. Right atrium and right ventricle.
B. Pulmonary trunk and right atrium.
C. Pulmonary trunk and right ventricle.
D. * Ascending aorta and right atrium.
E. Ascending aorta and right ventricle.
1106 During auscultation of the heart it was diagnosed doublening of the first tone on the level of cartilage
of 5-th right rib. Pathology of what vale listen physician?
A. * Tricuspid valve.
B. Mitral valve.
C. Aortic valve.
D. Pulmonary trunk valve.
E. Vena cava superior valve
1107 During auscultation of the heart it was diagnosed diastolic sound in the second intercostal space on
the right parasternal line. Pathology of what vale listen physician?
A. Tricuspid valve.
B. Mitral valve.
C. * Aortic valve.
D. Pulmonary trunk valve.
E. Vena cava superior valve
1108 During auscultation of the heart it was diagnosed diastolic sound in the second intercostal space on
the left parasternal line. Pathology of what vale listen physician?
A. Tricuspid valve.
B. Mitral valve.
C. Aortic valve.

D. * Pulmonary trunk valve.


E. Vena cava superior valve
1109 During auscultation of the patient physician listen sound in the projection of the heart apex.
Insufficiency of what valve can be suspect?
A. Aortic valve.
B. Tricuspid valve.
C. Pulmonary trunk valve.
D. * Mitral valve.
E. Valve of vena cava inferior.
1110 During investigation of 45-year old patient it was diagnosed insufficiency of tricuspid valve. Where
is it location?
A. Between left atrium and left ventricle.
B. * Between right atrium and right ventricle.
C. Aorta ostium.
D. Pulmonary trunk ostium.
E. Coronary sinus ostium.
1111 During ultrasound investigation of 1,5 year child it was found nonclosure of the oval opening, which
was confirmed clinically. In what part of the heart can be find this defect?
A. Interventricular septum.
B. * Interatrial septum.
C. Right atrioventricular septum.
D. Left atrioventricular septum.
E. Apex of the heart.
1112 On X-ray picture of the chest in the frontal direct position doctor diagnosed dilation of the left
contour of middle shadow in it low part. What part of the heart is increased?
A. Auricle of the left atrium.
B. Right ventricle.
C. Left atrium.
D. Right atrium.
E. * Left ventricle.
1113 Physician inspects the patient with insufficiency of systemic circulation. Which chamber of the heart
receives blood from systemic circulation?
A. * Right atrium.
B. Left atrium.
C. Coronary sinus.
D. Right ventricle.
E. Left ventricle.
1114 Physician inspects the patient with insufficiency of systemic circulation. Which chamber of the heart
sends the blood to systemic circulation?
A. Left atrium.
B. * Left ventricle.
C. Right atrium.
D. Right ventricle.
E. Coronary sinus.
1115 Physician inspects the patient with insufficiency of pulmonary circulation. Which chamber of the
heart sends the blood to pulmonary circulation?
A. Right atrium.
B. * Right ventricle.

C. Left atrium.
D. Left ventricle.
E. Coronary sinus.
1116 Physician inspects the patient with insufficiency of circulation. Which vessel carries the oxygenated
blood?
A. Pulmonary trunk.
B. Coronary sinus.
C. Venule.
D. * Brachiocephalic trunk.
E. Brachiocephalic vein.
1117 The investigation of patient show violation in blood supply of frontal part interventricular septum of
the heart. What artery has bad function?
A. * R. interventriculares anterior.
B. R. interventriculares posterior.
C. Right coronary artery.
D. Left coronary artery.
E. R. circumflex.
1118 The patient has myocardial infarction back wall of the right ventricle. Branches of what artery are
thrombosed?
A. Left subclavian artery.
B. Left coronary artery.
C. Left and right coronary artery.
D. Right subclavian artery.
E. * Right coronary artery.
1119 The patient has myocardial infarction frontal wall of the left ventricle. Branches of what artery are
thrombosed?
A. Anterior ventricular branches of right coronary artery.
B. * Anterior interventricular branche of left coronary artery.
C. Circumflex branche of left coronary artery.
D. Left marginal branche of the left coronary artery.
E. Atrioventricular branches of the left coronary artery.
1120 The physician inspected the patient with insufficiency of pulmonary circulation. Which chamber of
the heart receives blood from pulmonary circulation?
A. Right atrium.
B. Left ventricle.
C. Right ventricle.
D. * Left atrium.
E. Coronary sinus.
1121 The physician listening to the heart tones and discovered a violation of the tricuspid valve. At what
place did he auscultate it?
A. At the apex of the heart.
B. * Near the xiphoid processus.
C. Near the join of the 5th costal cartilages with the sternum to the right.
D. In the second intercostal space to the left of the sternum.
E. In the second intercostals space to the right of the sternum.
1122 The physitian listening to the heart tones and discovered a violation of the pulmonary trunk valve. At
what place did he auscultate it?
A. At the apex of the heart.

B. Near the xiphoid processus.


C. Near the join of the 5th costal cartilages with the sternum.
D. * In the second intercostals space to the left of the sternum.
E. In the second intercostals space to the right of the sternum.
1123 To the cardiologic department come patient with violation of venous drainage from back walls of
both ventricles. Violation of blood flow in what vein is it?
A. V. cordis parva.
B. V. cordis magna.
C. Vv. cordis minimae.
D. V. obliqua atria sinistri.
E. * V. cordis media. v. obliqua atria sinistri.
1124 To the cardiologic department come patient with violation of venous drainage from back walls of
both ventricles. Violation of blood flow in what vein is it?
A. V. cordis parva.
B. V. cordis magna.
C. Vv. cordis minimae.
D. * V. cordis media. v. obliqua atria sinistri.
E. V. obliqua atria sinistri.
1125 To the cardiologic department come patient with violation of blood supply of the left atrium. Changes
in perfusion of what artery bring to this pathology?
A. A. coronaria dextra.
B. R. interventriculares anterior.
C. R. circumflexus.
D. R. interventriculares posterior.
E. * Rr. Interventriculares.
1126 To the hospital come 63-year old patient for coronarografy (investigation of blood flow in heart
arteries). Radiologist must know on what branches divided left coronary artery?
A. * R. interventriculares anterior et r. circumflexus.
B. R. interventriculares anterior et r. ascendens.
C. R. interventriculares posterior et r. descendens.
D. R. interventriculares anterior et posterior.
E. R. ventriculi dexter et sinister.
1127 When physician listening to the heart tones he discovered a violation of bicuspidal(mitral) valve. At
what place did he auscultate it?
A. * At the apex of the heart.
B. Near the xiphoid processus.
C. Near the join of the 5th costal cartilages with the sternum.
D. In the second intercostal space to the left of the sternum.
E. In the second intercostal space to the right of the sternum.
1128 When the physitian listening to the heart tones he discovered a violation of the aortic valve. At what
place he auscultate it?
A. Near the xiphoid processus.
B. At the apex of the heart.
C. * In the second intercostals space to the right of the sternum.
D. Near the join of the 5th costal cartilages with the sternum.
E. In the second intercostals space to the left of the sternum.
1129 -th year old patient has abscess at the medial corner of the eye. After two days she was delivered to
the neuro-surgical department with thrombose of cavernose sinus. Infection come through:

A. V. maxillaris.
B. * V. angularis.
C. V. profunda faciei.
D. V. transversa faciei.
E. V. diploicae frontalis.
1130 After course of chemotherapy patient has decrease of all blood cells (pancitopenia). What organ was
damaged?
A. Yellow bone marrow.
B. Spleen.
C. Liver.
D. * Red bone marrow.
E. Thymus.
1131 During investigation of the patient on CT picture thrombosis of celiac trunk was diagnosted. What
organ will get decreased blood supplying?
A. Sigmoid colon.
B. Transverse colon.
C. Cecum.
D. * Stomach.
E. Ileum.
1132 During investigation of the patient on CT picture thrombosis of celiac trunk was diagnosteD. What
organ will get decreased blood supplying?
A. Sigmoid colon.
B. Transverse colon.
C. Cecum.
D. Ileum.
E. * Gallbladder.
1133 During investigation of the patient on CT picture thrombosis of celiac trunk was diagnosted. What
organ will get decreased blood supplying?
A. * Liver.
B. Transverse colon.
C. Cecum.
D. Ileum.
E. Sigmoid colon.
1134 During investigation of the patient on CT picture thrombosis of inferior mesenteric artery was
diagnosted. What organ will get decreased blood supplying?
A. Stomach.
B. * Colon.
C. Duodenum.
D. Ileum.
E. Appendix.
1135 During investigation of the patient on CT picture thrombosis of inferior mesenteric artery was
diagnosted. What organ will get decreased blood supplying?
A. Stomach.
B. Transverse colon.
C. * Sigmoid colon.
D. Spleen.
E. Liver.

1136 During investigation of the patient on CT picture thrombosis of superior mesenteric artery was
diagnosted. What organ will get decreased blood supplying?
A. Stomach,
B. Duodenum.
C. Sigmoid colon.
D. * Ileum.
E. Liver.
1137 During investigation of the patient on CT picture thrombosis of superior mesenteric artery was
diagnosted. What organ will get decreased blood supplying?
A. Stomach.
B. Duodenum.
C. Sigmoid colon.
D. Spleen.
E. * Transverse colon.
1138 During operation of extraction of the tumor in front of the tragus of auricle surgeon found vein. It
was:
A. V. facialis.
B. V. jugularis interna.
C. V. jugularis externa.
D. V. auricularis posterior.
E. * V. retromandibularis.
1139 During operation on the thyroid glande after opening of interaponeurotic suprasternal space some
vessel can be damaget it is:
A. A. thyroid inferior.
B. V. jugularis externa.
C. V. subclavia.
D. Jugular venous arch.
E. * A. thyroid superior.
1140 For extraction of 27-th tooth stomatologist made tuberal anaesthesia. Next day patient has
haematoma of soft tissues at paraauricular region. What vessels were damaged?
A. V. facialis dextra.
B. V. facialis sinistra.
C. V. retromandibularis.
D. Plexus pterygoideus dexter.
E. * Plexus pterygoideus sinister.
1141 Physician inspects the patient with insufficiency of circulation. Which vessels carries the oxygenated
blood?
A. Brachiocephalic vein.
B. Pulmonary trunk.
C. * Celiac trunk.
D. Capillary.
E. Venule.
1142 The 45 years old patient has liver cirrhosis. System of what vein was damaged?
A. V. cavae inferioris.
B. V. cavae superioris.
C. * V. portae hepatis.
D. V. azygos

E. V. hemiaxygos.
1143 The man 70 years old during shaving cut small abscess of the skin in the mastoid region. After two
days he was deliverd to the hospital with inflammation of the layers of the brain. Infection come to
the skull cavity through:
A. V. stylomastoideae.
B. V. labirinthi.
C. V. tympanici.
D. * V. emissaria mastoideae.
E. V. auriculares.
1144 The patiens has varicose veins on the left leg. Which superficial veins make drainage of this region?
A. V. safena parva and v. femoralis profunda.
B. * V. safena magna and v. safena parva.
C. V. poplitea and v. supervicialis subcutaneus.
D. V. tibialis and v. safena magna.
E. V. tibialis posterior and v. safena parva.
1145 The patient undergo operation of spleenectomy. What consequences can be after this operation?
A. * Diseases of blood system.
B. Frequent stool.
C. Frequent uresis.
D. Diseases of kidney.
E. Fractures of the bones.
1146 The patient 45 years old has varicose veins on the anteriomedial surface of the left leg. Which vein
drainage blood from this region?
A. * Safena magna.
B. Safena parva.
C. Lateral subcutaneous.
D. Medial subcutaneous.
E. Medial.
1147 The patient 67 years old has an inflammation of adipose body of the orbit, thromboflebitis of v.
ophtalmicae was developed and sinus cavernosus was damaged. Through what opening v.
ophtalmicae come to this sinus.
A. Fissure orbitalis inferior.
B. * Fissure orbitalis superior.
C. Canalis opticum.
D. Foramen ovale.
E. Foramen rotundum.
1148 The patient 68 years old after extraction of second low big tooth, meningitis was developed. Phisician
ascertain that infection come to dura mater through:
A. Plexux pterygoideus.
B. V. jugularis externA.
C. V. jugularis anterior.
D. V. facialis.
E. * V. retromandibularis.
1149 The patient has cancer of some organ in thoracic cavity. Tumor developes very quickly, because
lymph from this organ get out directly to the thoracic duct. It is:
A. * Esophagus.
B. Lung.

C. Trachea.
D. Heart.
E. Thymus.
1150 The patient has inflammation in the hip region (postinjection abscess). What lymphatic nodes will be
enlarge?
A. Paratracheal.
B. Ander knee.
C. Posterior cervical.
D. * Superficial and profound inguinal.
E. Submandibular.
1151 The patient was delivered to neuro-surgeon with trauma of spinal cord. Which nervous rami form
intercostal nerves?
A. * T1-T11.
B. C1-C4.
C. C5-T1.
D. T12-S4.
E. S5-Co1.
1152 The patient was delivered to neuro-surgeon with trauma of spinal cord. Which nervous rami form
subcostal nerves?
A. C1-C4.
B. * T12.
C. C5-T1.
D. T1—T11.
E. T12-S4.
1153 The patient with trauma of carotid triangle and massive bleeding of dark color was delivered to the
surgical department. What vessel damaged?
A. * V. jugularis interna.
B. V. jugularis externa.
C. A. carotis interna.
D. A. facialis.
E. V. facialis.
1154 The 50 years old patient has dilated paraumbilical veins. Which of the following does not take part in
porto-systemic anastomoses?
A. Gastric veins.
B. Vertebral plexus.
C. Veins around umbilicus.
D. Lower part of rectum.
E. * Vein at upper 1/3 of esophagus.
1155 The 50 years old patient has dilated veins of esophagus. Which of the following does not take part in
cava-caval anastomoses?
A. * Mesenteric veins.
B. Vertebral plexus.
C. Azygos vein.
D. Hemiazygos vein.
E. Lumbal veins.
1156 The 54 years old patient has dilated veins of esophagus. Which of the following does not take part in
cava-caval anastomoses?
A. * Veins of the stomach.

B. Vertebral plexus.
C. Azygos vein.
D. Hemiazygos vein.
E. Lumbal veins.
1157 The 56 years old patient has deficiency of blood supply of pelvic organs. Which artery supplies to
anal canal and anus?
A. * Internal pudental.
B. Internal iliaC.
C. Middle rectal.
D. Inferior vecical.
E. Obturator
1158 The 60 years old patient has dilated paraumbilical veins. Which of the following does not take part in
porto-systemic anastomoses?
A. Gastric veins.
B. Portal veins.
C. Veins around umbilicus.
D. Lower part of rectum.
E. * Ascending lumbal veins.
1159 The 62 years old patient has dilated veins of esophagus. Which of the following does not take part in
cava-caval anastomoses?
A. * Veins of the rectum.
B. Vertebral plexus.
C. Azygos vein.
D. Hemiazygos vein.
E. Lumbal veins.
1160 The 63 years old patient has deficiency of blood supply of pelvic organs. Which artery supplies to
transition of sigmoid colon to rectum?
A. A. obturatoria.
B. Inferior vesical.
C. Middle rectal.
D. Inferior rectal.
E. * Superior rectal.
1161 The 66 years old patient has dilated paraumbilical veins. Which of the following does not take part in
porto-systemic anastomoses?
A. Gastric veins.
B. Portal veins.
C. Veins around umbilicus.
D. Lower part of rectum.
E. * Azygos vein.
1162 The 67 years old patient has dilated veins of esophagus. Which of the following does not take part in
cava-caval anastomoses?
A. * Esophageal veins.
B. Vertebral plexus.
C. Azygos vein.
D. Hemiazygos vein.
E. Lumbal veins.
1163 The 70 years old patient has dilated paraumbilical veins. Which of the following does not take part in
porto-systemic anastomoses?

A. Gastric veins.
B. Portal veins.
C. Veins around umbilicus.
D. Lower part of rectum.
E. * Hemiazygos vein.
1164 The 70 years old patient has dilated paraumbilical veins. Which of the following does not take part in
porto-systemic anastomoses?
A. Gastric veins.
B. Portal veins.
C. Veins around umbilicus.
D. Lower part of rectum.
E. * Vertebral plexus.
1165 The CT picture show deficiency of blood supply of pelvic organs. Which artery supplies part of
rectum above the anal canal?
A. Internal pudental.
B. * Middle rectal.
C. Inferior rectal.
D. Inferior vesical.
E. Obturator.
1166 The CT picture show thrombosis in the joining of the inferior mesenteric and portal vein. This veins
joints by:
A. * Joining the splenic vein.
B. Joining the superior mesenteric vein.
C. Directly entering the portal vein.
D. Joining theу left branch of portal vein.
E. Joining the right branch of portal vein.
1167 The CT picture show thrombosis of the right suprarenal vein. Where does it empty?
A. Right renal vein.
B. Portal vein.
C. Lumbal veins.
D. * Inferior vena cava.
E. Left renal vein.
1168 The patient has deficiency of primary blood supply to perineum. Which artery supplies this structure?

A. * Internal pudental artery.


B. Urethral artery.
C. Perineal artery.
D. Inferior rectal artery.
E. Middle rectal artery.
1169 The patient has liver cirrhosis. Venous drainage of liver into vena cava inferior is trough:
A. Azygos vein.
B. Inferior mesenteric vein.
C. Superior mesenteric vein.
D. Portal vein.
E. * Hepatic veins
1170 The physician diagnosed that patient has blocking in system of portal vein. From what organs drains
the venous blood this vein?

A. * Stomach.
B. Liver.
C. Kidney.
D. Adrenal gland.
E. Abdominal wall.
1171 The physician diagnosed that patient has blocking in system of portal vein. From what organs drains
the venous blood this vein?
A. * Spleen.
B. Liver.
C. Kidney.
D. Adrenal gland.
E. Abdominal wall.
1172 The physician diagnosed that patient has blocking in system of portal vein. From what organs drains
the venous blood this vein?
A. * Small intestinum.
B. Liver.
C. Kidney.
D. Adrenal gland.
E. Abdominal wall.
1173 The physician diagnosed that patient has blocking in system of portal vein. From what organs drains
the venous blood this vein?
A. * Large intestinum.
B. Liver.
C. Kidney.
D. Adrenal gland.
E. Abdominal wall.
1174 The physician diagnosed that patient has blocking in system of portal vein. From what organs drains
the venous blood this vein?
A. * Colon.
B. Liver.
C. Kidney.
D. Adrenal gland.
E. Abdominal wall.
1175 The physician diagnosed that patient has blocking in system of portal vein. From what organs drains
the venous blood this vein?
A. * Caecum.
B. Liver.
C. Kidney.
D. Adrenal gland.
E. Abdominal wall.
1176 The physician diagnosed that patient has blocking in system of portal vein. From what organs drains
the venous blood this vein?
A. * Duodenum.
B. Liver.
C. Kidney.
D. Adrenal gland.
E. Abdominal wall.

1177 The physician diagnosed that patient has blocking in system of portal vein. From what organs drains
the venous blood this vein?
A. * Ileum.
B. Liver.
C. Kidney.
D. Adrenal gland.
E. Abdominal wall.
1178 The physician diagnosed that patient has blocking in system of portal vein. From what organs drains
the venous blood this vein?
A. * Jejunum.
B. Liver.
C. Kidney.
D. Adrenal gland.
E. Abdominal wall.
1179 During operation surgeon found inflammation of appendix. To what lymphatic nodes flow out lymph
from this organ?
A. Nodi lumbales.
B. Nodi mesocolici.
C. * Nodi iliocolici.
D. Nodi phrenici inferiors.
E. Nodi glutealis.
1180 During operation surgeon found some lymphatic nodes that form ring round the organ. Which nodes
they are?
A. * Gastric cardia.
B. Duodenojejunal junction.
C. Gastric pylorus.
D. Gastric body.
E. Duodenal bulb.
1181 During operation surgeon found tumor of ovarium. To what anatomic structure flow out lymph from
ovarii?
A. V. iliaca interna.
B. * Truncus lumbales.
C. Truncus intestinales.
D. Plexus limphaticus.
E. Ductus limphaticus dexter.
1182 During operation surgeon found tumor of stomach. To what lymphatic nodes flow out lymph from
fundus of stomach?
A. Nodi gastric dextri.
B. Nodi mesocolici.
C. Nodi gastroomentalis.
D. * Annulus limphaticus cardie.
E. Nodi gastroomentales sinistri.
1183 On the CT picture surgeon found tumor of abdominal part of esophagus. To what lymphatic nodes
can flow lymph from this structure?
A. Nodi igastrici dextri.
B. Nodi mesocolici.
C. Nodi ileocolici.
D. Nodi phrenici inferiors.

E. * Nodi gastric sinistri.


1184 Surgeon making operation and cut tissues near cisterna chili. What vessels empty to this structure?
A. Jugular trunk.
B. Thoracic duct.
C. Bronchomediastinal trunk.
D. * Lumbal trunk.
E. Right lymphatic duct.
1185 Surgeon making operation and cut tissues near cisterna chili. What vessels empty to this structure?
A. Jugular trunk.
B. Thoracic duct.
C. Bronchomediastinal trunk.
D. * Intestinal trunk.
E. Right lymphatic duct.
1186 Surgeon making operation and cut tissues near cisterna chili. What vessels start from this structure?
A. Jugular trunk.
B. * Thoracic duct.
C. Bronchomediastinal trunk.
D. Lumbal trunk.
E. Right lymphatic duct.
1187 The patient has abscess on the lateral surface of abdomen wall. To what lymphatic nodes flow out
lymph from aabdominal wall?
A. * Nodi epigastrici inferiors.
B. Nodi retrocavales.
C. Nodi sacrales.
D. Nodi obturatorii.
E. Nodi glutealis.
1188 The patient has chronic cholecyscitis. To what lymphatic nodes come lymph from this organ?
A. Nodi paravesicales.
B. Nodi retrocavales.
C. * Nodus foraminalis.
D. Nodi mesenterici superior.
E. Nodi mesenterici inferior.
1189 The patient has chronic inflammation of kidney. To what lymphatic nodes come lymph from this
organ?
A. Nodi paravesicales.
B. Nodi retrocavales.
C. Nodus foraminalis.
D. Nodi mesenterici superior.
E. * Nodi mesenterici inferior.
1190 The patient has chronic pain in the epigastric region. What organsare innervated by celiac plexus?
A. * Liver.
B. Cecum.
C. Ascending colon.
D. Ileum.
E. Kidney.
1191 The patient has chronic pain in the epigastric region. What organsare innervated by celiac plexus?
A. Cecum.

B. * Spleen.
C. Ascending colon.
D. Ileum.
E. Kidney.
1192 The patient has chronic pain in the epigastric region. What organs are innervated by superior
mesenteric plexus?
A. * Cecum.
B. Spleen.
C. Ascending colon.
D. Adrenal gland.
E. Kidney.
1193 The patient suffering from pain in left part of abdomen region. What organs are innervated by inferior
mesenteric plexus?
A. Sigmoid colon.
B. Transverse colon.
C. Ascending colon.
D. Rectum.
E. * Descending colon.
1194 The patient suffering from pain in lower part of abdomen region. What organs are innervated by
pelvic splanchnic nerves?
A. Urinary bladder.
B. Transverse colon.
C. Ascending colon.
D. Kidney.
E. * Uterus.
1195 The patient suffering from pain in lower part of abdomen region. What organs are innervated by
pelvic splanchnic nerves?
A. * Urinary bladder.
B. Transverse colon.
C. Ascending colon.
D. Kidney.
E. Gallbladder.
1196 The patient suffering from pain in lower part of abdomen region. What organs are innervated by
inferior hypogastric plexus?
A. * Urinary bladder.
B. Transverse colon.
C. Ascending colon.
D. Kidney.
E. Gallbladder.
1197 The patient suffering from pain in lower part of abdomen region. What organs are innervated by
inferior hypogastric plexus?
A. * Uterus.
B. Transverse colon.
C. Ascending colon.
D. Kidney.
E. Gallbladder.
1198 The patient suffering from pain in lower part of abdomen region. What organs are innervated by
inferior hypogastric plexus?

A. * Vagina.
B. Transverse colon.
C. Ascending colon.
D. Kidney.
E. Gallbladder.
1199 The patient suffering from pain in lower part of abdomen region. What organs are innervated by
inferior hypogastric plexus?
A. Sigmoid colon.
B. Transverse colon.
C. Ascending colon.
D. * Prostate.
E. Gallbladder.
1200 The patient suffering from pain in lower part of abdomen region. What organs are innervated by
inferior mesenteric plexus?
A. * Sigmoid colon.
B. Transverse colon.
C. Ascending colon.
D. Rectum.
E. Gallbladder.
1201 The patient suffering from pain in lower part of abdomen region. What organs are innervated by
inferior mesenteric plexus?
A. * Sigmoid colon.
B. Transverse colon.
C. Ascending colon.
D. Ampule of rectum.
E. Gallbladder.
1202 The patient suffering from pain in lower part of abdomen region. What organs are innervated by
pelvic splanchnic nerves?
A. * Sigmoid colon.
B. Transverse colon.
C. Ascending colon.
D. Rectum.
E. Gallbladder.
1203 The patient suffering from pain in the anal region. What organs are innervated by inferior hypogastric
plexus?
A. * Rectum.
B. Transverse colon.
C. Ascending colon.
D. Kidney.
E. Gallbladder.
1204 Physician checks the pulse rate on lower lateral portion of forearm. Which artery passes there?
A. Brachial artery.
B. * Radial artery.
C. Axillary artery.
D. Ulnar artery.
E. Anterior interosseus artery.

1205 Physician inspects forearm of the patient. Which vessels are most numerous on anterior surface of
forearm?
A. Ulnar veins.
B. * Intermediate antebrachial veins.
C. Basilica veins.
D. Cephalic veins.
E. Intermediate cubital veins.
1206 Physician inspects the hand. Which arteries anastomose with common digital arteries?
A. Dorsal carpal arteries.
B. Dorsal metacarpal arteries.
C. * Palmar metacarpal arteries.
D. Proper digital arteries.
E. Carpal arteries.
1207 Physician inspects the hand. Which arteries branch off from deep palmar arch??
A. Dorsal carpal arteries.
B. Dorsal metacarpal arteries.
C. * Palmar metacarpal arteries.
D. Carpal arteries.
E. Proper digital arteries.
1208 Physician inspects the hand. Which arteries branch off from superficial palmar arch??
A. Dorsal carpal arteries.
B. Dorsal metacarpal arteries.
C. Palmar metacarpal arteries.
D. * Common digital arteries.
E. Carpal arteries.
1209 Physician inspects the hand. Which arteries branch off from dorsal carpal arch??
A. Dorsal carpal arteries.
B. Carpal arteries
C. Palmar metacarpal arteries.
D. Common digital arteries.
E. * Dorsal metacarpal arteries.
1210 Physician inspects the hand. Which arteries branch off from common digital arteries??
A. * Proper digital arteries.
B. Carpal arteries.
C. Palmar metacarpal arteries.
D. Common digital arteries.
E. Dorsal metacarpal arteries.
1211 Physician inspects the hand. Which arteries branch off from dorsal metacarpal arteries??
A. * Proper digital arteries.
B. Carpal arteries.
C. Palmar metacarpal arteries.
D. Common digital arteries.
E. Dorsal metacarpal arteries.
1212 Physician inspects the palmar surface of the hand. Which artery forms main part of superficial palmar
arch?
A. * Ulnar artery.
B. Metacarpal arteries.
C. Radial artery.
D. Anterior interosseus artery.
E. Posterior interosseua artery.
1213 Physician inspects the palmar surface of the hand. Which artery forms main part of deep palmar
arch?
A. Ulnar artery.
B. Metacarpal arteries.
C. * Radial artery.
D. Anterior interosseus artery.
E. Posterior interosseua artery.
1214 Physician inspects the patient with insufficiency of circulation. Which vessels carries the oxygenated
blood?
A. Pulmonary trunk.
B. Pulmonary artery.
C. Capillary.
D. Venule.
E. * Thyreocervical trunk.
1215 Physician inspects the patient with insufficiency of circulation. Which vessels carries the oxygenated
blood?
A. Pulmonary trunk.
B. Pulmonary artery.
C. Capillary.
D. Venule.
E. * Costocervical trunk.
1216 Physician inspects the wrist. Which artery passes in “anatomical snuff-box”?
A. * Radial artery.
B. Brachial artery.
C. Axillary artery.
D. Ulnar artery.
E. Anterior interosseus artery.
1217 Surgeon performing operation of the fracture of acromion scapulae. Which artery anastomose with
circumflexa scapulae artery?
A. Anterior circumflexa humeri artery.
B. Posterior circumflexa humeri artery.
C. * Suprascapular artery.
D. Axillary artery.
E. Subscapular artery.
1218 Surgeon performing operation of the fracture of acromion scapulae. Which artery gives off
suprascapular artery?
A. Anterior circumflexa humeri artery.
B. Posterior circumflexa humeri artery.
C. Suprascapular artery.
D. * Subscapular artery.
E. Thyreocervical trunk.
1219 The patient delivered to the hospital with the cut trauma of medial part of arm, with the venous
bleeding. What vessels are probably damaged?
A. Cephalic vein.
B. Deep brachial vein.
C. Subscapular vein.
D. Brachial vein.
E. * Basilica vein.
1220 The patient delivered to the hospital with the cut trauma of lateral part of arm, with the venous
bleeding. What vessels are probably damaged?
A. * Cephalic vein.
B. Deep brachial vein.
C. Basilica vein.
D. Subscapular vein.
E. Brachial vein.
1221 The patient delivered to the hospital with the cut trauma of cubital fossa, with the venous bleeding.
What vessels are probably damaged?
A. Cephalic vein.
B. * Intermediate cubital vein.
C. Basilica vein.
D. Subscapular vein.
E. Brachial vein.
1222 The patient with wound of elbow presented to the surgeon. Which artery anastomoses with superior
collateral ulnar artery?
A. * Anterior recureent ulnar artery.
B. Interosseus recurrent artery.
C. Posterior recurrent ulnar artery.
D. Collateral media artery.
E. Collateral radial artery.
1223 The patient with wound of elbow presented to the surgeon. Which artery anastomoses with inferior
collateral ulnar artery?
A. Anterior recureent ulnar artery.
B. Interosseus recurrent artery.
C. * Posterior recurrent ulnar artery.
D. Collateral media artery.
E. Collateral radial artery.
1224 The patient with wound of elbow presented to the surgeon. Which artery anastomoses with collateral
media artery?
A. Anterior recureent ulnar artery.
B. * Interosseus recurrent artery.
C. Posterior recurrent ulnar artery.
D. Collateral media artery.
E. Collateral radial artery.
1225 The patient with wound of elbow presented to the surgeon. Which artery anastomoses with collateral
radial artery?
A. Anterior recureent ulnar artery.
B. Interosseus recurrent artery.
C. Posterior recurrent ulnar artery.
D. * Radial recurrent artery.
E. Collateral radial artery.
1226 The patient with wound of elbow presented to the surgeon. Which artery anastomoses with recurrent
radial artery?

A. Anterior recureent ulnar artery.


B. Interosseus recurrent artery.
C. Posterior recurrent ulnar artery.
D. Radial recurrent artery.
E. * Collateral radial artery.
1227 The patient with wound of elbow presented to the surgeon. Which artery anastomoses with recurrent
interosseus artery?
A. * Collateral media artery.
B. Interosseus recurrent artery.
C. Posterior recurrent ulnar artery.
D. Radial recurrent artery.
E. Collateral radial artery.
1228 The patient with wound of elbow presented to the surgeon. Which artery anastomoses with recurrent
ulnar artery?
A. * Collateral media artery.
B. Inferior collateral ulnar artery.
C. Posterior recurrent ulnar artery.
D. Radial recurrent artery.
E. Collateral radial artery.
1229 28 years old patient of trauma department has fracture of distal part of humerus and separation of a
medial epicondyle. At examination is detected delicacy in flection of a hand and medial adduction.
What nerve is damaged?
A. Ulnar nerve.
B. Median nerve.
C. Radial nerve.
D. Musculocutaneus nerve.
E. * Axillar nerve.
1230 After fracture of surgical neck of brachial bone the patient can’t adduct arm. What nerve is damaged?

A. Radial nerve.
B. Median nerve.
C. Musculocutaneus nerve.
D. * Axillary nerve.
E. Ulnar nerve.
1231 After the inflammatory process ill began to mark delicacy at flexion of the 1,2,3 and 4 digits of the
hand, decreasing of a volume of a muscle, increase of the thenar. At examination: 3 and radial
surfaces of the fouth digits are detected pain disturbance and thermoesthesia in a palm surface of 1,2
digits. What nerve is damaged?
A. Radial nerve.
B. * Median nerve.
C. Musculocutaneus nerve.
D. Axillary nerve.
E. Ulnar nerve.
1232 After trauma patient complains for difficulties of abduction of arm. Which nerve supply the
supraspinatus muscle?
A. * Suprascapular nerve.
B. Dorsal scapulae nerve.
C. Subscapular nerve.

D. Long thoracic nerve.


E. Thoracodorsal nerve.
1233 After trauma patient complains for difficulties of adduction of arm. Which nerve supply the
latissimus dorsi muscle?
A. Suprascapular nerve.
B. Dorsal scapulae nerve.
C. Subscapular nerve.
D. Long thoracic nerve.
E. * Thoracodorsal nerve.
1234 After trauma patient complains for difficulties with abduction of arm. Which nerve can be damaged?
A. Pectoral nerves.
B. * Axillary nerve.
C. Subscapular nerve.
D. Long thoracic nerve.
E. Thoracodorsal nerve.
1235 After trauma patient complains for pain during deep inspiration. Which nerve supply the serratus
anterior muscle?
A. Suprascapular nerve.
B. Dorsal scapulae nerve.
C. Subscapular nerve.
D. * Long thoracic nerve.
E. Thoracodorsal nerve.
1236 After trauma the patient can’t flexes forearm in the elbow joint. What nerve is damaged?
A. Radial nerve.
B. Median nerve.
C. * Musculocutaneus nerve.
D. Axillary nerve.
E. Ulnar nerve.
1237 At an inflammation of deep lymph nodes of an inguinal region the surgeon was compelled to open
deeply located gathering. Neverless, after operation in patient there was a complication: was lost
flexion of forearm in a elbow joint and was broke dermal sensitivity of the antero-lateral surface of a
forearm. What of brachial plexus nerve was damaged at operation?
A. Radial nerve.
B. * Musculocutaneus nerve.
C. Ulnar nerve.
D. Median nerve.
E. Cutaneus medial antebrachii nerve.
1238 In ill after a trauma in a region of an ulnar join the handicapping has appeared at a bending 4 and 5
fingers, has decreased in the sizes protrusion of a little finger. At the review is noticed also decrease
pain and thermoesthesia in a 1,5 fingers of a palmar surface and 2,5 digits on a dorsal surface of a
little finger. What nerve is damaged?
A. * Ulnar nerve.
B. Radial nerve.
C. Median nerve.
D. Musculocutaneus nerve.
E. Medial cutaneus nerve of a forearm.
1239 In patient after fracture of the upper third of humerus was advanced the paralis of posterior group of
muscles of an arm and forearm. What nerve is damaged?

A. Ulnar nerve.
B. * Radial nerve.
C. Median nerve.
D. Musculocutaneus nerve.
E. Medial cutaneus nerve of a forearm.
1240 Patient after trauma of anterior surface of the upper third of forearm have affected pronation loosened
palmar flexion of the hand, and affected sensitivity of a skin of the1,2,3 and radialside of 4th digits,
and of dorsal surface of distal and medial phalanxes of a 2,3,4 digits. What nerve is struck at
disturbance of the indicated functions?
A. * Median nerve in the upper third of forearm.
B. Radial nerve.
C. Median nerve.
D. Musculocutaneus nerve.
E. Medial cutaneus nerve of a forearm.
1241 Patient complains for violation of skin sensitivity on medial palmar part of hand. What nerve is
damaged?
A. Radial nerve.
B. Median nerve.
C. Musculocutaneus nerve.
D. Medial cutaneus nerve of a forearm.
E. * Ulnar nerve.
1242 Patient of 30 years old with a cut wound of a forearm has appeared the disturbance of fingers
extension. It testifies about damage of:
A. * Radial nerve.
B. Median nerve.
C. Musculocutaneus nerve.
D. Medial cutaneus nerve of a forearm.
E. Ulnar nerve.
1243 Patient present to neuro-surgeon with trauma of spinal cord. Which nervous rami form brachial
plexus?
A. * Ventral rami 5-T1.
B. Ventral rami C1-4.
C. Ventral rami T12-L4.
D. Ventral rami L5-S4.
E. Ventral rami S5-Co1.
1244 Patient present to neuro-surgeon with trauma of spinal cord. Which nervous rami form cervical
plexus?
A. Ventral rami 5-T1.
B. * Ventral rami C1-4.
C. Ventral rami T12-L4.
D. Ventral rami L5-S4.
E. Ventral rami S5-Co1.
1245 The girl complains of the hindered and morbid extension of a digits and a hand, anesthesia of the skin
of posterior surface of arm and forearm and 1-3 fingers. What nerve is struck?
A. * Radial nerve.
B. Median nerve.
C. Musculocutaneus nerve.
D. Medial cutaneus nerve of a forearm.

E. Ulnar nerve.
1246 The patient can’t adduct right hand after transferred trauma. At examination passive motions are not
limited. Detected atrophy of deltoid muscle. What nerve is damaged?
A. Radial nerve.
B. Median nerve.
C. Musculocutaneus nerve.
D. * Axillary nerve.
E. Ulnar nerve.
1247 The patient complains for violation of skin sensitivity on dorsal lateral part of hand. What nerve is
damaged?
A. N. medianus.
B. N. ulnaris.
C. * N. radialis.
D. N. musculocutaneus.
E. N. cutaneus antebrahii medialis.
1248 The patient complains for violation of skin sensitivity on dorsal medial part of hand. What nerve is
damaged?
A. N. medianus.
B. * N. ulnaris.
C. N. radialis.
D. N. musculocutaneus.
E. N. cutaneus antebrahii medialis.
1249 The patient complains for violation of skin sensitivity on lateral part of forearm. What nerve is
damaged?
A. N. medianus.
B. N. ulnaris.
C. N. radialis.
D. * N. musculocutaneus.
E. N. cutaneus antebrahii medialis.
1250 The patient complains for violation of skin sensitivity on dorsal part of arm. What nerve is damaged?
A. N. medianus.
B. N. ulnaris.
C. * N. radialis.
D. N. musculocutaneus.
E. N. cutaneus antebrahii medialis.
1251 The patient complains for violation of skin sensitivity on dorsal part of forearm. What nerve is
damaged?
A. N. medianus.
B. N. ulnaris.
C. * N. radialis.
D. N. musculocutaneus.
E. N. cutaneus antebrahii medialis.
1252 The patient complains for violation of skin sensitivity on upper lateral part of arm. What nerves
damaged?
A. N. medianus.
B. N. ulnaris.
C. N. radialis.

D. N. musculocutaneus.
E. * N. axillaris.
1253 The patient complains for violation of skin sensitivity on lateral palmar part of hand. What nerve is
damaged?
A. * N. medianus.
B. N. ulnaris.
C. N. radialis.
D. N. musculocutaneus.
E. N. cutaneus antebrahii medialis.
1254 The patient has swelling and pain on the way of vessels on medial surface of femur. What veins are
damaged?
A. Femoral artery.
B. V. safena parva.
C. * V. safena magna.
D. Anterior tibial artery.
E. Posterior tibial artery
1255 After trauma patient came to physician with complains of the pain, blue color and brashing on the
premedial surface of the tibya. What vessel is damaged?
A. Femoral artery.
B. V. safena parva.
C. * V. safena magna.
D. Anterior tibial artery.
E. Posterior tibial artery
1256 After wound in the region of genu articulation surgeon put some stitches on the vessels. What
elements of neuro- vascular bundle locallised in poplitea fossa?
A. Femoral artery, femoral vein, femoral nerve.
B. * Poplitea artery, poplitea vein, ischiatic nerve.
C. Poplitea artery, poplitea vein, poplitea nerve.
D. Poplitea artery, vsafena magna, subcutaneous nerve.
E. Poplitea artery, poplitea vein, femoral nerve.
1257 Man 47-years old complains for deficiency of blood supply of leg. Which artery pases in lacuna
vasorum?
A. * Femoral artery.
B. Arteria mediana plantaris.
C. Poplitea artery.
D. Plantar metatarsal artery.
E. All of these.
1258 Man 47-years old complains for deficiency of blood supply of leg. Which artery pases in adductorial
canal?
A. * Femoral artery.
B. Arteria mediana plantaris.
C. Poplitea artery.
D. Plantar metatarsal artery.
E. All of these.
1259 The patient has the slight swelling on medial surface of thigh, increased of sizes vein. What vein is
pathology marked?
A. * V. safena magna.
B. V. tibialis anterior.

C. V. safena parva.
D. V. cephalica.
E. V. basilica.
1260 The patient has the slight swelling on medial surface of thigh, increased of sizes vein. What vein is
pathology marked?
A. * V. safena magnA.
B. V. tibialis anterior.
C. V. safena parva.
D. V. cephalica.
E. V. basilica.
1261 The patient has the slight swelling on medial surface of leg, increased of sizes vein. What vein is
pathology marked?
A. * V. safena magna.
B. V. tibialis anterior.
C. V. safena parva.
D. V. cephalica.
E. V. basilica.
1262 The patient has the slight swelling on the posterior surface of leg, increased of sizes vein. What vein
is pathology marked?
A. V. safena magna.
B. * V. safena parva.
C. V. jugularis interna.
D. V. cephalica.
E. V. basilica.
1263 The patient has wound on the plantar surface of foot. What vessels formed plantar arterial arches?
A. Lateral and medial anterior maleolar arteries.
B. * Deep plantar, dorsal arteria pedis, lateral plantar artery.
C. Posterior maleolar arteries, medial and lateral.
D. Lateral and medial metatarsal arteries.
E. Arteria arcuata, medial ,lateral maleolar arteries.
1264 The patient suffers from deficiency of blood supply of leg. Which artery supplies frontal femoral
muscles?
A. Obturatory artery.
B. * Femoral artery.
C. Deep femoral artery.
D. Perforating artery.
E. Poplitea artery.
1265 The patient suffers from deficiency of blood supply of leg. Which artery pulsation physician palpate
between medial malleolus and heel?
A. Femoral artery.
B. Fibular artery.
C. Poplitea artery.
D. Anterior tibial artery.
E. * Posterior tibial artery.
1266 The patient suffers from deficiency of blood supply of knee. Which artery does not take part in
articular rete of knee joint?
A. * Peroneal artery.

B. Lateral superior genicular artery.


C. Descending genicular artery.
D. Lateral inferior genicular artery.
E. Medial superior genicular artery.
1267 The patient suffers from deficiency of blood supply of knee. Which artery does not take part in
articular rete of knee joint?
A. * Femoral artery.
B. Lateral superior genicular artery.
C. Descending genicular artery.
D. Lateral inferior genicular artery.
E. Medial superior genicular artery.
1268 The patient suffers from deficiency of blood supply of knee. Which artery does not take part in
articular rete of knee joint?
A. * Deep femoral artery.
B. Lateral superior genicular artery.
C. Descending genicular artery.
D. Lateral inferior genicular artery.
E. Medial superior genicular artery.
1269 The patient suffers from deficiency of blood supply of leg. Which artery pulsation physician palpate
near extensor hallucis in region of ankle joint?
A. Dorsalis pedis artery.
B. Fibular artery.
C. Poplitea artery.
D. * Anterior tibial artery.
E. Posterior tibial artery.
1270 The patient suffers from deficiency of blood supply of leg. Which artery pulsation physician palpate
between medial malleolus and heel?
A. Femoral artery.
B. Fibular artery.
C. Poplitea artery.
D. Anterior tibial artery.
E. * Posterior tibial artery.
1271 The patient suffers from deficiency of blood supply of leg. Which artery passes in musculofibular
canal?
A. Femoral artery.
B. * Fibular artery.
C. Popliteal artery.
D. Anterior tibial artery.
E. Posterior tibial artery.
1272 The patient suffers from deficiency of blood supply of leg. Which artery supplies back femoral
muscles?
A. Obturatory artery.
B. Femoral artery.
C. Deep femoral artery.
D. * Perforating artery.
E. Popliteal artery.
1273 The patient suffers from deficiency of blood supply of leg. Which artery supplies frontal femoral
muscles?

A. Obturatory artery.
B. * Femoral artery.
C. Deep femoral artery.
D. Perforating artery.
E. Popliteal artery.
1274 The patient suffers from deficiency of blood supply of leg. Which artery supplies femoral adductors
muscles?
A. * Obturatory artery.
B. Femoral artery.
C. Deep femoral artery.
D. Perforating artery.
E. Popliteal artery.
1275 The patient, female 55-years old suffers from femoral hernia. Which vessels form lateral wall of
femoral canal?
A. * Femoral vein.
B. Femoral artery.
C. External iliac artery.
D. External iliac vein.
E. Internal iliac artery.
1276 The patient, male 45 years old complains for deficiency of blood supply of leg. Which artery passes
in superior musculofibular canal?
A. * No one artery.
B. Popliteal artery.
C. Fibular artery.
D. Anterior tibial artery.
E. Posterior tibial artery.
1277 The patient, male 45 years old complains for deficiency of blood supply of leg. Which artery passes
in superior musculofibular canal?
A. * No one artery.
B. Popliteal artery.
C. Fibular artery.
D. Anterior tibial artery.
E. Posterior tibial artery.
1278 The patient, male 45 years old complains for deficiency of blood supply of leg. Which artery passes
in femoral triangle?
A. * Femoral artery.
B. Popliteal artery.
C. Common iliac artery.
D. Internal iliac artery.
E. Obturatory artery.
1279 A 65-year old patient got inflammation of ischiorectal fossa and needs the operation. What nerves are
located here?
A. * Pudendal.
B. Obturatory.
C. Inferior gluteal.
D. Superior gluteal.
E. Sciatic.

1280 A man got deep knife wound of anterior leg muscles. What nerve can be damaged?
A. * Peroneus profundus.
B. Cutaneus surae lateralis.
C. Peroneus superficialis.
D. Tibialis.
E. Peroneus communis.
1281 A man got deep knife wound of lateral leg muscles. What nerve can be damaged?
A. Peroneus profundus.
B. Cutaneus surae lateralis.
C. * Peroneus superficialis.
D. Tibialis.
E. Peroneus communis.
1282 In trauma department was delivered patient with a wound of right lumbar region. It was detected
wound of psoas major muscle, he lost capability of leg extension in a knee joint. What nerve is
damaged?
A. * Femoral.
B. Ilioepigastric.
C. Ilioinguinal.
D. Genitofemoral.
E. Obturatorial.
1283 Patient complains for sensitivity and motion deficiency in lower limb. What nerve supplies cremaster
muscle?
A. * Genitofemoral.
B. Anterior cutaneus femoral.
C. Lateral femoral cutaneus.
D. Femoral.
E. Ilioinguinal.
1284 The patient complains for absence of sensitiveness of skin on plantar foot. What nerves needs to be
checked up?
A. * Plantar nerves.
B. Sural nerves.
C. Tibial nerves.
D. Common peroneal nerves.
E. Deep peroneal nerves.
1285 The patient complains for absence of sensitiveness of skin in perineal region. Which nerve is
branching off there?
A. Obturator nerve.
B. * Pudendal nerve.
C. Femoral nerve.
D. Inferior gluteal.
E. Superior gluteal.
1286 The patient complains for difficulties with abduction of the thigh after trauma. Which nerve can be
damaged?
A. * Superior gluteus.
B. Femoral.
C. Sciatic.
D. Obturator.

E. Inferior gluteus.
1287 The patient complains for difficulties with adduction of the thigh after trauma. Which nerve can be
damaged?
A. Superior gluteus.
B. Femoral.
C. Sciatic.
D. * Obturator
E. Inferior gluteus.
1288 The patient complains for difficulties with eversion of the foot after trauma. He cannot lift lateral
edge of foot. Which nerve can be damaged?
A. * Superficial peroneal.
B. Sciatic.
C. Tibial.
D. Deep peroneal.
E. Common peroneal.
1289 The patient complains for difficulties with extension of the thigh after trauma. Which nerve can be
damaged?
A. Femoral.
B. * Sciatic.
C. Obturator.
D. Ilioinguinal.
E. Pudendal.
1290 The patient complains for difficulties with extension of the leg in knee joint after trauma. Which
nerve can be damaged?
A. Obturator.
B. * Femoral.
C. Sciatic.
D. Superior gluteus.
E. Inferior gluteus.
1291 The patient complains for difficulties with extension of the foot in ankle joint after trauma. Which
nerve can be damaged?
A. Obturator.
B. Femoral.
C. Deep peroneal.
D. * Superior gluteus.
E. Inferior gluteus.
1292 The patient complains for difficulties with flexion of the thigh after trauma. Which nerve can be
damaged?
A. * Femoral.
B. Sciatic.
C. Obturator.
D. Ilioinguinal.
E. Pudendal.
1293 The patient complains for difficulties with flexion of the leg in knee joint after trauma. Which nerve
can be damaged?
A. Obturator.
B. * Tibial.
C. Femoral.

D. Superior gluteus.
E. Inferior gluteus.
1294 The patient complains for difficulties with flexion of the foot in ankle joint after trauma. Which nerve
can be damaged?
A. Obturator.
B. Femoral.
C. * Tibial.
D. Superior gluteus.
E. Inferior gluteus.
1295 The patient complains for difficulties with eversion of the foot after trauma. He cannot lift lateral
edge of foot. Which nerve can be damaged?
A. * Superficial peroneal.
B. Sciatic.
C. Tibial.
D. Deep peroneal.
E. Common peroneal.
1296 The patient complains for sensitivity and motion deficiency in lower limb. Which nerve innervated
the skin on medial leg region?
A. * Saphenus nerve.
B. Anterior femoral cutaneus nerve.
C. Cutaneus medial surae nerve.
D. Femoral nerve.
E. Tibial nerve.
1297 The patient complains for sensitivity deficiency in lower limb. Which nerve passes through the
lacuna vasorum?
A. Genital branch.
B. * Femoral branch.
C. Femoral nerve.
D. Ilioinguinal nerve.
E. Pudendus nerve.
1298 The patient complains for sensitivity deficiency in lower limb. Which nerve passes through the
lacuna musculorum?
A. Genital branch.
B. Femoral branch.
C. * Femoral nerve.
D. Ilioinguinal nerve.
E. Pudendus nerve.
1299 The patient complains for sensitivity deficiency in lower limb. Which nerve passes below the anterior
superior iliac spine?
A. * Lateral femoral cutaneus nerve.
B. Anterior femoral cutaneus nerve. posterior femoral cutaneus nerve.
C. Posterior femoral cutaneus nerve.
D. Ilioinguinal nerve.
E. Pudendus nerve.
1300 The patient presented to neuro-surgeon with trauma of spinal cord. Which nervous rami form the
lumbar plexus?
A. * Ventral rami L5-S4.
B. Ventral rami C1-C4.

C. Ventral rami C5-T1.


D. Ventral rami T12-L4.
E. Ventral rami S5-Co1.
1301 The patient presented to neuro-surgeon with trauma of spinal cord. Which nervous rami form the
coccygeal plexus?
A. Ventral rami L5-S4.
B. Ventral rami C1-C4.
C. Ventral rami C5-T1.
D. Ventral rami T12-L4.
E. * Ventral rami S5-Co1.
1302 The patient presented to neuro-surgeon with trauma of spinal cord. Which nervous rami form the
sacral plexus?
A. * Ventral rami L5-S4.
B. Ventral rami C1-C4.
C. Ventral rami C5-T1.
D. Ventral rami T12-L4.
E. Ventral rami S5-Co1.
1303 The patient suffers from sensitivity deficiency in lower limb. Which nerve passes through inguinal
canal?
A. Femoral branch.
B. * Genital branch.
C. Obturator nerve.
D. Ilioinguinal nerve.
E. Pudendus nerve.
1304 The patient has compressive fracture at the beginning of spinal cord. At what vertebra is it?
A. * C1
B. L4
C. C3
D. T11
E. T7
1305 Where the a needle should be inserted for spinal taps?
A. * L3-L4
B. L4-L5
C. L5-S1
D. Th12-L1
E. Th11-Th12
1306 During general anesthesia doctor is blocking the basic functional unit of the nervous system. It is…
A. * Neuron
B. Glial cell
C. White mater and gray mater
D. Portion of spinal cord with pair of spinal nerves
E. Anterior and posterior horns
1307 During general anesthesia doctor is blocking the brain. It is the part of…
A. * Central nervous system
B. Somatic nervous system
C. Autonomic nervous system
D. Portion of spinal cord with pair of spinal nerves

E. Splanchnic nervous system


1308 During spinal anesthesia doctor blocking segment of spinal cord it is…
A. * Portion of spinal cord with pair of spinal nerves
B. Portion of spinal cord with membranes
C. White mater and gray mater
D. Portion of spinal cord between upper and lower spinal roots
E. Anterior and posterior horns
1309 On the CT picture doctor can indicate position of the end of spinal cord on adults in the…
A. * L2 vertebrae
B. L1 vertebrae
C. S1 vertebrae
D. Th 12 vertebrae
E. Th11 vertebrae
1310 On the CT picture doctor can indicate position of the end of spinal cord on child in the…
A. * L3 vertebrae
B. Th 12 vertebrae
C. L1 vertebrae
D. S1 vertebrae
E. L2 vertebrae
1311 On the CT picture doctor can indicate position of the terminal ventricle in the…
A. * Spinal cord
B. Brain steam
C. Midbrain
D. Diencephalon
E. Hind brain
1312 The child 5 years old diagnosed meningitis. At what level punctuate must be done?
A. * L3-L4
B. L2-L3
C. L4-L5
D. L1-L2
E. T12-L1
1313 The doctor neuropathology examine knee reflex the sensory nerve fibers carry information…
A. * Toward the CNS
B. Away from the ANS
C. Away from the CNS to the ANS
D. Away from the visceral nervous system
E. Away from the CNS
1314 The doctor want punctuate spinal canal. At what level it must be done?
A. T4
B. C5
C. T12
D. T8
E. * L2
1315 The patient cannot detect the amount of stretch or tension in skeletal muscles, what receptors not
functioned?
A. * Proprioceptors

B. Pain receptors
C. Heat receptors
D. Cutaneous receptors
E. Away from the CNS
1316 The patient has fever and weakness in foots, doctor suspect meningitis and examine…
A. * Cerebral spinal fluid
B. The subarachnoid space
C. Extra cerebral spinal fluid
D. Fat
E. Blood
1317 The patient has not temperature sensitivity of the legs skin. What is the minimum number of neurons
is blocked?
A. * -2
B. -4
C. -5
D. -1
E. -3
1318 What functions of the central nervous system examine the doctor neuropathology?
A. * All of the above
B. Sending out messages to glands and organs
C. Receiving, processing and interpreting incoming information
D. Sending out messages to muscles
E. None of the above
1319 You see CT picture indicate position of filum terminale in the…
A. * L2-Co2
B. C1-S1
C. C1-L2
D. C2-L2
E. L1-Co1
1320 You see CT picture indicate position of lumbosacral enlargement in the…
A. * Th 12 vertebrae
B. L1 vertebrae
C. S1 vertebrae
D. L2 vertebrae
E. Th11 vertebrae
1321 You see CT picture what you can indicate in central canal of vertebrae?
A. * Spinal cord
B. Brain steam
C. Midbrain
D. Diencephalon
E. Hind brain
1322 After auto accident to the neurological department of hospital delivered the man. Found traumatic
brain injury .Suspected lesion of the hindbrain. Which parts of the brain can be damaged?
A. * pons
B. none of the above
C. hypophysis
D. corpus callosum

E. all correct answers


1323 After auto accident to the neurological department of hospital delivered the man. Found traumatic
brain injury .Suspected lesion of the hindbrain. Which parts of the brain can be damaged?
A. * vermis of cerebellum
B. frontal lobe
C. corpus callosum
D. hypophysis
E. temporal lobe
1324 After auto accident to the neurological department of hospital delivered the man. Found traumatic
brain injury .Suspected lesion of the hindbrain. Which parts of the brain can be damaged?
A. * decussation of pyramids
B. hypophysis
C. frontal lobe
D. corpus callosum
E. temporal lobe
1325 After traumatic brain injury of occipital area of the head to the neurological department of hospital
delivered the man. Suspected lesion of the medulla oblongata. What disorders of the central nervous
system can be detected?
A. * cardiovascular disorders
B. speech defect
C. auditory loss
D. a loss of coordination and balance
E. distorted pain perception
1326 After traumatic brain injury of occipital area of the head to the neurological department of hospital
delivered the man. Suspected lesion of the medulla oblongata. What disorders of the central nervous
system can be detected?
A. * cardiac and vasomotor disorders
B. auditory loss
C. speech defect
D. a loss of coordination and balance
E. distorted pain perception
1327 After traumatic brain injury of occipital area of the head to the neurological department of hospital
delivered the man. Suspected lesion of the medulla oblongata. What disorders of the central nervous
system can be detected?
A. * respiratory rhythmicity disorders
B. auditory loss
C. a loss of coordination and balance
D. speech defect
E. distorted pain perception
1328 After traumatic brain injury of occipital area of the head to the neurological department of hospital
delivered the man. Suspected lesion of the cerebellum. What disorders of the central nervous system
can be detected?
A. * a loss of coordination
B. auditory loss
C. speech defect
D. respiratory rhythmicity disorders
E. distorted pain perception

1329 After traumatic brain injury of occipital area of the head to the neurological department of hospital
delivered the man. Suspected lesion of the cerebellum. What disorders of the central nervous system
can be detected?
A. * a loss of balance
B. speech defect
C. auditory loss
D. respiratory rhythmicity disorders
E. distorted pain perception
1330 After traumatic brain injury patient complains of loss of visual motor response. What part of the brain
may be damaged?
A. * corpora quadrigemina
B. Corpus callosum.
C. cerebellum
D. none of the above
E. medulla oblongata
1331 After traumatic brain injury to the neurological department of hospital delivered the man. Suspected
lesion of the hindbrain. Which parts of the brain can be damaged?
A. * all correct answers
B. cerebellar hemispheres
C. vermis of cerebellum
D. decussation of pyramids
E. pons
1332 After traumatic brain injury to the neurological department of hospital delivered the man. Suspected
lesion of the hindbrain. Which parts of the brain can be damaged?
A. * cerebellar hemispheres
B. hypophysis
C. none of the above
D. corpus callosum
E. all correct answers
1333 During operation neurosurgeon remove some parts of the pons of the brain. What structures may be
destroyed?
A. * All of the above
B. Processes of visual and auditory information
C. Connections of the cerebellum to the brain stem
D. None of the above
E. Processing centers for sensory information
1334 During the examination of the patient of neurological department found violations of hypoglossal
nerve. Indicate the level of location of the nucleus of hypoglossal nerve?
A. * Medulla oblongata
B. The upper segments of the spinal cord;
C. The lower part of the pons;
D. Legs brain;
E. Top of the pons.
1335 During the MRI examination revealed pathological formation in the area of the lower triangle of
rhomboid fossa. What nucleus of cranial nerves can be damaged?
A. * from IX to XII pair
B. from V to VIII pair
C. from V to XII pair

D. from I to IV pair
E. from I to IV pair
1336 During the MRI examination revealed pathological formation in the area of the upper triangle of
rhomboid fossa. What nucleus of cranial nerves can be damaged?
A. * from V to VIII pair
B. from IX to XII pair
C. from V to XII pair
D. from I to IV pair
E. from I to IV pair
1337 During the MRI examination revealed pathological formation in the area of the lateral triangle of
rhomboid fossa. What can be damaged?
A. * Vestibular field
B. Triangle pneumogastric
C. Triangle hyoid nerve
D. Caudal pole
E. Cranial pole
1338 During the MRI examination revealed pathological formation in the area of superior colliculus and
brachium of superior colliculus. The function of what centers can be damaged?
A. * subcortical visual centers
B. olfactory centers
C. subcortical motor centers
D. subcortical auditory centers
E. centers of stereognozy
1339 During the MRI examination revealed pathological formation in the area of trapezoid body. Which
parts of the brain can be damaged?
A. * Pons
B. Rhomboid fossa
C. Cerebellum
D. Isthmus rhomboencephalon
E. Medulla oblongata
1340 During the MRI examination revealed pathological formation in the area of interpeduncular fossa.
Which nerve can be damaged?
A. * 3d cranial nerve
B. 2nd cranial nerve
C. 4th cranial nerve
D. 5th cranial nerve
E. 1st cranial nerve
1341 Following an auto accident, a man with an obvious head injury was observed stumbling about the
scene. An inability to walk properly and loss of balance were quite obvious. What brain area was
involved?
A. * cerebellum
B. occipital lobe of cerebrum
C. temporal lobe of cerebrum
D. medulla oblongata
E. corpus callosum
1342 Following an auto accident, a man with an obvious head injury was observed stumbling about the
scene. An inability to walk properly and loss of balance were qute obvious. What brain area was
involved?

A. * Cerebellum
B. Occipital lobe of cerebrum
C. Corpus callosum
D. Medulla oblongata
E. Temporal lobe of cerebrum
1343 In patient heart rhythm irregularities. What part of the brain may be damaged?
A. * medulla oblongata
B. cerebrum.
C. superior coliculi.
D. none of the above
E. cerebellum
1344 The CT picture indicates hemorrhage in the fourth ventricle. Where it locate?
A. * Rhomboencephalon
B. Mesencephalon
C. Diencephalon
D. Medulla oblongata
E. Isthmus rhomboencephalon
1345 The CT picture indicates hemorrhage in the pyramids region. Where it locate?
A. * Isthmus rhomboencephalon
B. Rhomboid fossa
C. Medulla oblongata
D. Cerebellum
E. Pons
1346 The CT picture show small hemorrhage to the reflex center of the medulla. What center not involve
to the process?
A. Digestive centers
B. Cardiac and vasomotor centers
C. * Respiratory rhythmicity center
D. Cardiovascular center
E. None of the above
1347 The CT picture shows that patient has closed connection between 3-d and 4-th ventricles. What
structure does not function correctly?
A. * Cerebral aqueduct
B. Lateral ventricle
C. Amiculum olive
D. Corpus callosum
E. Tractus pyramidalis
1348 The doctor diagnosed pathology in the cerebellum. What functions will be damage?
A. * Balance
B. Visual activity
C. Speech
D. Binocular vision
E. Emotional behavior
1349 The doctor on CT picture diagnosed pathology in the medulla oblongata. What control functions
except heart rate, blood pressure will be damage?
A. * Rate and depth of breathing
B. Balance

C. B and D
D. Body temperature
E. Emotions
1350 The patient has hemorrhage to the corpora quadrigemina ,it would most like result in…
A. * Loss of visual motor response
B. Speech defect
C. Auditory loss
D. A loss of coordination and balance
E. Distorted pain perception
1351 The patient has hemorrhage to the region of the brain that regulates heart rate I t is located…
A. * Medulla oblongata
B. Cerebrum
C. Cerebullum
D. Superior coliculi
E. Non of the above
1352 The patient has less quantity of cerebrospinal fluid. What structure is response for it production?
A. * Chroid plexus
B. Dura mater
C. Pia mater
D. Dural sinus
E. Arachnoid granulation
1353 The patient has less quantity of cerebrospinal fluid. What functional process will be break?
A. * All of the above
B. Transports nutrients, chemical messengers and waste products
C. Supports of the brain
D. None of the above
E. Cushions neural structures
1354 The patient has partial loss of control wake. Which area of the brain is damaged?
A. * Reticular activating system (RAS)
B. Midbrain
C. All of the above
D. Cerebellum
E. Non of the above
1355 The patient in ICU has problems with respiratory rhythm. Where can be damage of the brain locate?
A. * Mesencephalon
B. Diencephalon
C. Medulla oblongata
D. Isthmus rhomboencephali
E. Rhomboencephalon
1356 The surgeon during operation cut nerve in the interpeduncular fossa. Which number of cranial nerve
it was?
A. * 3-d cranial nerve
B. 2-nd cranial nerve
C. 4-th cranial nerve
D. 5-th cranial nerve
E. 1-st cranial nerve

1357 To the emergency department brought the patients after auto accident during the examination in
patient found heart rhythm irregularities. What part of the brain may be damaged?
A. * medulla oblongata
B. cerebellum
C. cerebrum.
D. superior coliculi.
E. none of the above
1358 After injury of head MRI picture indicate, that patient has some adhesions in the middle frontal
gyrus. What center is located here?
A. * Writing center
B. Hearing area
C. Sensory area
D. Speech area
E. Visual cortex
1359 After injury of the head, subdural hemorrhage and compression of the occipital lobe was detected by
CT investigation. With what this region is associated?
A. * Are concerned with vision
B. Are the most posterior part of the cerebellum
C. Help regulate gross movements performed unconsciously
D. Are located within the white matter of the cerebrum
E. Are the part of peripheral nervous system
1360 After scanning, tumor was detected at the Brocar’s Area in the brain of patient. Abnormalities of
which function may be observe?
A. * Speak
B. Interpret sounds
C. Maintain a state of wakefulness
D. Remember
E. Play a musical instrument
1361 At the clinical researches scientists explored, that anterior pituitary gland receives hypothalamic
relising factors via…
A. * System of vessels in the infundibulum
B. Transport along nerve axons to the adenohypophysis
C. The systemic arterial system
D. Transport through the lymphatic system
E. Direct diffusion through the interstitial spaces
1362 By clinical researches it was explored, that the part of the brain concerned with water balance,
appetite and regulating body temperature is the…
A. * Hypothalamus
B. Medulla oblongata
C. Cerebral cortex
D. Cerebellum
E. Thalamus
1363 During clinical scientific researches of the brain, it was explored ,that general sensory center is
located at…
A. * Thalamus
B. Epithalamus
C. Metathalamus
D. Subthalamic area

E. Hypothalamus
1364 During clinical scientific researches of the brain, it was explored, that the cortical hearing area is
located at…
A. * Superior temporal gyrus
B. Unculus
C. Superior frontal sulcus
D. Insula
E. Calcarine sulcus
1365 During clinical scientific researches of the brain, it was explored, that cortical smell area is locate
at…
A. * Uncus
B. Superior temporal gyrus
C. Insula
D. Superior frontal sulcus
E. Calcarine sulcus
1366 During clinical researches on volunteers it was explored, that next structure plays an important role in
regulating of voluntary movements. It is…
A. * Primary motor gyrus of frontal lobe
B. Basal ganglias
C. Cerebral peduncles
D. Hypotalamus
E. Postcentral gyrus of parietal lobe
1367 During operation it was diagnosed that free circulation of the intracerebral liquid is blocked at the
aqueduct. What formations it communicate?
A. * Fourth and third ventricles
B. Third and second ventricles
C. Central canal of fourth ventricle
D. Central canal and subarachnoid space
E. Fourth ventricle and subarachnoid space
1368 On the MRI picture it is indicate small hemorrhage in the pineal body. Where it is locate?
A. * Epithalamus
B. Thalamus
C. Metathalamus
D. Subthalamic araea
E. Hypothalamus
1369 On the MRI picture of brain doctor can indicate the four distinct lobes of the cortex, they are…
A. * Occipital, parietal, temporal and frontal lobes
B. Front, back, side and top lobes
C. Sensory, auditory, visual and motor lobes
D. None of the above
E. Hind, mid, fore and association lobes
1370 On the MRI picture of brain doctor can indicate the largest part of the diencephalon, it is…
A. * Thalamus
B. Pineal body
C. Hypothalamus
D. Intermediate mass
E. Pituitary gland

1371 On the picture of brain, made by electronic microscope, we can see the most common type of
neurons, which have several dendrites and one axon extending from the cell body .They are…
A. * Multipolar neurons
B. Afferent neurons
C. Pseudounipolar neurons
D. None of the above
E. Interneurons
1372 The patient has concussion of the brain, he complain about dysfunction of verbal communication,
higher intellectual processes, voluntary control of skeletal muscles. Which cerebral lobes are injured?

A. * Frontal
B. Occipital
C. Temporal
D. Parietal
E. Non of the above
1373 The patient has some problems with regulating sensory activity. What region of brain may be
involved in pathologic process?
A. * Somatic ,parietal
B. Voluntary motor, occipital
C. Somatic, temporal
D. Autonomic, parietal
E. Autonomic, temporal
1374 The patient has some problems with regulating consciously movements of skeletal muscles. What
region of brain may be involved in pathologic process?
A. * Primary motor, frontal lobe
B. Primary motor, temporal
C. Primary motor parietal
D. Somatic sensory, parietal
E. Somatic sensory frontal
1375 The scientist explored by clinical researches of the brain, that autonomic center is located at…
A. * Hypothalamus
B. Epithalamus
C. Thalamus
D. Optic chiasma
E. Subthalamic area
1376 The scientist explored by clinical researches of the brain, that visual cortex is located at…
A. * Calcarine sulcus
B. Uncus
C. Insula
D. Superior temporal gyrus
E. Superior frontal sulcus
1377 After clinical scientific researches it was explored that “Pleasure center” and “Punishment center” of
the brain are located in the…
A. * Limbic system
B. Prefrontal area
C. Intermediate mass
D. Precentral gyrus

E. Mammilliary bodies
1378 After examination of CT image of the brain structures, can you say what sentence is true?
A. * The lateral ventricles are two cavities, one in each cerebral hemisphere
B. The fourth ventricle is diamond-shaped expansion of the central canal within the brainstem
C. The third ventricle is a thin chamber lying below the corpus callosum and septum pellucidum
D. None of the above
E. All of the above
1379 After examination of CT image indicate structure that represent structure of lateral ventricle, it is…
A. * Corn anterior
B. Mesencephalon
C. Pons
D. Medulla oblongata
E. Spinal cord
1380 After examination of CT image indicates structures that represent structure of lateral ventricle, it is…
A. * Foramen interventriculare
B. Pons
C. Mesencephalon
D. Spinal cord
E. Medulla oblongata
1381 After examination of CT image indicates structure that represents structure of lateral ventricle, it is…
A. * Cornu frontale
B. Pons
C. Medulla oblongata
D. Mesencephalon
E. Spinal cord
1382 After examination of CT image indicate structure that represent structure of lateral ventricle, it is…
A. * Stria terminalis
B. Pons
C. Spinal cord
D. Medulla oblongata
E. Mesencephalon
1383 After examination of CT image indicate structure that represent structure of lateral ventricle, it is…
A. * Pars centralis
B. Spinal cord
C. Pons
D. Mesencephalon
E. Medulla oblongata
1384 After examination of CT image indicate structure that represent structure of lateral ventricle, it is…
A. * Cornu temporal
B. Mesencephalon
C. Pons
D. Medulla oblongata
E. Spinal cord
1385 After examination of CT image indicate structure that represent structure of lateral ventricle, it is…
A. * Cornu inferius
B. Pons

C. Medulla oblongata
D. Mesencephalon
E. Spinal cord
1386 Indicate, after examination of the MRI picture of the brain, what structures connect corpus callosum?
A. * Right and left cerebral hemispheres
B. Lateral lobes of the cerebellum
C. The calluses on the palms of your hand
D. Cerebellum to the pons and medulla
E. The calluses on the palm of your hands
1387 Indicate, after examination of the MRI picture of the brain, what is the principal connection between
cerebral hemispheres?
A. * The corpus callosum
B. The cerebral peduncles
C. Cerebellum to the pons and medulla
D. The brain system
E. The fornix
1388 Indicate, after examination of the MRI picture of the brain, what structure doesn’t take part in
formation of corpus callosum?
A. * Cortex cerebelli
B. Rostrum
C. Splenium corporis callosi
D. Truncus corpora callosi
E. Radiatio corporis callosi
1389 The patient has problems with motor function of left hand. Where the first neuron (axon) of lateral
corticospinal tract is passes through?
A. * Internal capsule
B. Lentiform nucleus
C. External capsule
D. Extrema capsule
E. Claustrum
1390 The patient has significations of high intracranial pressure. What structure of brain has concern to
production of cerebrospinal fluid?
A. * Choroid plexus
B. Meninges
C. Corpora quadrigmina
D. Crura cerebri
E. Adenohypophysis glands
1391 At patient was diagnosed extra production of cerebrospinal fluid. What structure response about it?
A. * Choroid plexus
B. Arachnoid granulations
C. Spinal cord
D. Pia mater
E. Dural sinus
1392 At the patient was diagnosed “nearsightedness”, it means:
A. * Myopia
B. Presbiopia
C. Hemianopia

D. Hyperopia
E. Emmetropia
1393 During microscopic investigation of eyeball doctor can see that photoreceptors are found in the..
A. * Retina
B. Cornea
C. Choroid coat
D. All the above
E. Sclera
1394 During microscopic investigation of retina doctor can see that ora serrata separate…
A. * Parts of retina
B. Retina and chroidea
C. Chroidea and cornea
D. Iris and ciliary body
E. Choroidea and sclera
1395 During neurosurgical operation doctor cut three connective tissue membranes covering and
protecting CNS structures are the…
A. * Meninges, dura mater
B. Periosteum, dura mater
C. Meninges, arachnoid
D. Endosteum, pia mater
E. Meninges, pia mater
1396 The doctor diagnosed that the patient has blocked drainage of aqueous humor at the canal of
Schlemm, what condition results?
A. * Glaucoma
B. Cataract
C. Iritis
D. Conjunctivitis
E. Blefaritis
1397 The patient cannot see clear picture, focusing the light rays involves which of the following?
A. * Change in the sharp of the lens
B. Change in the sharp of the cornea
C. Change in the relative position of the lens
D. Change in the location of the optic disc
E. Change in the density of the aqueous humor
1398 The patient has an inflammation of delicate membrane that lines the eyelids and covers part of outer
surface of the eyeball, it is…
A. * Conjunctiva ,cornea
B. Conjunctiva, ciliary glands
C. Arachnoid, cornea
D. Conjunctiva, lacrimal apparatus
E. Endothelium, cornea
1399 A 2-year-old child experienced convulsions because of lowering calcium ions concentration in the
blood plasma. Function of what structure is decreased?
A. * Parathyroid glands
B. Hypophysis
C. Adrenal cortex
D. Pineal gland
E. Thymus
1400 During the fetal period of the development in the vascular system of the fetus a large arterial
(Botallo’s) duct is functioning which converts into lig.arteriosum after birth. What anatomical
formations does this duct connect?
A. * Pulmonary trunk and aorta
B. Right and left auricle
C. Aorta and inferior vena cava
D. Pulmonary trunk and superior vena cava
E. Aorta and superior vena cava
1401 A young man felt sharp pain in the back during active climbing. Objectively: pain while moving
upper extremity, limited pronation and adduction functions. Sprain of what muscle can be observed
here?
A. * М.latissimus dorsi
B. М.levator scapulae
C. М.romboideus major
D. М.trapezius
E. М.subscapularis
1402 When a patient with traumatic impairment of the brain was examined, it was discovered that he had
stopped to distinguish displacement of an object on the skin. What part of the brain was damaged?
A. * Posterior central gurus
B. Occipital zone of the cortex
C. Parietal zone of the cortex
D. Frontal central gurus
E. Frontal zone
1403 Some diseases reveal symptoms of aldosteronism with hypertension and edema due to sodium
retention in the organism. What organ of the internal secretion is affected on aldosteronism?
A. * Adrenal glands
B. Testicle
C. Ovaries
D. Pancreas
E. Hypophysis
1404 A patient has a malignisation of thoracic part of esophagus. What lymphatic nodes are regional for
this organ?
A. * Anulus lymphaticus cardiae
B. Nodi lymphatici paratrachealis
C. Nodi lymphatici prevertebralis
D. Nodi lymphatici pericardiales laterales
E. Nodi lymphatici mediastinales posteriors
1405 A 55-year-old patien was hospitalized in result of the trauma of the medial group of femoral muscles.
What kind of movements is the patient unable to do?
A. * Adduction of femur
B. Abduction of femur
C. Flexion of femur
D. Extension of femur
E. Suppination of femur
1406 During the endoscopy the inflammation of a major papilla of the duodenum and the disturbances of
bile secretion were found. In which part of duodenum were the problems found?
A. * Descendent part

B. Ascendant part
C. Bulb
D. Upper horizontal part
E. Lower horizontal part
1407 An 18-year-old patient came to the hospital with the complaints of bleeding trauma in the vestibule of
his nose. On examination: the mechanical injure of the mucous layer of the vestibule without
continuation into proper nasal cavity. What is the boundary between the vestibule and proper nasal
cavity?
A. * Nasal limen
B. Nasal roller
C. Nasal septa
D. Choanes
E. Nostrils
1408 A 32-year-old patient has been diagnosed with bartholinitis (inflammation of Bartholin’s glands). In
what part of the female urogenital system are the Bartholin’s glands located?
A. * The labia major
B. The labia minor
C. The clitoris
D. The vagina
E. The uterus
1409 A 50 year-old patient had hemorrhage of the brain and was taken to the hospital. The place of
hemorrhage was revealed on the lateral hemispheres surfaces during the medical examination. What
artery was injured?
A. * The middle cerebral artery
B. The anterior cerebral artery
C. The posterior cerebral artery
D. The anterior communicating artery
E. The posterior communicating artery
1410 A 30-year-old patient was hospitalized due to bleeding of the facial artery. What place on the face has
to be pressed to stop bleeding?
A. * The mandible’s edge
B. The mental process
C. The mandible’s branch
D. The nose’s back
E. The molar bone
1411 A 19 year-old patient was diagnosed with appendicitis and was hospitalized. The surgical operation
on appendectomy is to be performed. What artery must be fixed to stop bleeding during the surgical
operation?
A. * The ileocolic artery
B. Thecolicadextra
C. The colica media
D. The colica sinistra
E. The iliac
1412 An old woman was hospitalized with acute pain, edema in the right hip joint; the movements in the
joint are limited. Which bone or part of it was broken?
A. * The neck of the femur
B. The body of the femur
C. Condyle of the femur

D. Pubic bone
E. Ischial bone
1413 A 45-year-old man fell on the right knee and felt the acute pain in the joint. On examination: severe
edema on the anterior surface of the knee joint. Crunching sounds are heard while moving the joint.
Which bone is destroyed?
A. * Knee-cap
B. Neck of the femur
C. Left epicondyle of the femur
D. Right epicondyle of the femur h
E. Head of the femur
1414 During the operation on the hip joint of a 5-year-old childher ligament was damaged which caused
bleeding.What ligament was damaged?
A. * Lig. Capitis femoris
B. Perpendicular lig of the acetabule
C. Iliofemoral
D. Pubofemoral
E. Ischiofemoral
1415 A 6-year-old child fell on the cutting object and traumatized soft tissues between tibia and ?bula.
What kind of bone connection was injured?
A. * Membrane
B. Suture
C. Ligament
D. Fontanel
E. Gomphosis
1416 Usually the intravenous injection is done into median cubital vein because it is slightly movable due
to ?xation by the soft tissues. What does it ?x in the cubital fossa?
A. * Aponeurosis of biceps muscle
B. Tendon of the triceps muscle
C. Brachial muscle
D. Brachioradial muscle
E. Anconeus muscle
1417 A patient has elbow joint trauma with avulsion of medial epicondyle of humerus. What nerve can be
damaged in this trauma?
A. * Ulnar
B. Radial
C. Musculocutaneous nerve
D. Cardiac cutaneous nerve
E. Medial cutaneous nerve of forearm
1418 Obturative jaundice developed in a 60-year-old patient because of malignant tumour of the great
papilla of the duodenal. Lumen of what anatomical structure is compressed with tumour?
A. * Hepatopancreatic ampulla
B. Cystic duct
C. Common hepatic duct
D. Right hepatic duct
E. Left hepatic duct
1419 A histological specimen presents parenchymal organ, which has cortex and medulla. Cortex consists
of epitheliocytes with blood capillaries between them; these cells form three zones. Medulla consists
of chromaf?nocytes and venous sinusoids. Which organ has these morphological features?

A. * Adrenal gland
B. Kidney
C. Lymph node
D. Thymus
E. Thyroid
1420 A patient complains of frequent and dif?cult urination. Imperfection of what formation can cause it?
A. * Prostate
B. Testicles
C. Bulbouretral glands
D. Testicles
E. Ductus deference
1421 A 45-year-old man with an arm injuiry came to the trauma unit. The objective data are: there are no
extension, adduction or pronation functions of the arm. What muscle damage caused this condition?
A. * Teres major
B. Subscapular
C. Teres minor
D. Infraspinous
E. Supraspinous
1422 Ovarian tumour was diagnozed in a woman. Surgery should be perfomed. What ligament should be
extracted by the surgeon to disconnect the ovary and the uterus?
A. * The proper ovarial ligament
B. Broad ligament of uterus
C. Lateral umbilical ligament
D. Suspensory ligament of ovary
E. Round ligament of uterus
1423 Where should the catheter for evacuation of the lymph from the thoracic lymph duct is inserted?
A. * To the left venous corner
B. To the right venous corner
C. To the superior vena cava
D. To the inferior vena cava
E. To the left inguinal vein
1424 During complicated labour the pubic symphysis ruptured. What organ can be damaged mostly?
A. * Urinary bladder
B. Rectum
C. Ovary
D. Uterine tubes
E. Uterus
1425 A young man felt acute pain in the back during active push-ups on the horizontal bar. Objectively:
pain while moving upper extremity, reduced pronation and adduction functions. Sprain of what
muscle can be observed here?
A. * m.latissimus dorsi
B. m.levator scapulae
C. m.romboideus major
D. m.trapezius
E. m.subscapularis
1426 Children often have heavy nasal breathing resulting from excessive development of lymphoid tissue
of pharyngeal mucous membrane. What tonsils growth may cause this effect?

A. * Tonsilla pharyngea
B. Tonsilla palatina
C. Tonsilla lingualis
D. Tonsilla tubaria
E. All above mentioned tonsils
1427 When a patient with traumatic impairment of the brain was examined, it was discovered that he had
stopped to distinguish displacement of an object on the skin. What part of the brain was damaged?
A. * Posterior central gurus
B. Occipital zone of the cortex
C. Parietal zone of the cortex
D. Frontal central gurus
E. Frontal zone
1428 A person has steady HR not exceeding 40 bpm. What is the pacemaker of this person’s heart rhythm?

A. * Atrioventricular node
B. Sinoatrial node
C. His’ bundle
D. Branches of His’ bundle
E. Purkinje’s ?bers
1429 During examination of a patient, there was found a neoplasm in the white substance of cerebral
hemispheres with localization in the genu and frontal part of posterior crus of internal capsule. Fibers
of what conductive tract of the brain will be disrupted?
A. * Tr. pyramidalis
B. Tr. frontothalamicus
C. Tr. thalamocorticalis
D. Tr. frontopontinus
E. Tr. Parietooccipitopontinus
1430 Under some diseases it is observed aldosteronism accompanied by hypertension and edema due to
sodium retention in the organism. What organ of the internal secretion is affected under
aldosteronism?
A. * Adrenal glands
B. Testicle
C. Ovaries
D. Pancreas
E. Hypophysis
1431 A patient has tissue ischemia below the knee joint accompanied with intermittent claudicating. What
artery occlusion should be suspected?
A. * Popliteal artery
B. Peroneal artery
C. Posterior tibial artery
D. Anterior tibial artery
E. Proximal part of femoral artery
1432 During the endoscopy the in?ammation of a major papilla of the duodenum and the disturbances of
bile secretion were found. In which part of duodenum were the problems found?
A. * Descendent part
B. Ascendant part
C. Bulb
D. Upper horizontal part

E. Lower horizontal part


1433 An 18-year-old patient came to the out-patient department with the complaints of bleeding trauma in
the vestibule of his nose. On examination: the mechanical injury of the mucous layer of the vestibule
without continuation into nasal cavity proper. What is the boundary between the vestibule and nasal
cavity proper?
A. * Nasal limen
B. Nasal roller
C. Nasal septa
D. Choanes
E. Nostrils
1434 A 50-year-old patient was injured on the occipital region of the head. The closed skull trauma was
diagnosed. She was taken to the hospital. The medical examination showed: deregulation of walking
and balance, trembling of arms. What part of brain was injured?
A. * The cerebellum
B. The medulla oblongata
C. The mind-brain
D. The inter-brain
E. The spinal cord
1435 During the operation on the hip joint of a 5-year-old child her ligament was damaged which caused
bleeding. What ligament was damaged?
A. * The lig. Capitis femoris
B. Tranversum acetabular
C. Iliofemoral
D. Pubofemoral
E. Ischiofemoral
1436 A patient has elbow joint trauma with avulsion of medial epicondyle of humerus. What nerve can be
damaged in this trauma?
A. * Ulnar
B. Radial
C. Musculocutaneous nerve
D. Cardiac cutaneous nerve
E. Medial cutaneous nerve of forearm
1437 As a result of spinal-cord trauma a 33 y.o. man has a disturbed pain and temperature sensitivity that is
caused by damage of the following tract:
A. * Spinothalamic
B. Medial spinocortical
C. Posterior spinocerebellar
D. Lateral spinocortical
E. Anterior spinocerebellar
1438 After a 2 y.o. child has had ?u, there appeared complaints about ear ache. A doctor revealed hearing
impairment and in?ammation of the middle ear. How did the infection penetrate into the middle ear?
A. * Through the auditory tube
B. Through foramen jugularis
C. Through canalis caroticus
D. Through atrium mastoideum
E. Through canalis nasolacrimalis

1439 After resection of the middle third of femoral artery obliterated by a thromb the lower extremity is
supplied with blood due to the surgical bypass. Name an artery that plays the main role in
reestablishment of blood ?ow:
A. * Deep femoral artery
B. Super?cial circum?ex artery of hip bone
C. Descending genicular artery
D. Super?cial epigastric artery
E. Deep external pudendal artery
1440 A patient’s knee joint doesn’t extend, there is no knee-jerk re?ex, and skin sensitivity of the anterior
femoral surface is disturbed. What nerve is damaged?
A. * Femoral nerve
B. Superior gluteal nerve
C. Big ?bular nerve
D. Obturator nerve
E. Inferior gluteal nerve
1441 A 50 y.o. patient was admitted to the hospital with complaints about pain behind his sternum,
asphyxia during physical activity. Angiography revealed pathological changes in the posterior
interventricular branch of the right coronary artery. What heart structures are affected?
A. * Posterior wall of the right and left ventricles
B. Anterior wall of the right and left ventricles
C. Right atrium
D. Right atrioventricular valve
E. Left atrium
1442 As a result of damage to certain structures of brainstem a man lost orientation re?exes. What
structures were damaged?
A. * Quadrigeminal plate
B. Medial nuclei of reticular formation
C. Red nuclei
D. Vestibular nuclei
E. Black substance
1443 A young man consulted a doctor about disturbed urination. Examination of his external genitals
revealed that urethra is split on top and urine runs out of this opening. What anomaly of external
genitals development is the case?
A. * Epispadia
B. Phimosis
C. Hermaphroditism
D. Paraphimosis
E. Hypospadia
1444 According to audiometry data a patient has a disturbed perception of medium frequency sounds. It
might have been caused by a damage of:
A. * Middle part of helix
B. Cochlear nuclei
C. Spiral ganglion
D. Quadrigeminal plate
E. Lateral geniculate bodies
1445 In course of an operation surgeon removed a part of a lung that was ventilated by a tertiary bronchus
accompanied by branches of pulmonary artery and other vessels. What part of a lung was removed?
A. * Bronchopulmonary segment

B. Middle lobe
C. Inferior lobe
D. Superior lobe
E. Pulmonary lobule
1446 Examination of a patient revealed an abscess of pterygopalatine fossa. Where can the infection spread
to unless the disease was cured in time?
A. * To the orbit
B. To the interpterygoid space
C. To the frontal sinus
D. To the subgaleal temporal space
E. To the tympanic cavity
1447 A patient has a haemorrhage into the posterior central gyrus. What type of sensitivity on the opposite
side will be disturbed?
A. * Skin and proprioceptive
B. Visual
C. Auditory
D. Olfactory
E. Auditory and visual
1448 A patient has pain, edema and reddening of his skin in the anterosuperior area of his thigh and his
foot’s thumb. What lymph nodes of his lower extremity responded to the in?ammatory process?
A. * Super?cial inguinal
B. Deep inguinal
C. Internal longitudinal
D. Super?cial longitudinal
E. Popliteal
1449 A patient got a craniocerebral trauma that resulted in right-side convergent strabismus. Damage of
which cranial nerve caused such consequences?
A. * n.abducens
B. n.facialis
C. n.trigeminus
D. n.trochlearis
E. n.aculomotorius
1450 In case of a penetrating wound of the anterior abdominal wall the wound tract is above the lesser
curvature of stomach. What peritoneum formation is most likely to be injured?
A. * Ligamentum hepatogastricum
B. Ligamentum gastrocolicum
C. Ligamentum hepatoduoduodenale
D. Ligamentum hepatorenale
E. Ligamentum triangulare sinistrum
1451 A 35-year-old man has been delivered into a surgical ward with a suppurating wound in the neck,
anterior to trachea (previsceral space). If a surgical operation is not performed urgently, there is a risk
of infection spreading to:
A. * Thoracic cavity - anterior mediastinum
B. Thoracic cavity - middle mediastinum
C. Thoracic cavity - posterior mediastinum
D. Retrovisceral space
E. Interaponeurotic suprasternal space

1452 While examining foot blood supply a doctor checks the pulsation of a large artery running in the
separate ?brous channel in front of articulatio talocruralis between the tendons of long extensor
halluces and digitorum muscles. What artery is it?
A. * A. dorsalis pedis
B. A. tibialis anterior
C. A. tarsea medialis
D. A. tarsea lateralis
E. A. ?bularis
1453 A man was delivered to a traumatological department with a trauma of the right shoulder.
Examination revealed a displaced humeral shaft fracture on the right in the middle one-third of the
humerus; the patient cannot extend the ?ngers of his right hand. What nerve is damaged?
A. * Radial
B. Ulnar
C. Median
D. Musculocutaneous
E. Axillary
1454 Due to destruction of certain structures of the brainstem a women has lost orientation re?exes in
response to strong light stimuli. What structures were destroyed?
A. * Superior colliculi
B. Inferior colliculi
C. Red nuclei
D. Vestibular nuclei
E. Substantia nigra
1455 Angiocardiography of a 60-year-old man revealed constriction of a vessel located in the left coronary
sulcus of the heart. Name this vessel:
A. * Ramus circum?exus
B. Ramus interventricularis posterior
C. A. coronaria dextra
D. V. cordis parva
E. Ramus interventricularis anterior
1456 A patient complains of pain in the right lateral abdominal region. Palpation revealed a dense,
immobile, tumor-like formation. The tumor is likely to be found in the following part of the digestive
tube:
A. * Colon ascendens
B. Colon transversum
C. Colon descendens
D. Colon sigmoideum
E. Caecum
1457 A specimen shows an organ covered with the connective tissue capsule with radiating trabeculae.
There is also cortex containing lymph nodules, and medullary cords made of lymphoid cells. What
organ is under study?
A. * Lymph node
B. Thymus
C. Spleen
D. Red bone marrow
E. Tonsils
1458 After a craniocerebral injury a patient has lost the ability to recognize shapes of objects by touch
(stereognosis). What area of cerebral cortex normally contains the relevant center?

A. * Superior parietal lobule


B. Inferior parietal lobule
C. Supramarginal gyrus
D. Postcentral gyrus
E. Angular gyrus
1459 A patient has been delivered into a surgical ward with an incised wound of the anterior surface of the
shoulder in its lower one-third. Flexing function was disrupted in the shoulder and elbow joints,
which is caused by the damage to the:
A. * Biceps brachii muscle
B. Triceps muscle
C. Anconeus muscle
D. Deltoid muscle
E. Coracobrachial muscle
1460 A surgeon has detected in?ammation of the Meckel’s diverticulum of a patient. It can be located in
the:
A. * Ileum
B. Jejunum
C. Colon
D. Duodenum
E. Sigmoid colon
1461 A patient complains of acute pain attacks in the right lumbar region. During examination the
nephrolithic obturation of the right ureter in the region between its abdominal and pelvic segments
was detected. What anatomical boundary exists between those two segments?
A. * Linea terminalis
B. Linea semilunaris
C. Linea arcuata
D. Linea transversa
E. Linea inguinalis
1462 Examination of a patient revealed hypertrophy and in?ammation of lymphoid tissue, edema of
mucous membrane between palatine arches (acute tonsillitis). What tonsil is normally situated in this
area?
A. * Tonsilla palatina
B. Tonsilla pharyngealis
C. Tonsilla tubaria
D. Tonsilla lingualis
E. –
1463 A specimen of a parenchymal organ shows poorly delineated hexagonal lobules surrounding a central
vein, and the interlobular connective tissue contains embedded triads (an artery, a vein and an
excretory duct). What organ is it?
A. * Liver
B. Pancreas
C. Thymus
D. Spleen
E. Thyroid
1464 A patient had a trauma that caused dysfunction of motor centers regulating activity of head muscles.
In what parts of cerebral cortex can these centers normally be located?
A. * Inferior part of precentral gyrus
B. Superior part of precentral gyrus

C. Supramarginal gyrus
D. Superior parietal lobule
E. Angular gyrus
1465 A patient demonstrates functional loss of nasal halves of the retina. What area of visual pathways is
affected?
A. * Optic chiasm
B. Left optic tract
C. Right optic tract
D. Left optic nerve
E. Right optic nerve
1466 An oncology department has admitted a patient with suspected pulmonary tumor. On examination a
pathology localized within the lower lobe of the right lung was detected. How many
bronchopulmonary segments are there in this lobe?
A. * 5
B. 6
C. 4
D. 3
E. 2
1467 A patient with injury sustained to a part of the central nervous system demonstrates disrupted
coordination and movement amplitude, muscle tremor during volitional movements, poor muscle
tone. What part of the central nervous system was injured?
A. * Cerebellum
B. Medulla oblongata
C. Diencephalon
D. Mesencephalon
E. Prosencephalon
1468 A woman complaining of sharp pain in her lower abdomen has been delivered into an admission
room. A gynecologist on examination makes a provisional diagnosis of extrauterine pregnancy. What
anatomical structure should be punctated to con?rm diagnosis?
A. * Rectouterine pouch
B. Utriculosaccular chamber
C. Recto-vesical pouch
D. Retropubic space
E. Intersigmoidal recess
1469 Parenchyma of an organ is composed of pseudounipolar neurons localized under the capsule of
connective tissue. Central place belongs to nerve ?bers. Name this organ:
A. * Spinal ganglion
B. Sympathetic ganglion
C. Intramural ganglion
D. Nerve trunk
E. Spinal cord
1470 During a surgery for femoral hernia a surgeon operates within the boundaries of femoral trigone.
What structure makes up its upper margin?
A. * Lig. inguinale
B. Arcus iliopectineus
C. Lig. lacunare
D. Lig. pectinale
E. Fascia lata

1471 During appendectomy a patient had the a. appendicularis ligated. This vessel branches from the
following artery:
A. * A. ileocolica
B. A. colica dextra
C. A. colica media
D. A. sigmoidea
E. A. mesenterica inferior
1472 A 29-year-old man with a knife wound of the neck delivered to the hospital with bleeding. During the
initial inspection of the wound a surgeon revealed the injury of a vessel situated along the lateral edge
of the sternocleidomastoid muscle. Specify this vessel:
A. * V. jugularis externa
B. A. carotis externa
C. A. carotis interna
D. V. jugularis interna
E. V. jugularis anterior
1473 A patient addressed to a hospital with complaints of lost sensitivity of the skin of the little ?nger.
What nerve is the most likely to be damaged?
A. * Ulnar
B. Median
C. Radial
D. Musculocutaneous
E. Medial cutaneous nerve of the forearm
1474 Parkinson’s disease is caused by disruption of dopamine synthesis. What brain structure synthesizes
this neurotransmitter?
A. * Substantia nigra
B. Globus pallidus
C. Corpora quadrigemina
D. Red nucleus
E. Hypothalamus
1475 During examination of a patient a doctor should use anatomical division of anterior abdominal wall
into regions for more precise diagnostics. How many regions can abdomen be divided into?
A. * 9
B. 8
C. 6
D. 5
E. 4
1476 A child has a wound located posterior to the mastoid process. Bright red blood ?ows from the wound.
Damaged are the branches of the following artery:
A. * A. occipitalis
B. A. temporalis superior
C. A. maxillaris
D. A. carotis externa
E. A. carotis interna
1477 In course of laparotomy a surgeon revealed gangrenous lesion of descending colon. It was caused by
thrombosis of the following artery:
A. * Sinister colic
B. Median colic
C. Dexter colic

D. Ileocolic
E. Superior mesenteric artery
1478 A 60 year old patient has impaired perception of high-frequency sounds. These changes were caused
by damage of the following auditory analyzer structures:
A. * Cochlea membrane near the oval window
B. Cochlea membrane near the helicotrema
C. Eustachian tube
D. Middle ear muscles
E. Tympanic membrane
1479 In?ammation of the tympanic cavity (purulent otitis media) was complicated by in?ammation of
mastoid process cells. What wall of tympanic cavity did the pus penetrate through?
A. * Posterior
B. Anterior
C. Medial
D. Lateral
E. Superior
1480 A patient caught a cold after which there appeared facial expression disorder. He cannot close his
eyes, raise his eyebrows, bare his teeth. What nerve is damaged?
A. * Facial
B. Vagus
C. Trigeminus
D. Glossopharyngeal
E. Infraorbital
1481 A 70 year old female patient was dignosed with fracture of left femoral neck accompanied by
disruption of ligament of head of femur. The branch of the following artery is damaged:
A. * Obturator
B. Femoral
C. External iliac
D. Inferior gluteal
E. Internal pudendal
1482 A patient complained about being unable to adduct and abduct ?ngers in the metacarpophalangeal
articulations towards and away from the 3rd ?nger. Which muscles’ function is impaired?
A. * Interosseous muscles
B. Lumbrical muscles
C. Flexor digitorum brevis
D. Flexor digitorum longus
E. Extensors
1483 During preparation of a patient to a heart surgery it was necessary to measure pressure in heart
chambers. In one of it pressure varied from 0 mm Hg up to 120 mm Hg within one cardiac cycle.
What heart chamber is it?
A. * Left ventricle
B. Right ventricle
C. Right atrium
D. Left atrium
E. A and B
1484 A patient has difficulties with hand movement. Examination revealed in?ammation of common
synovial sheath of ?exor muscles. It is known from the patient’s anamnesis that he got a stab wound
of ?nger a week ago. Which ?nger was most probably damaged?

A. * Digitus minimus
B. Pollex
C. Digitus medius
D. Index
E. Digitus anularis
1485 Examination of a newborn boy’s genitalia revealed a urethral hiatus that opens on the underside of
his penis. What malformation is it?
A. Hermaphroditism
B. Epispadia
C. Monorchism
D. Cryptorhidism
E. * Hypospadias
1486 A patient complains about impaired evacuatory function of stomach (long term retention of food in
stomach). Examination revealed a tumor of initial part of duodenum. Specify localization of the
tumor:
A. * Pars superior
B. Pars inferior
C. Pars descendens
D. Pars ascendens
E. Flexura duodeni inferior
1487 A patient with cholelithiasis diagnosed with mechanic jaundice. Examination revealed that the stone
was in the common bile duct. What bile-excreting ducts make up the obturated duct?
A. * Ductus hepaticus communis et ductus cysticus
B. Ductus hepaticus dexter et sinister
C. Ductus hepaticus dexter et ductus cysticus
D. Ductus hepaticus sinister et ductus cysticus
E. Ductus hepaticus communis et ductus Choledochus
1488 Re?exes of orientation were lost after trauma of CNS structures of 45 y. o. patient. What structure
was destroyed?
A. * Quadrigeminal plate
B. Red nucleus
C. Lateral vestibular nuclei
D. Black substance
E. Medial reticular nuclei
1489 An isolated cell of human heart automatically generates excitement impulses with frequency of 60
times per minute. This cell was taken from the following heart structure:
A. * Sinoatrial node
B. Atrium
C. Ventricle
D. Atrioventricular node
E. His’ bundle
1490 Examination of a patient with impaired blood coagulation revealed thrombosis of a branch of inferior
mesenteric artery. What bowel segment is damaged?
A. * Colon sigmoideum
B. Ileum
C. Caecum
D. Colon transversum

E. Colon ascendens
1491 Roentgenological examination of skull base bones revealed enlargement of sellar cavity, thinning of
anterior clinoid processes, destruction of different parts, and destruction of different parts of sella
turcica. Such bone destruction might be caused by a tumor of the following endocrines gland:
A. * Hypophysis
B. Epiphysis
C. Thymus gland
D. Adrenal glands
E. Thyroid gland
1492 A patient was admitted to the surgical department with inguinal hernia. During the operation the
surgeon performs plastic surgery on posterior wall of inguinal canal. What structure forms this wall?
A. * Transverse fascia
B. Aponeurosis of abdominal external oblique muscle
C. Inguinal ligament
D. Loose inferior edge of transverse abdominal muscle
E. Peritoneum
1493 A patient got a trauma that caused dysfunction of motor centers regulating activity of head muscles.
In what parts of cerebral cortex is the respective centre normally localized?
A. * Inferior part of precentral gyrus
B. Superior part of precentral gyrus
C. Supramarginal gyrus
D. Superior parietal lobule
E. Angular gyrus
1494 A patient was diagnosed with paralysis of facial and masticatory muscles. The haematoma is inside
the genu of internal capsule. What conduction tract is damaged?
A. * Tr. cortico-nuclearis
B. Tr. cortico-spinalis
C. Tr. cortico-thalamicus
D. Tr. cortico-fronto-pontinus
E. Tr. cortico-temporo-parieto-occipito-pontinus
1495 A foreign body (a button) closed space of the right superior lobar bronchus. What segments of the
right lung won’t be supplied with air?
A. * Apical, posterior, anterior
B. Superior and inferior lingular
C. Apical and posterior basal
D. Apical and median basal
E. Medial and lateral
1496 A patient was diagnosed with bartholinitis (in?ammation of greater vulvovaginal glands). In which
organ of urogenital system are these glands localized?
A. * Labia pudenda major
B. Labia pudenda minor
C. Clitoris
D. Vagina
E. Uterus
1497 Ultrasonic examination of a patient revealed aneurism in the area of aortic arch that caused alteration
of vocal function of larynx. What nerve was constricted?
A. * Recurrent laryngeal
B. Diaphragmatic

C. Superior laryngeal
D. Mandibular
E. Sublingual
1498 One of sections of central nervous system has layers arrangement of neurocytes. Among them there
are cells of the following forms: stellate, fusiform, horizontal, pyramidal. What section of central
nervous system is this structure typical for?
A. * Cortex of cerebrum
B. Spinal cord
C. Cerebellum
D. Medulla oblongata
E. Hypothalamus
1499 Surgical approach to the thyroid gland from the transverse (collar) approach involves opening of
interaponeurotic suprasternal space. What anatomic structure localized in this space is dangerous to
be damaged?
A. * Jugular venous arch
B. External jugular vein
C. Subclavicular vein
D. Inferior thyroid arthery
E. Superior thyroid arthery
1500 In course of an experiment thalamocortical tracts of an animal were cut. What type of sensory
perception remained intact?
A. * Olfactory
B. Auditory
C. Exteroreceptive
D. Visual
E. Proprioreceptive
1501 Brain tomography revealed a tumor in the region of red nucleus. What part of brain is damaged?
A. * Midbrain
B. Medulla oblongata
C. Cerebellum
D. Interbrain
E. Pons cerebelli
1502 A 25 year old patient was examined by a physician. Examination revealed pathology of chest.
Transverse dimensions were too small and the sternum was strongly protruding. What chest type is
it?
A. * Keeled chest
B. Funnel chest
C. Flat chest
D. Cylindrical chest
E. Barrel chest
1503 A laboratory experiment on a dog was used to study central parts of auditory system. One of the
mesencephalon structures was destroyed. The dog has lost the orienting response to auditory signals.
What structure was destroyed?
A. * Inferior colliculi of corpora quadrigemina
B. Superior colliculi of corpora quadrigemina
C. Substantia nigra
D. Reticular formation nuclei
E. Red nucleus

1504 A surgeon has to ?nd the common hepatic duct during operative intervention for treatment of
concrements in the gall ducts. The common hepatic duct is located between the leaves of:
A. * Hepatoduodenal ligament
B. Hepatogastric ligament
C. Hepatorenal ligament
D. Round ligament of liver
E. Venous ligament
1505 On examination of a newborn boy’s external genitalia a ?ssure in the urethra opening on the inferior
surface of his penis is detected. What male development is it?
A. * Hypospadias
B. Hermaphroditism
C. Epispadia
D. Monorchism
E. Cryptorchidism
1506 As a result of a continuous chronic encephalopathy a patient has developed spontaneous motions and
disorder of trunk muscles tone. These are the symptoms of the disorder of the following conduction
tract:
A. * Tractus rubrospinalis
B. Tractus corticospinalis
C. Tractus corticonuclearis
D. Tractus spinothalamicus
E. Tractus tectospinalis
1507 Angiocardiography of a 60-year-old man revealed constriction of the vessel located in the left
coronary sulcus of his heart. Name this pathological vessel:
A. * Ramus circum?exus
B. Ramus interventricularis posterior
C. A. coronaria dextra
D. V. cordis parva
E. Ramus interventricularis anterior
1508 A patient complains of pain in the right lateral abdominal region. Palpation revealed a dense,
immobile, tumor-like formation. The tumor is likely to be found in the following part of the digestive
tube:
A. * Colon ascendens
B. Colon transversum
C. Colon descendens
D. Colon sigmoideum
E. Caecum
1509 When playing a child received a hit to the sternum region. As a result of this trauma an organ located
behind the sternum was damaged. Name this organ:
A. * Thymus
B. Thyroid gland
C. Heart
D. Pericardium
E. Larynx
1510 A doctor examined a patient, studied the blood analyses, and reached a conclusion, that peripheral
immune organs are affected. What organs are the most likely to be affected?
A. * Tonsils
B. Thymus

C. Kidneys
D. Red bone marrow
E. Yellow bone marrow
1511 A doctor asked a patient to make a deep exhalation after a normal inhalation. What muscles contract
during such exhalation?
A. * Abdominal muscles
B. External intercostal muscles
C. Diaphragm
D. Trapezius muscles
E. Pectoral muscles
1512 A specimen of a parenchymal organ shows poorly delineated hexagonal lobules surrounding a central
vein, and the interlobular connective tissue contains embedded triads (an artery, a vein and an
excretory duct).What organ is it?
A. * Liver
B. Pancreas
C. Thymus
D. Spleen
E. Thyroid
1513 A patient had a trauma that caused dysfunction of motor centres regulating activity of head muscles.
In what parts of cerebral cortex can the respective centre normally be located?
A. * Inferior part of precentral gyrus
B. Superior part of precentral gyrus
C. Supramarginal gyrus
D. Superior parietal lobule
E. Angular gyrus
1514 Examination of a 6-month-old child revealed a delay in closure of the occipital fontanelle. When
should it normally close?
A. * Before 3 months
B. Before the child is born
C. Before 6 months
D. Before the end of the ?rst year of life
E. Before the end of the second year of life
1515 During a surgery for femoral hernia a surgeon operates within the boundaries of femoral trigone.
What structure makes up its upper margin?
A. * Lig. inguinale
B. Arcus iliopectineus
C. Lig. lacunare
D. Lig. pectinale
E. Fascia lata
1516 An obstetrician-gynecologist measures pelvis size of a pregnant woman. A caliper was used to
measure the distance between the two iliac crests. What measurement of large pelvis was made?
A. * Distantia cristarum
B. Distantia throchanterica
C. Distantia spinarum
D. Conjugata vera
E. Conjugata anatomica
1517 During appendectomy a patient had the a. appendicularis ligated. This vessel branches from the
following artery:

A. * A. Ileocolica
B. A. colica dextra
C. A. colica media
D. A. sigmoidea
E. A. mesenterica inferior
1518 A patient has a tumor of the orbit tissues behind the eyeball. Disruption of accomodation and pupil
constriction is observed. What anatomical structure is damaged?
A. * Ganglion ciliare
B. N. nasociliaris
C. N. lacrimalis
D. N. opticus
E. N. Trochlearis
1519 A patient has damaged spinal cord white matter in the middle area of the posterior white column,
disrupted proprioceptive sensibility of the lower limb joints and muscles. What ?bers are affected?
A. * Fasciculus gracilis
B. Tr. spinothalamicus lateralis
C. Tr. spinocerebellaris anterior
D. Tr. spinocerebellaris posterior
E. Fasciculus cuneatus
1520 A patient has been found to have a marked dilatation of the veins in the region of anterior abdominal
wall around the navel. This is symptomatic of pressure increase in the following vessel:
A. * V. portae hepatis
B. V. cava superior
C. V. cava inferior
D. V. mesenterica inferior
E. V. mesenterica superior
1521 Parkinson’s disease is caused by disruption of dopamine synthesis. What brain structure synthesizes
this neurotransmitter?
A. * Substantia nigra
B. Globus pallidus
C. Corpora quadrigemina
D. Red nucleus
E. Hypothalamus
1522 A soldier with explosion-caused trauma was delivered to a hospital. Examination revealed his
tympanic membrane to be intact. What defense re?ex prevented the tympanic membrane from
rupturing?
A. * Contraction of m. tensor tympani
B. Relaxation of m. tensor tympani
C. Contraction of m. auricularis arterior
D. Relaxation of m. auricularis arterior
E. Relaxation of m. Stapedins
1523 During surgery performed in abdominal cavity a surgeon observed ligament of liver stretching from
anterior abdominal wall (navel) to inferior surface of liver. What ligament is it?
A. * Round ligament of the liver
B. Falciform ligament of the liver
C. Coronary ligament of the liver
D. Venous ligament of the liver

E. Triangular ligament of the liver


1524 A patient with suspected necrosis of the upper part of abdominal cavity organs was delivered to a
surgical department. This condition is associated with acute circulatory disturbance of the following
vessel:
A. * Tuncus coeliacus
B. A. mesenterica inferior
C. A. mesenterica superior
D. A. iliaca communis
E. A. Renalis
1525 After a trauma of the upper third of the anterior forearm surface a patient presents with dif?cult
pronation, weakening of palmar ?exor muscles and altered skin sensitivity of 1-3 ?ngers. Which
nerve is damaged?
A. * n. medianus
B. n. musculocutaneus
C. n. ulnaris
D. n. radialis
E. n. cutaneus antebrachii medialis
1526 A patient has been given high doses of hydrocortisone for a long time. This caused atrophy of one of
the adrenal cortex zones. Which zone is it?
A. * Fascicular
B. Glomerular
C. Reticular
D. Glomerular and reticular
E. All of these
1527 A victim of an accident has bleeding from the soft tissues anterior the mandibular angle. Which
vessel should be ligated for the stop bleeding?
A. * A.facialis
B. A.carotis interna
C. A.temporalis super?cialis
D. A.alveolaris inferior
E. A.lingvalis
1528 A 38-year-old patient came to a traumatology centre and complained about an injury of his right
hand. Objectively: the patient has a cut wound in the region of the thenar eminence on the right hand;
distal phalanx of the I ?nger cannot be ?exed. What muscle was injured?
A. * Fexor pollicis longus muscle
B. Fexor pollicis brevis muscle
C. Abductor pollicis brevis muscle
D. Opponence pollicis muscle
E. Adductor pollicis muscle
1529 An 18-year-old man was delivered to the hospital after a road accident. Examination at the
traumatological department revealed multiple injuries of soft tissues of face in the region of the
medial eye angle. The injuries caused massive haemorrhage. What arterial anastomosis might have
been damaged in this region?
A. * a. carotis externa et a. carotis interna
B. a. carotis externa et a. subclavia
C. a. carotis interna et a. subclavia
D. a. subclavia et a. ophthalmica
E. a. carotis interna et a. ophthalmica

1530 A 75-year-old-female patient with complaints of visual impairment has been delivered to the
ophthalmology department. Objective examination revealed a brain tumor in area of the left optic
tract. The patient has a visual ?eld defect in the following area:
A. * Left half of both eyes retina
B. Right half of both eyes retina
C. Left and right halves of the left eye retina
D. Left and right halves of the right eye retina
E. Left and right halves of both eyes retina
1531 A 35-year-old patient complains about having severe rhinitis and loss of sense of smell for a week.
Objectively: the nasal cavity contains a lot of mucus covering the mucous membrane and blocking
olfactory receptors. In what region of the nasal cavity are these receptors located?
A. * Superior nasal concha
B. Median nasal concha
C. Inferior nasal concha
D. Common nasal meatus
E. Vestibule of nose
1532 While examining the oral cavity a stomatologist revealed in?ammation of papillae on the border of
the median and posterior third of the back of tongue. What papillae are in?amed?
A. * Papillae vallatae
B. Papillae fungiformes
C. Papillae foliatae
D. Papillae ?liformes
E. Papillae conicae
1533 A weightlifter has a disruption of thoracic lymphatic duct as a result of lifting a weight. Choose the
most likely site of injury:
A. * In the region of aortic hiatus
B. In the region of lumbosacral plexus
C. In the posterior mediastinum
D. In the region of venous angle
E. In the region of neck
1534 A woman suffering from osteochondrosis has acute pain in her humeral articulation that gets worse
when she tries to abduct her shoulder. These symptoms might be caused by damage of the following
nerve:
A. * Axillary nerve
B. Subscapular nerve
C. Dorsal scapular nerve
D. Subclavicular nerve
E. Thoracodorsal nerve
1535 A patient complains of hydruria (7 liters per day) and polydipsia. Examination reveals no disorders of
carbohydrate metabolism. These abnormalities might be caused by the dysfunction of the following
endocrine gland:
A. * Neurohypophysis
B. Adenohypophysis
C. Islets of Langerhans (pancreatic islets)
D. Adrenal cortex
E. Adrenal medulla
1536 During ?ghting a man had a cardiac arrest as a result of a hard blow to the upper region of anterior
abdominal wall. Which of the described mechanisms might have provoked the cardiac arrest?

A. * Parasympathetic unconditioned re?exes


B. Sympathetic unconditioned re?exes
C. Parasympathetic conditioned re?exes
D. Sympathetic conditioned re?exes
E. Peripheric re?exes
1537 A patient has been diagnosed with a compression fracture of a lumbar vertebra. As a result he has a
considerable increase in curvature of the lumbar lordosis. Which ligament damage can induce such
changes in the spine curvature?
A. * Anterior longitudinal ligament
B. Posterior longitudinal ligament
C. Yellow ligament
D. Iliolumbar ligament
E. Interspinous ligament
1538 After a craniocerebral trauma a patient lost the ability to execute learned purposeful movements
(apraxia). The injury is most likely localized in the following region of the cerebral cortex:
A. * Gyrus supramarginalis
B. Gyrus angularis
C. Gyrus paracentralis
D. Gyrus lingualis
E. Gyrus parahippocampalis
1539 A man having a hearing loss after a head trauma was delivered to the neurosurgery department. The
cause of the hearing loss might be the damage of the following lobe of cerebral cortex:
A. * Temporal
B. Postcentral gyrus
C. Parietal
D. Occipital
E. Frontal
1540 A man has an increased tonus of extensor muscles. This is the result of intensive information
transmission to the motoneurons of the spinal cord through the following descending pathways:
A. * Vestibulospinal
B. Medial corticospinal
C. Reticulospinal
D. Rubrospinal
E. Lateral corticospinal

Вам также может понравиться